Jump to content

Wikipedia:Reference desk/Science: Difference between revisions

From Wikipedia, the free encyclopedia
Content deleted Content added
→‎molar mass: new section
Line 783: Line 783:
:# The map that Gandalf61 linked to shows that almost everywhere has had an eclipse at some time in the past 1,000 years - over longer periods of time, sooner or later, every point on the earth will have one. Figuring how long it would take for everywhere to get one requires more math than I'm prepared to think about! The fact that neither earth nor moon have perfectly circular orbits - that nothing in the solar system is perfectly lined up - the precession of orbits...it's too complicated for a quick answer. The 1,000 year map suggests that 10,000 to 100,000 years ought to get even the last of the polar regions covered...but it's possible that some odd coincidence of orbital timings might put it out further than that.
:# The map that Gandalf61 linked to shows that almost everywhere has had an eclipse at some time in the past 1,000 years - over longer periods of time, sooner or later, every point on the earth will have one. Figuring how long it would take for everywhere to get one requires more math than I'm prepared to think about! The fact that neither earth nor moon have perfectly circular orbits - that nothing in the solar system is perfectly lined up - the precession of orbits...it's too complicated for a quick answer. The 1,000 year map suggests that 10,000 to 100,000 years ought to get even the last of the polar regions covered...but it's possible that some odd coincidence of orbital timings might put it out further than that.
:[[User:SteveBaker|SteveBaker]] ([[User talk:SteveBaker|talk]]) 14:18, 22 July 2009 (UTC)
:[[User:SteveBaker|SteveBaker]] ([[User talk:SteveBaker|talk]]) 14:18, 22 July 2009 (UTC)

== molar mass ==

i'm stuck with this problem. plz anyone help me!

a sample of gaseous substance weighing 0.5 g occupies a volume of 1.12 litres at NTP. calculate the molar mass of the substance.

as 1 mole of any gas at NTP occupies 22.4 litres, there are 0.5 moles in the sample, and that weighs 0.5 grams. so, 1 mole of the sample will weigh 1g. so this will be the molar mass, won't it be?

(but unfortunately, it's not. my teacher gave me a big zero on the left side of my answer)

Revision as of 16:26, 22 July 2009

Welcome to the science section
of the Wikipedia reference desk.
Select a section:
Want a faster answer?

Main page: Help searching Wikipedia

   

How can I get my question answered?

  • Select the section of the desk that best fits the general topic of your question (see the navigation column to the right).
  • Post your question to only one section, providing a short header that gives the topic of your question.
  • Type '~~~~' (that is, four tilde characters) at the end – this signs and dates your contribution so we know who wrote what and when.
  • Don't post personal contact information – it will be removed. Any answers will be provided here.
  • Please be as specific as possible, and include all relevant context – the usefulness of answers may depend on the context.
  • Note:
    • We don't answer (and may remove) questions that require medical diagnosis or legal advice.
    • We don't answer requests for opinions, predictions or debate.
    • We don't do your homework for you, though we'll help you past the stuck point.
    • We don't conduct original research or provide a free source of ideas, but we'll help you find information you need.



How do I answer a question?

Main page: Wikipedia:Reference desk/Guidelines

  • The best answers address the question directly, and back up facts with wikilinks and links to sources. Do not edit others' comments and do not give any medical or legal advice.
See also:



July 16

The Cancer Genome Atlas

Did they name it that just so the abbreviation would be composed of letters that represent the bases of DNA? or is it just a coinkydink? —Preceding unsigned comment added by 129.64.55.234 (talk) 13:35, 16 July 2009 (UTC)[reply]

I like your explanation. You could be right, it's certainly been noted before. AlmostReadytoFly (talk) 13:43, 16 July 2009 (UTC)[reply]
While I don't see a specific "what's the name mean?" on their site, "TCGA" strikes me as too topical to be a coincidence. I would guess that there's a degree of backronyming at work. — Lomn 13:45, 16 July 2009 (UTC)[reply]
I mean, "Atlas" is an interesting word to use. They could have easily named it The Cancer Genome Project. Maybe they got bored of the word "Project" 129.64.55.234 (talk) 14:02, 16 July 2009 (UTC)[reply]
A "gene atlas" (hmm...no article?) is a common term for this kind of thing. DMacks (talk) 14:58, 16 July 2009 (UTC)[reply]
Charles Stross once used the phrase the Genetic Algorithm That Ate Calcutta. I asked whether the acronym was intentional (this was when Gattaca was fresher in memory); he had not noticed it. —Tamfang (talk) 00:20, 21 July 2009 (UTC)[reply]

Plants listed by oxygen production/toxin removal

I recently watched a Ted Lecture [1] by Kamal Meattle in which he outlines how to create an oxygen neutral building by using 3 varieties of plants that:

-Remove Toxins from the air (Money Plants)

-Provide oxygen during the day (Areca Palms)

-Provide oxygen during the night (Mother-in-law's Tongues)

The specific plants he names might not work for me because of sunlight/space/decor/availablity reasons in my home. Where can I find a list of common household plants in order of oxygen production? Or by Toxin removing ability (I know that can be ambiguous)? How much do these attributes vary from plant to plant (Does it even matter what plants I get?)

I had been under the assumption that oxygen production was based off sun facing surface area of the plant and how much of that sunlight it processes, since the oxygen is a byproduct of photosynthesis, but then it seem strange that plants would produce any oxygen at night, like the Mother-in-law's Tongue, so I am now comptelely lost as to what determines the above attributes.

Any other advice to what plants I can use to improve air quality in my home will be greatly appreciated. Anythingapplied (talk) 20:09, 16 July 2009 (UTC)[reply]

I suspect that guy is a crank and should be ignored. His biography (linked to from that page) doesn't suggest any scientific training and his technique makes no sense. As you say, oxygen is produced by photosynthesis, it can't be produced at night (unless you leave your office lights on over night, I guess). As for removing toxins, as far as I am aware the only "toxin" plants remove from the air is carbon dioxide, also by photosynthesis, and you don't need to worry about that as long as you have reasonable ventilation. --Tango (talk) 21:04, 16 July 2009 (UTC)[reply]
People who give talks at TED (conference) are not usually cranks - and should almost certainly not be ignored. As it happens, it looks like all of his claims have at least some truth to them. Our article on Sansevieria trifasciata (Mother-in-law's tongue) says (with a pretty decent reference) that they remove "certain" toxins from the air and can flourish in low light levels. That may well mean that they can continue to do photosynthesis under artificial lighting - which may be why Meattle thinks they emit oxygen at night...but in complete darkness, that seems impossible. There are three completely different plants called Money plants - but I suspect he means Epipremnum aureum which (according to our article) is good at sucking up all sorts of pollutants. Areca is a family of about 50 different kinds of palm tree - we only have articles on a few of them - and most of those are slim on details - but Areca catechu is claimed to be a popular indoor plant so that's probably what this guy is talking about...however, pretty much all green plants will produce oxygen during the day...so that's an easy choice. SteveBaker (talk) 23:34, 16 July 2009 (UTC)[reply]
Your "decent reference" is a broken link... Given that any half decent ventilation will get plenty of oxygen into a building, I'm still voting crank. Even TED can get fooled. --Tango (talk) 00:19, 17 July 2009 (UTC)[reply]
You are entirely wrong. The reference is to a scientific paper - the link to a copy of the paper on the NASA site is broken - but simply typing the title into Google and accepting the first hit gives you the paper. Did you watch the TED lecture? Ventilating a building is not "oxygen neutral" and it wastes heating/airconditioning energy. The goal of making a building oxygen-neutral has value. SteveBaker (talk) 02:49, 17 July 2009 (UTC)[reply]
Nasa has a couple of pages, this one works [2] and discusses TCE, Benzene and Formaldehyde reduced in a chamber of plants. They don't single out Sansevieria. A place called Stennis Space Center seems to have studied plants for cleaning air and waste water. BTW: Pollutant would probably be a better term than Toxin here. Maybe that's what is causing the confusion. 71.236.26.74 (talk) 00:36, 17 July 2009 (UTC)[reply]
Actually, that's a good ref. The reason why words like "toxin" and "polutant" and things like that are often dismissed is because of their non-specific nature which makes claims which use those terms largely unfalsifiable. If you want to believe a claim, find one that lists specific compounds which are filtered by the plants. Which it looks like you have done. But in general, that's why words like "toxin" are clear bullshit markers. Nothing filters "toxins". Things may filter or remove specific compounds or classes of compounds, but if so then those compounds can be identified, and any serious scientificly-based report on such methods WILL list those compounds. If it doesn't, it's bullshit. --Jayron32.talk.say no to drama 01:48, 17 July 2009 (UTC)[reply]
If anyone could find an explanation of the mechanism somewhere could you post a link? I've found a couple of studies for quantative analyses, but nothing explaining the mechanism so far. 71.236.26.74 (talk) 02:43, 17 July 2009 (UTC)[reply]
Is someone here planning to build a spaceship / nuclear submarine? Cause these are the only two applications I know of where being "oxygen-neutral" would matter at all.76.21.37.87 (talk) 04:42, 17 July 2009 (UTC)[reply]

Thanks guys for all your help. I'm not sure I know much about which ones produce more oxygen than others, but the NASA link was very helpful for toxin. Apparently the NASA stuff was already on Wikipedia and I wasn't looking hard enough: List_of_air-filtering_soil_and_plants. While you do have a point 76.21, by putting enough plants in my house to cancel out my oxygen use in addition to it already be ventilated I am actually creating an oxygen rich environment, so perhaps I shouldn't be expressing my desire as specifically archiving an "oxygen-neutral" environment. Plus, I think ANY submarine would probably want to be oxygen neutral, not just nuclear ones. The scientific names were actually in the video now that I look back at it:

Chrysalidocarpus lutescens, Epipremnum aureum, and Sansevieria trifasciata

Which are actually all from the NASA list, credited for removing xylene and toluene for the first one in the above list and formaldehyde for the second two.

Out of curiosity, any idea how many plants I would need to provide for 1 person's oxygen needs? Anythingapplied (talk) 05:16, 17 July 2009 (UTC)[reply]

It may be worth reading the comments [3] here. In terms of plants that release oxygen during the night, it appears he's referring to plants that have Crassulacean Acid Metabolism. While my first thoughts on this were similar to Tango's it may also be worth remembering he's from India where air pollution is a lot worse then many Western countries and on his page it says "including an air-cleaning system that involves massive banks of plants instead of massive banks of HVAC equipment" and "making a green office accessible to more businesses in New Delhi and serving as an example of low-cost, low-energy office life" in other words, he acknowledges that you can do the same thing with HVAC equipment, he primarily wants to reduce energy expenditure. [4] may also be worth a read. Nil Einne (talk) 18:34, 19 July 2009 (UTC)[reply]

On further consideration, I was wrong to suggest CAM plants release oxygen during the night. They may primarily uptake their CO2 during the night but don't release oxygen. The oxygen release still takes place during the day shown by this study for example [5] Nil Einne (talk) 09:20, 17 October 2009 (UTC)[reply]


"I think ANY submarine would probably want to be oxygen neutral, not just nuclear ones." -- True, but in the case of a diesel sub, that would be impossible (the diesel engine uses A LOT of air in a short time), and not really critical in any case cause they'd still have to surface every 24-48 hours to recharge the batteries. FWiW 76.21.37.87 (talk) 00:20, 20 July 2009 (UTC)[reply]

Parasympatholytica

Where can I find a list of parasympatholytic drugs? (Need not be drugs prescribed by medics, just chemicals that act as parasympatholytica no matter of strength or side effects.)

Second question: if the parasympathic nervous system was malfunctioning (to any degree) what would be the effects on the body?

No, I'm not trying to poison someone, there would be easier ways and besides, wikipedians are not to give legal advice anyway. 95.112.156.108 (talk) 20:41, 16 July 2009 (UTC)[reply]

Did you see our article on Parasympatholytic drugs? There are some links and references on there that may be able to help you out. Livewireo (talk) 20:55, 16 July 2009 (UTC)[reply]
The little there is to see, yes. That's why I'm posting here. 95.112.156.108 (talk) 21:06, 16 July 2009 (UTC)[reply]
Acetylcholine#Antimuscarinic Agents lists a few, but I'm sure there are a lot more. Looie496 (talk) 03:59, 17 July 2009 (UTC)[reply]
These sound like homework questions. Anyway, a quick look at anticholinergics leads to antimuscarinics and antinicotinics - which both list many drugs. For the second question, well the effects of anticholinergics basically is what would happen if the PNS wasn't working.. --Mark PEA (talk) 10:29, 17 July 2009 (UTC)[reply]

Atmosphere and Gravity

How much gravity on another planet would be necessary to have an atmosphere where people could thrive (assume other factors such as average temp, composition, are similar to earth although I'm interested in what other factors would have an influence). Also, how much gravity could people comfortably live with? TheFutureAwaits (talk) 20:57, 16 July 2009 (UTC)[reply]

We really don't know what gravity humans can live in. Pretty much no research has been done on gravity greater than Earth's, and pretty much the only research on lower gravity is zero gravity and humans can't survive in that for long without taking some precautions (lots of exercise, mainly, I think). I'm not sure about gravity and atmospheres. I know temperature is a big factor - it is easier to "hold on to" cold air than hot air. That's why Titan has a thick atmosphere (about 50% greater pressure than Earth) despite being much smaller than Earth. The composition also matters - humans can survive (at least temporarily) with about a fifth of Earth's atmospheric pressure if it is pure oxygen or with pressure of about 10 times Earth's if you replace most of the air with something inert like Helium (see Heliox). I think composition also affects pressure and density - Venus has about the same gravity as Earth and is really hot, so you would expect it to have a thinner atmosphere, but it is actually about 100 times thicker. I'm guessing that's because it is mainly CO2 which is a little under twice as heavy as air (per mole). --Tango (talk) 21:28, 16 July 2009 (UTC)[reply]
(after ec without rereading the new contribution) Sorry that I don't have sources to cite but from what I have heard, the size of our earth is on the verge of being too light and loosing hydrogen (THE key element for live as we know it) and being too heavy and attracting excessive hydrogen and finally becoming a gas giant. I'm hoping for answers that are less based on private memory and hearsay. 95.112.156.108 (talk) 21:30, 16 July 2009 (UTC)[reply]
We have lost all our (free) hydrogen. The only hydrogen that remains is that in compounds with heavier elements (particularly oxygen in water). Any hydrogen that gets released into the atmosphere escapes sooner or later. Free hydrogen isn't important for life. --Tango (talk) 21:36, 16 July 2009 (UTC)[reply]
As far as I understand, on a geological time scale any hydrogen, temporarily bound or not, will escape a lighter planet with a temperature comparable to earth. Look at venus, look at mars. 95.112.156.108 (talk) 21:50, 16 July 2009 (UTC)[reply]
The hydrogen on Earth is pretty permanently bound. It undergoes various chemical reactions but that just changes what it is bound to. There aren't many naturally occurring reactions that produce free hydrogen. I've certainly never heard of any risk of running out of water before the sun heats up and boils it all off. --Tango (talk) 23:03, 16 July 2009 (UTC)[reply]
Besides, Venus and Mars don't have a "temperature comparable to Earth"... 76.21.37.87 (talk) 04:47, 17 July 2009 (UTC)[reply]
The temperature on Mars CAN get up to Earthlike values - they've recorded 70 degF on a few occasions. However, mostly it's way colder - and Venus is crazily hotter. SteveBaker (talk) 14:12, 17 July 2009 (UTC)[reply]
The question is how much gravity is necessary for a planet to have an atmosphere where humans could thrive. The answer is just enough gravity to hold water vapor for a large amount of time. A Mars sized planet is likely close to the right answer. Dauto (talk) 05:51, 17 July 2009 (UTC)[reply]
Then how come there ain't no water on Mars anymore? 76.21.37.87 (talk) 06:29, 17 July 2009 (UTC)[reply]
No one knows at the moment (No one even knows how much water is gone and how much is just "hidden", e.g. subsurface). There are some hypothesis were the water went and why and when, but the history of mars is not really known to any degree of certainty. And btw, also for venus we do not understand at the moment what "planetary development" has led to its present state. So science cannot answer these questions at present, because we do not have any additional samples beside earth for a habitable planet. --TheMaster17 (talk) 09:57, 17 July 2009 (UTC)[reply]
We know from studies of craters and erosion that water has been on the Martian surface 'recently in geological terms'. The gravitational force of Mars hasn't changed measurably for a billion years - so how come the water would have stayed around for all that time if a lack of gravity was the cause of it 'going away'. That means that it wasn't a lack of gravity that caused the water to "go away" - or all of the evidence of flowing water would long ago have been eradicated by meteor impact and wind erosion. SteveBaker (talk) 14:12, 17 July 2009 (UTC)[reply]
There is water on Mars. May be your question should have been 'how come there ain't no ocean on Mars anymore' The answer is related to surface temperature and pressure. The question was about surface gravity. Some of Mars water has likely been lost due to low gravity over a period of hundreds of millions of years. That's more than enough time for humans to thrive which was the citerion stablished by the question. Dauto (talk) 16:20, 17 July 2009 (UTC)[reply]
Wouldn't the low pressure be bad for people too? 76.21.37.87 (talk) 01:44, 18 July 2009 (UTC)[reply]
Yes, if the atmospheric pressure is too low it can be bad for people. But that's a separate question because surface pressure is not determined by surface gravity alone. You also must take into consideration the atmospheric total mass. Dauto (talk) 04:03, 18 July 2009 (UTC)[reply]
So would it be possible to have a decent-sized atmosphere on a Mars-sized planet? 76.21.37.87 (talk) 04:41, 18 July 2009 (UTC)[reply]
Yes. Dauto (talk) 02:05, 19 July 2009 (UTC)[reply]
OK then, I rest my case. 76.21.37.87 (talk) 04:10, 19 July 2009 (UTC)[reply]
What counts for holding atmosphere is not surface gravity but escape velocity (which, for a round planet, is proportional to the square root of the product of surface gravity and radius) in relation to the speed of a molecule at the relevant temperature. —Tamfang (talk) 22:43, 21 July 2009 (UTC)[reply]
Which in the case of Mars is less than half that of the Earth (5 km/s vs. 11.2 km/s); would that still be enough to hold a decent-sized atmosphere? I'm not a planetary physicist but only a petroleum chemist, could somebody crunch the numbers, please? 76.21.37.87 (talk) 01:11, 22 July 2009 (UTC)[reply]

Pluto's diameter and why gas giant's moon number keep growing

Is Pluto 1400 miles or 1500 miles (0.18 or 0.19 x Earth). [6] say Pluto is x0.18 Earth, NASA say is 0.19 x Earth. Which one is accurate. I thought Microsoft Windows, any kids website is not a valid source. Is Saturn up to 61 moons right now? Anyways, how we know gas giant's moon keep growing. First when last Voyager 2 went to Uranus and Neptune, they had 15 and 8 moons, then in 2000 Uranus had 22 moons then finally Uranus have 27 moons, and finally Neptune have 13 moons. When last Voyagers visit Jupiter, it had 16 moons, then few in 2000 Jupiter's moon jump to 20s, then 30s in 2003 Jupiter's moon skyrocket to 61 moons then finally it's 63. Saturn's moon start with 18 moons, then 30s, then 56 in 2006, then finally 61. Did they use HST or what tools to find new and new moons. Is this possible numbers of Uranus and neptune will continue skyrocketing? Could somday Uranus have up to 40 moons and Neptune have over 19 moons? adn jupter and Saturn over 100 moons?--69.228.145.50 (talk) 21:00, 16 July 2009 (UTC)[reply]

I don't think we know the diameter of Pluto very accurately. It depends on its albedo - how bright its surface is. If it is quite dark then it must be quite big in order to reflect the amount of sunlight it does, if it is brighter then it must be quite small. As for the moons, we just keep discovering more (they've all been there the whole time, we just hadn't noticed them). A lot of them are very small and there is no reason to believe we've found them all or will do any time soon. You also have to remember that there isn't really any difference between the tiny dust particles in the rings and the smallest moons (other than size, obviously), so you could argue that we already know of millions of moons around each gas planet - it depends where you draw the line. --Tango (talk) 21:34, 16 July 2009 (UTC)[reply]
O, this explains why scientific plotting always fails. Even surface temperatures can't be pin right. By the way nothing is space is exactly pin right, always universally error bars. People can literally say Venus is >1000 F, this is why we have book say Venus is 600 C. I would say Jupiter and Saturn have 100s of moons if 100% is found, for Uranus and Neptune, this is impossible to be over 50 moons if 100% of them if found.--69.228.145.50 (talk) 22:18, 16 July 2009 (UTC)[reply]
There are always error bars, you are right, but for some things they are pretty small. We've sent probes for Venus, so we have a pretty good idea of its temperature. The error bars on our measurements will be much smaller than the variation in the temperature. --Tango (talk) 23:05, 16 July 2009 (UTC)[reply]

Glycerin free personal lubricants

I notice many new "personal" (sexual) lubricants proclaim they are glycerin and paraben free. What about those ingredients are bad during sex? --70.167.58.6 (talk) 21:51, 16 July 2009 (UTC)[reply]

Substances that dissolve your condom counteract the reasons why you were using it. It has nothing to do with sex itself. 95.112.156.108 (talk) 22:05, 16 July 2009 (UTC)[reply]
True, but not applicable to the substances the OP listed. Only water-based lubricants (i.e. not Vaseline) should be used with condoms, for the reason you state. Glycerine is a common allergen, and parabens are both allergens and potential carcinogens; these are the reasons for wanting lubricants without those ingredients. jeffjon (talk) 12:50, 17 July 2009 (UTC)[reply]

I wonder the valid source of this site. Most showing I see is African plate moving north, the slide show African plate moving east only, eventually it suppose to collide with European plate closing Arabian and Black Sea. Isn't North and South Amercan plates moving west only no north. I don't expect over 15 lattitude north shifting in about another 100 million years. I expect at least the Northern Africa would be up to Artic Circle in 100 million years. Australia is just quickly moving north, and will collide and form a huge nountain in East Asia.--69.228.145.50 (talk) 22:23, 16 July 2009 (UTC)[reply]

healthy diet and muscle spasms

What vitamins or minerals are supposed to be linked to muscle spasms when in deficiency? 76.123.145.220 (talk) 23:39, 16 July 2009 (UTC)[reply]

Potassium. --Tango (talk) 23:44, 16 July 2009 (UTC)[reply]
And Calcium. Please see Hypokalemia and Hypocalcaemia, respectively. --Dr Dima (talk) 01:10, 17 July 2009 (UTC)[reply]
Iron? -- 71.236.26.74 (talk) 06:20, 17 July 2009 (UTC)[reply]
And magnesium. It should be noted that fasciculation can be caused by many things other than vitamin/mineral deficiency. --Mark PEA (talk) 10:20, 17 July 2009 (UTC)[reply]

Man on Moon

Have anybody else besides those 3 people in July 1969 Buzz what's his name, Neil Armstrong and anyther guy all 79 right now have been on Moon. Have anybody been on dayside of Moon, or only night side. Do we have 1000s of people been on Moon or it's only those 3 peoples in 1969. Space travel is not a vacation spot. It is not just few days to get to Moon, I expect two weeks from Earth to Moon. Plus to survive in space, people have to bring gravity to hold things together, and lots of foods and fuels and liquids, and place to sleep and shower on.--69.228.145.50 (talk) 23:49, 16 July 2009 (UTC)[reply]

Twelve people have walked on the moon. Only two did so in July 1969 (the other guy stayed in orbit). We do not yet have a way of taking portable gravity into space. Algebraist 23:52, 16 July 2009 (UTC)[reply]
"The other guy" is Michael Collins. The poor guy, nobody ever remembers his name... --Tango (talk) 00:12, 17 July 2009 (UTC)[reply]
Nobody except for you and me, that is. 76.21.37.87 (talk) 04:49, 17 July 2009 (UTC)[reply]
And Cuddlyable3, apparently... 76.21.37.87 (talk) 04:38, 19 July 2009 (UTC)[reply]
As a matter of interest, have Aldrin and Collins ever expressed any resentment towards Neil Armstrong? I know that there's been plenty of comedy sketches about 'nth men on the moon' being bitter about their lack of historical recognition over the years... --Kurt Shaped Box (talk) 02:16, 19 July 2009 (UTC)[reply]
If it’s seeming like that tiny Lunar Module is too small to hold everything needed for a trip to the moon, you are right. Although the most exciting photos and videos of the moon landing only show the lunar module, the lunar module was only one piece of the Apollo spacecraft, which did hold enough stuff. Not much fuel is needed to get back from the moon, since it’s relatively easy to escape from the moon’s gravity, due to there being a lot less gravity on the moon than on the earth. Escaping the earth’s gravity, however, does require a lot more fuel, which is why the Apollo spacecraft started out on the tip of a huge launch vehicle.
A bed to sleep in and the ability to take showers are luxuries of modern living, that aren’t really requirements when you’re on a monumental adventure.
Gravity isn’t required to hold things together. The main thing that holds thing together are chemical bonds.
I could go into the details of how the energy content of the fuel used on the Apollo missions was enough to get the Apollo spacecraft traveling fast enough so that it could get to the moon in only a few days. However, no offense if I’m wrong, but I’m guessing that you’re fairly young, and haven’t yet taken the elementary physics classes that would be required for you to understand the explanation. However, the idea of it being impossible to reach a speed fast enough to get to the moon in three days isn’t even something that we have listed as being one of the objections of the moon landing hoax conspiracy theorists. Red Act (talk) 01:24, 17 July 2009 (UTC)[reply]

As to the question about which side of the Moon, all of the Apollo missions landed on the near side of the Moon (the side facing the Earth), so they could communicate with the Earth by radio; and they all landed when it was daytime at the landing site, and left when it was still daytime, so they didn't have to deal with the day/night temperature change and so they had full sunlight to work by. (Remember, daytime on the moon lasts for almost two weeks. Apollo 17 stayed longest on the lunar surface, about 75 hours.) --Anonymous, 04:46 UTC, July 17, 2009.

LOL. Two gravity to go, hold the ketchup- KoolerStill (talk) 13:20, 17 July 2009 (UTC)[reply]
ec. the sides of moon they been on is nighttime with most photos we have with no suns, yes the image always fails when trying to make things look true when try to pin a green dot to show you what it will look like when you actually on moon. Different gravity and lighting conditions on moon, beleive it or not; the images we take is 99.99% hopelessly wrong, then our orange, blue, green light go haywire. Dummies think it is so easy to say Moon is gray, that's just balonies! We had this discussion many times in past, that's over!--69.228.145.50 (talk) 21:21, 17 July 2009 (UTC)[reply]
They landed during lunar morning for idea temperature, and on the side facing earth for best radio contact. We will soon be celebrating the 40th anniversary on earth, but I would like to point out that the moon's phase will not be correct (because Earth's revolution around the sun is not closely linked to Moon's revolution and orientation around Earth). Flimsy human calendars fail to capture useful astronomical information! LCROSS arrived around lunar morning a few weeks ago; it's probably a better "anniversary" than July 16 or July 20, from the point of view of the Moon. Nimur (talk) 14:30, 17 July 2009 (UTC)[reply]

At the risk of stating the obvious : The Moon has its own gravity. It does not have as much gravity as Earth, but it still has a useful amount. The astronauts don't need to bring any with them. This is a good thing because there is no known way to make gravity "portable". (When NASA needs to "hold stuff down", they usually glue Velcro to everything Velcro.) APL (talk) 14:52, 17 July 2009 (UTC)[reply]

Gravity is portable, you just need a really huge engine. If you built a spaceship of the same dimensions and weight as earth, and you could move it around, then you would in effect have portable gravity. You would have to ask if the endeavor would be worth the resources though. Googlemeister (talk) 18:30, 17 July 2009 (UTC)[reply]

The OP should remember the name Buzz Aldrin. He is not to be confused with Buzz what's his name the inventor of the rubber suction cups used to hold things together in space. Cuddlyable3 (talk) 01:45, 19 July 2009 (UTC)[reply]

Good point, Cuddlyable. Unfortunately, few people bother to learn the names of true American heroes like Lewis and Clark, Davy Crockett, Charles Lindbergh, Amelia Earhart, John Glenn, and Buzz Aldrin anymore; instead, they learn all the names of the latest trashy, slutty celebrities like Britney Spears, Christina Aguilera, George Michael, Snoop Dogg, Alec Baldwin, George Clooney -- you name it. In fact, this generation's (MY generation's!) knowledge (or lack thereof) of American history is perfectly shocking. Maybe that's the reason why God told me to write historical ballads. I wouldn't be surprised if most people my age thought that the Alamo was only a car-rental company... :-) 76.21.37.87 (talk) 02:12, 19 July 2009 (UTC)[reply]

Just for fun, and as an example of what I mean, here's an actual student's answer to an actual teacher's in-class question (I got this from the website www.stupidest.com): Teacher: "Who was the first man to walk on the Moon"? Student: "Louis Armstrong".  :-) 76.21.37.87 (talk) 02:17, 19 July 2009 (UTC)[reply]

Something is terribly wrong when a great american hero such as the first moon walker is unsung. Cuddlyable3 (talk) 21:35, 20 July 2009 (UTC)[reply]
:-) 76.21.37.87 (talk) 03:40, 21 July 2009 (UTC)[reply]
As an aside, the Moonwalk article says that the first moonwalker was Bill Bailey, not Michael Jackson... 76.21.37.87 (talk) 03:44, 21 July 2009 (UTC)[reply]


July 17

Some people smell bad. Why?

I've found over the years that some people smell very bad to me. I'm not talking about people who're dirty or flatulent or unwashed or have bad breath or have body odor typically associated with heavy exertion and/or sweating; rather, it seems more that some people's body chemistry creates an odor I find extremely unpleasant. It's difficult to characterize or describe; it smells of nothing but "smelly person". I have also noticed that others often seem not to detect any such an offputting smell from the same people I find smelly, at the same time I find them smelly. What's at work here? Is it akin to the ability (or lack thereof) to taste PTC? 00:04, 17 July 2009 (UTC)

Body odor? —Preceding unsigned comment added by 79.75.79.236 (talk) 00:42, 17 July 2009 (UTC)[reply]

It's mostly to do with what you've been eating, what they've been eating and what you're used to. The compounds in many aromatic substances survive just fine in the body and then get exuded with the various bodily fluids, including the sweat we make almost constantly. Curry, for example, has a very distinctive odour and people that eat a lot of it will tend to exude that smell from their breath and bodies. If you're not used to that because you and your family don't eat lots of curry, then you'll be very aware of the change in odour. It works the other way as well: you may be smelling very strongly of milk, or pepper, or any one of a number of things, but you'd be unaware of it because it's part of your background smell. Matt Deres (talk) 13:16, 17 July 2009 (UTC)[reply]

Let's not overlook the smeller's own nose. If you've got too much X in your system, certain odors will repel you more than the usual person. Vranak (talk) 14:35, 17 July 2009 (UTC)[reply]

Putting fiber in my fiber

Is there any downside to adding Benefiber to Metamucil? Are two types of fiber compatible (one is a guar gum and the other is psyllium husks) --70.167.58.6 (talk) 00:08, 17 July 2009 (UTC)[reply]

I'm afraid we can't give medical advice, but I will point out that, baring very unusual circumstances, you can get plenty of fibre in your diet by just eating plenty of vegetables. --Tango (talk) 00:15, 17 July 2009 (UTC)[reply]
I'm not asking about medical advice. I'm asking if mixing compound A and compound B have a chemical reaction or produce any harmful substances. It's a science question, not a health question. --70.167.58.6 (talk) 12:37, 17 July 2009 (UTC)[reply]
You are asking for advice on dietary supplements. That seems like a health question to me. --Tango (talk) 20:14, 17 July 2009 (UTC)[reply]
Not offering any advice here, but consumption of very high levels of fiber (along with inadequate water, I think) has been known to cause intestinal blockages. ike9898 (talk) 17:35, 17 July 2009 (UTC)[reply]

"I'm asking if mixing compound A and compound B have a chemical reaction or produce any harmful substances." -- It shouldn't, cause neither compound is reactive under normal conditions (they're both mostly cellulose anyway, i.e. sawdust). FWiW 76.21.37.87 (talk) 01:51, 18 July 2009 (UTC)[reply]

Perception of square-wave sounds

Considering this very cool toy (http://www.ladyada.net/make/drawdio/, that produces square-wave audio) caused even more consideration:

  1. is there a simple passive filter that would "round off" the lower-frequency square waves (i.e. low-passing the fundamental of the square waves so they would approximate sine waves) without attenuating the higher frequencies?
  2. the nominal human hearing cut-off of 20kHz would suggest that any periodic wave above 10kHz would sound the same since higher harmonics can't be heard (very well). Is this actually the case? If so, are distortion specs on tweeters above 10kHz worth much?

Thanks. Saintrain (talk) 02:04, 17 July 2009 (UTC)[reply]

To the first question, yes — a suitable low-pass filter will attenuate the higher frequencies, leaving the fundamental (and lower harmonics, depending on the filter) essentially intact. Here's a YouTube video demonstrating the principle with square and sawtooth waveforms: [7]. The simple RC circuit shown in our article would serve. TenOfAllTrades(talk) 02:47, 17 July 2009 (UTC)[reply]
I’m not sure if I’m understanding your first question correctly, but if I’m correctly rephrasing it, it sounds like maybe you’re wanting the amplitude of the fundamental frequency on the filter’s output to be higher than the amplitude of the fundamental frequency on the input, due to the presence of the higher frequencies on the input? Like by having the filter somehow phase-shift the higher frequencies such that they’ll contribute to the fundamental frequency on output, rather than just being attenuated? That isn’t possible with a linear passive filter, e.g., any filter consisting of just resistors, capacitors and inductors, but it would be possible to do something like that with a nonlinear passive filter.
It isn’t true that any periodic wave above 10kHz will sound the same. For example, an 11kHz sine wave sounds quite different from a 19kHz sine wave. So if a tweeter attenuates those two frequencies differently, that’s going to make an audible difference. Red Act (talk) 03:35, 17 July 2009 (UTC)[reply]
Sorry. English is a dangerous language in unskilled hands.
  1. The toy can produce a wide tonal range but the (555) oscillator produces square waves. I was wondering if the lower frequencies can be (simply) "sine-ufied" without attenuating the higher ones. I'm guessing not.
  2. What I meant was, since a 15kHz square wave is made up of 15kHz, 30kHz, 45kHz, 60kHz, ... sinusoids and nothing above (nominally) 20kHz can be perceived, that any 15kHz waveform would sound the same as a 15kHz sinusoid. Sounds (haha) too easy.
Thanks. Saintrain (talk) 05:32, 17 July 2009 (UTC)[reply]
  1. “Not with a linear passive filter” would be an answer to your first question.
  2. That’s true, a 15kHz sine wave would sound the same as any 15kHz waveform, as long as the 15kHz component of the waveform had the same amplitude and phase as the sine wave. However, your original question number 2 asked if distortion specs were worth much above 10kHz. If a frequency response distortion causes the tweeter to disproportionately attenuate the 15kHz component of a sound, that’s going to be audible. Was the 10kHz a typo, and you meant to say 20kHz? If you meant to say 20kHz, then the answer would be yes.
I’m afraid some audiophile is going to come along and point out that some people can actually detect distortion above 20kHz, and I’m sure that’s true, particularly with younger people. But this discussion is a theoretical one based on the assumption that people can’t hear anything above 20kHz. Red Act (talk) 06:56, 17 July 2009 (UTC)[reply]


To provide a mostly-useless answer to the first question: yes. A Fourier series decomposition of a 15kHz square wave shows that the 15kHz component is already a sine wave. The higher frequencies that you desire not to attenuate are what turn your 15kHz sine wave into a square wave (see the animation in square wave). So, if you really seek to turn a 15kHz square wave into a 15kHz sine wave with all the higher frequencies unattenuated, your passive filter of choice would be … a wire. – 74  08:33, 17 July 2009 (UTC)[reply]
The OP is incorrect in supposing that a 15kHz square wave is made up of 15, 30, 45, 60... kHz sinusoids. A square wave has only odd-multiple harmonics i.e. a 15kHz square wave is made up of 15, 45, 75... kHz sinusoids. (Even multiple harmonics exist only if the wave is rectangular.)
I think the OP questions whether distortion of tones above 10KHz matter, i.e. is easily audible, and the answer is no. (A tweeter could produce harmonics starting at 30kHz due to symmetrical distortion or starting at 20kHz due to asymmetrical distortion. Neither is easy to hear.) A different question is whether the tweeter has level frequency response over its audible range because that affects the reproduction of possible harmonic content of all tones below 10kHz, which includes most music.Cuddlyable3 (talk) 01:29, 19 July 2009 (UTC)[reply]

lcd and tft

what is the diference between lcd and tft —Preceding unsigned comment added by 122.163.144.201 (talk) 03:22, 17 July 2009 (UTC)[reply]

You probably want Thin film transistor liquid crystal display. Algebraist 03:27, 17 July 2009 (UTC)[reply]

voltage and charging batteries.

Here in China the voltage is 220V. I've recently bought a Game Boy Micro which comes with a power cord (not an adaptor) to charge the battery saying it should only be used with 110V.

I'm currently using a stepdown converter, but am wondering if this is really necessary.

Why? Because I see kids all over the place playing JPN Game Boy Advanceds and Game Boy Micros and charging them willy-nilly.

I know that improper voltage can hose appliances... but what does it do (if anything) to batteries? 61.189.63.221 (talk) 03:52, 17 July 2009 (UTC)[reply]

Too high a charge voltage could make the batteries blow up, and also fry the diode rectifier that changes the AC from your socket to the DC needed to charge batteries. Stick to using your stepdown converter at all times. 76.21.37.87 (talk) 04:57, 17 July 2009 (UTC)[reply]

Theory vs. law

I have some homework which states:

Discuss how theories such as those above [Newton's/Galileo's theories about gravity] develop into laws with time & experimental testing.

Now, this is implying that laws are somehow better or more solid than theories. My understanding is that a law is simply a description of what happens, while a theory is an explanation for observations. How would I answer the homework question? (This is going to be marked, and I have NO way of talking to my teacher about the question at any point before the work is due.) --wj32 t/c 04:16, 17 July 2009 (UTC)[reply]

It looks to me like your understanding of the difference between a physical law and a theory is more accurate than your teacher’s! Unfortunately, the “theories develop into laws” idea that they teach at some schools does not mesh well with how those terms are actually used by scientists. My impression is that the “theories develop into laws” idea is just something they teach in like junior high or high school, and isn’t something you’d be likely to encounter in college. I don’t know the best way to explain to your teacher that this wasn’t a very good homework question. Red Act (talk) 04:52, 17 July 2009 (UTC)[reply]
Welcome to the Wikipedia Reference Desk. Your question appears to be a homework question. I apologize if this is a misevaluation, but it is our policy here to not do people's homework for them, but to merely aid them in doing it themselves. Letting someone else do your homework does not help you learn how to solve such problems. Please attempt to solve the problem yourself first. If you need help with a specific part of your homework, feel free to tell us where you are stuck and ask for help. If you need help grasping the concept of a problem, by all means let us know. Thank you. —Scheinwerfermann T·C05:00, 17 July 2009 (UTC)[reply]
Thank you for using HumanBrain(r). HumanBrain(r) is one of the most advanced WetWare(tm) systems available. Please note that the new HumanBrain(r) LanguagePack II(tm) (European Languages/English) is now available. LanguagePack II(tm) adds the ability for semantic analysis beyond simple keyword scanning. LanguagePack II(tm) is a free download for your HumanBrain(r). Installation can take between 5 minutes and 10 years depending on existing programming and specific HumanBrain(r) model employed. --Stephan Schulz (talk) 09:27, 17 July 2009 (UTC)[reply]
Judging from the idea that the development occurs after time and testing, your teacher may be confusing the difference between theories and laws with the difference between hypotheses and theories. Maelin (Talk | Contribs) 05:02, 17 July 2009 (UTC)[reply]
As far as marks are concerned, just give what the teacher expects. Assume a Law to mean something like indesputably proved beyond doubt fact of life, and explain how a theory matures into a "Law" over time through tests by experiments and observation. However, afetr you are marked, do have a discussion with the teacher about this. Rkr1991 (talk) 05:31, 17 July 2009 (UTC)[reply]
Seems like a good idea. Thanks for all the suggestions, everyone. :) --wj32 t/c 06:03, 17 July 2009 (UTC)[reply]
It’s kind of intellectually repulsive to intentionally give an incorrect answer, but I think that probably really is the best advice. There’s no point in getting marked down just to prove a point, and actually knowing the right answer is what’s really important, anyway, not whether the teacher acknowledges you for knowing the right answer. It’s also probably best to point the teacher’s mistake out to him quietly, after class, rather than pointing it out to him during class. I pointed out a couple of my high school math teacher’s errors in class right after he made them, and it really made him kind of not like me a little. In my experience, college profs are much less flustered by being corrected in class than high school teachers; in fact, they’ll often thank you for correcting their mistake. Red Act (talk) 11:16, 17 July 2009 (UTC)[reply]
Your should tell your teacher (in a tactful way) that their hypothesis that "theories develop into laws with time & experimental testing" is not supported by the way that these terms are used in the scientific community. As evidence, show your teacher these pages - [8], [9], [10]. Clear consensus is that there is no hierarchy of proof or reliability between theories and laws - the only difference in usage (and it is a very marginal one) is that the term "law" is usually applied to a short and concise statement or principle whereas a "theory" is a larger and more complex body of interrelated concepts with a wider, more general scope.
Quite often the historical progression of a scientific field starts with the discovery of empirical laws and progresses to a general theory that incorporates and explains the separate laws. For example, the kinetic theory of gases incorporates the ideal gas law, which in turn combines Boyle's law and Charles's law; Hubble's law is incorporated into and explained by the Big Bang model; the law of multiple proportions is explained by atomic theory; and Mendel's laws are explained by genetics.
Maybe I am being cycnical here, but it seems to me that the erroneous idea that "theories develop into laws with time & experimental testing" could be used to set the stage for the bogus "evolution is just a theory" argument. Gandalf61 (talk) 11:28, 17 July 2009 (UTC)[reply]
No, here in Australia we aren't anti-evolution :) --wj32 t/c 23:59, 17 July 2009 (UTC)[reply]
Well, I think you can do this right and still get good marks - and since you CLEARLY have a better grasp of this than your teacher, I'm strongly of the opinion that you should stick to the truth no matter what. The trick here is to write the truth - and yet still answer what the question is really seeking for you to explain - which is how a "hypothesis" turns into either a theory or a law with time and experimental testing. This is more work for you - but that's what happens when you stand up for a principle.
I think you need to write two sections:
  • Firstly, you need to deflect the question into one that you CAN answer. You must explain how the standard dictionary definition of the words "theory" and "law" differ sharply from common scientific usage - look the words up in a dictionary - and quote what it says. I found this, for example. Explain that you are assuming that the word "theory" in the question is being used in the non-scientific sense of the word (As Merriam-Webster puts it "6 a: a hypothesis assumed for the sake of argument or investigation b: an unproved assumption". However, you can't make it sound like this is just your opinion because essays like this aren't about opinion. You also can't make it sound like this is something you heard from a bunch of random guys on the Wikipedia reference desk (which it's not because you already knew it). You are going to have to find some solid references that you can point to that say this - preferably more than one. You need to be able to say that "so-and-so (a highly respected scientist/organisation) said such-and-such (which supports your claim) in such-and-such document (hopefully something your teacher can easily find and read - but at least something respectable...but not Wikipedia - which many teachers hate)". For example, you could quote the American Association for the Advancement of Science here - in the section "Is evolution "just a theory"?" - they say "In detective novels, a "theory" is little more than an educated guess, often based on a few circumstantial facts. In science, the word "theory" means much more. A scientific theory is a well-substantiated explanation of some aspect of the natural world, based on a body of facts that have been repeatedly confirmed through observation and experiment. Such fact-supported theories are not "guesses" but reliable accounts of the real world. The theory of biological evolution is more than "just a theory." It is as factual an explanation of the universe as the atomic theory of matter or the germ theory of disease. Our understanding of gravity is still a work in progress. But the phenomenon of gravity, like evolution, is an accepted fact.". This clearly says that a theory is a RESULT of time and experimental testing - not the STARTING POINT for such work. That done and clarified, you can proceed to do the assignment you were given - because you can't use this little misunderstanding to duck out of doing the work!
  • Now, having established (eg from Merriam-Webster) that the word "hypothesis" would be a better choice, you must proceed to explain how a scientific hypothesis can develop into either a scientific theory OR a law with time and experimental testing...because that's what the homework assignment is all about - and if you don't do it, you won't get a good grade. You've actually increased your workload in the process - because now you need to describe how a THEORY gets made as well as how a LAW comes into being. But it's worth the effort.
I'm 99% certain that your efforts will be rewarded with a decent grade IF you do it like that - subtly and with style and integrity - and not whining at the teacher's incompetence!! On the off-chance that you have an utterly bone-headed teacher (I sure hope not!) - you might ask your teacher to "help you understand" by providing an example of a "theory" that turned into a "law" - we know that there aren't any - so you're on solid ground here - you may have to say "but that wasn't a theory - that was just a hypothesis". If all else fails, you may need to get your parents to appeal the grade to a student councillor or the head of the science department - or the principle...whatever it takes. I had to fight for my son's (correct) answers to bogus questions on many occasions...and every time, we won and he got a good grade in the end. But if you do it right - and back up your assertions with solid references - I'm sure you'll be OK without any fuss whatever.
You're in good company by the way - my personal hero (Richard Feynman) was particularly displeased with these kinds of errors in assignments and school books and took on a personal crusade to try to eradicate them that wound up with him having to write three of the best Physics books in existance - "Feynman's Lectures on Physics". The fight over the teaching of evolution in US schools lies entirely in the misunderstanding (intentional or not) of the word "theory". I applaud your knowledge and your desire to stand by your principles. No matter what, you'll always know that "You were right" - and that's worth more than a grade.
SteveBaker (talk) 14:00, 17 July 2009 (UTC)[reply]

Thinking about this a little, my guess is that the OP’s textbook is correct, which is where the OP got his correct information from, and it’s just the teacher that’s made a mistake. If the textbook matched the teacher, the OP wouldn’t have gotten confused and asked us for a clarification. So pointing the teacher’s mistake out to the teacher might be as simple of a matter as pointing out the pertinent page(s) in the textbook. Red Act (talk) 14:52, 17 July 2009 (UTC)[reply]

Well, the homework is actually taken directly from our science syllabus. Pretty depressing, I know... --wj32 t/c 00:01, 18 July 2009 (UTC)[reply]

Falling objects

There is an article [11] which states:

Also intriguing is Galileo's report, based on experiment, that balls of unequal weight do not only fall at different rates, but that the lighter one initially pulls ahead of the heavier one until the heavier catches up. In the early 1980s the science historian Thomas Settle tried to repeat Galileo's falling-body experiments and, astonishingly, noted the same thing. He suggested that fatigue induced in the hand holding the heavier object tends to cause this hand to let go more slowly, even when the dropper believes the objects are released simultaneously.

I'm a bit confused. Is this actually true, or is the article just saying that Galileo observed it but it's wrong? Why would this (the lighter one being faster at first) happen? --wj32 t/c 06:20, 17 July 2009 (UTC)[reply]

If the two objects are mechanically released at precisely the same time in a vacuum chamber, the two objects will fall at exactly the same rate, even initially. The effect of the lighter one falling faster initially only applies to the sloppier situation in which the objects are dropped in air, by hand.
The fatigue hypothesis seems reasonable to me. Another hypothesis that seems reasonable to me is that it isn’t due to fatigue, per se, but that it takes longer for a tightly clenched hand to unclench than a lightly clenched hand, simply due to it involving a greater change in physiological state. Another hypothesis that seems reasonable to me is that when one’s fingers move rapidly out from under something really light, something like the Bernoulli effect will basically cause the feather to get sucked downward slightly. But the latter two hypotheses are purely original research. Red Act (talk) 07:22, 17 July 2009 (UTC)[reply]
Galileo's result is only approximate because he couldn't test it in a vacuum - and air resistance has an effect. But done 'properly' with some kind of mechanical release system in a vacuum, there would be no such effect as our OP describes. SteveBaker (talk) 13:12, 17 July 2009 (UTC)[reply]
Didn't Galileo roll same sized but different weight spheres down an incline, rather than merely dropping things? The effort of holding on an ioncline would not be great. A mechanical release would be an obvious control. Ref checking is indicated. Edison (talk) 14:41, 17 July 2009 (UTC)[reply]

kindly help....for spectrophotometer

1) how to calibrate and standardise visible spectrophotometer using aquous solutions? 2) which aquous solutions should be used for it? and what should be their concentrations? —Preceding unsigned comment added by Ddrcpan (talkcontribs) 06:56, 17 July 2009 (UTC)[reply]

I believe you calibrate by putting a test tube with distilled water, which will reflect, refract and absorb the minimal amount of light of any liquid, into the machine and taring it so that that whatever is reflected, refracted or absorbed is blanked. DRosenbach (Talk | Contribs) 17:19, 17 July 2009 (UTC)[reply]

Documenting 2009 solar eclipse

Someone else may have asked something similar already, sorry if this is a repost!

I'm going to be observing the solar eclipse next week and I hope to take some footage/pictures, and I'm just wondering how careful I should be. I've got a recent Canon Ixy (Powershot) just so that I can have some sort of video record, and I will be photographing using my Nikon D60 and a 200mm lens. I've never photographed an eclipse before so I'm not really expecting anything spectacular (besides it won't be complete where I live) but I'm mainly concerned for the safety of my cameras! Basically every website I look at warns me that the eclipse could harm the lens/sensors of my digicam, and I guess I need a solar lens? of some sort to protect them, but what exactly am I looking at purchasing here, how much will it set me back, and where can I get one? Also, is it possible to make my own? If anybody knows a site with some easy instructions to make a home-made eclipse glasses, please let me know! And lastly, is it really necessary to use a solar lens for a digicam, or is that just a precaution for when you're getting in really close? Thanks! 210.254.117.186 (talk) 08:31, 17 July 2009 (UTC)[reply]

What do you mean by "getting in really close"? The distance to the sun won't change. I can assure you that taking pictures with a normal digicam without any filter will destroy the sensor of the cam. When there was a total solar eclipse here in germany, where I live, two friends of mine destroyed their cams in this way. NEVER look at the eclipse with your naked eyes, use an appropriate filter! In germany there were special glasses on sale weeks before the eclipse, which were certified to protect human eyes. Normal sunglasses won't do! It might seem that the sun is "dark" enough, but in reality it isn't. You will damage unprepared optical equipment, including your eye, if you point it to the eclipse. There are cases in the literature where people went partially blind from staring into an eclipse, so I'm begging you not to take the risk of loosing your sight. --TheMaster17 (talk) 10:10, 17 July 2009 (UTC)[reply]

I don't know about damaging the sensor - I think that quickly pointing the camera, taking the photo and then pointing it away again should be OK...but perhaps it's not worth the risk. But in any case, the problem is that in a non-total eclipse, the amount of light coming from the sun will overwhelm the sensors and you won't get a decent picture. Digital cameras can only handle so much light before they have hit the maximum number that the sensor can output and everything goes completely white. So I think you could benefit from having a filter of some kind to cut the amount of light down to something reasonable. You could test this by trying to take photos of the un-eclipsed sun...which will be only maybe twice as bright as a half-eclipsed sun. Obviously (I hope!) you understand that staring at the eclipse with unprotected eyes is very dangerous - and looking at it through your camera's optical viewfinder is possibly even more dangerous...especially if you have an optical zoom. By all means line up the shot using the LCD viewfinder if your camera has one...that's entirely safe. I've been saying this a lot recently - but I'll say it one more time. Observing the sun (and even photographing it) can be done very effectively by projecting an image of it through a pinhole into a darkened room. If you have a window facing where the sun will be during the eclipse - you can tape some thin (preferably black) cardboard to the windows to make the room completely dark - and poke a small pinhole into the cardboard to let in some sunlight (this is effectively just a 'pinhole camera'). This will project an image of the sun onto any flat, white surface held up in front of the pinhole - which you can move closer to make a smaller but brighter image or further away for larger and dimmer. You can also make the image brighter - but fuzzier - by slightly enlarging the pinhole - dimmer but sharper by using a finer pinhole. It's easy to experiment. You won't hurt your eyes or your camera by looking at this image. It's very cool (especially now that the sunspots are coming back) and everyone should do this at least once - eclipse or no eclipse! Using a conventional camera with flash turned off and a 'macro' or 'closeup' lens setting if your camera has one - lets you take photos of the projected image that will look really great. Best of all, you can get everything set up and take practice photos of the uneclipsed sun any time to be sure you have everything right for the big day. That's a good idea - because you only have a few minutes to get what you need - and you don't want to be fiddling around getting the camera settings right. Just be sure to do the setup at the same time of day as the maximum amount of totality that's predicted so you'll have the best-facing window for the event. SteveBaker (talk) 12:51, 17 July 2009 (UTC)[reply]

I forgot a piece of pinhole camera advice. To get a really SHARP image - cut a square hole a couple of inches across in your cardboard and tape some aluminium kitchen foil across it. Then punch your pinhole in the foil. It makes a much smaller and more accurate hole for the ultimate in sharpness. SteveBaker (talk) 13:09, 17 July 2009 (UTC)[reply]
Total solar eclipse during totality, March 29, 2006, Sallum, Egypt
,
Playing with double shadows during an eclipse
You can use an ordinary camera to take pictures during a solar eclipse. Be sure to SECURELY mount a piece of #14 welder's glass in front of the camera (Not the side with the display the other side i.e. before the light hits the camera.) I tried a mylar filter last time and was unhappy with the results. (It was expensive but buckled and got me reflections on the images.) Friends used the welder's glass and got awesome pictures. Since your eclipse is not total you don't have to worry about removing/putting on the filter arrangement during the event. That usually makes for more difficult constructions. Build yourself a solid frame or box. Sticky tape or Duck tape are not suitable for this task. Make sure absolutely no light can enter the setup except through the welder's glass. There are many things to observe apart from the sun turning into pac man. See if you can spot double shadows as in my pic. (No the guy was not Aquaman or an alien with extra knuckles between his fingers:-). Have fun, but be safe. --Lisa4edit (talk) 15:40, 17 July 2009 (UTC)[reply]
Thanks Lisa, I'll see if I can find me some welder's glass then and make a little box for it. 210.254.117.186 (talk) 16:12, 17 July 2009 (UTC)[reply]
My brother travels the world photgraphing solar eclipses as an enthusiastic amateur. There are some tips from him here: http://myweb.tiscali.co.uk/steve.holmes/eclphoto.htm. --Phil Holmes (talk) 12:05, 18 July 2009 (UTC)[reply]

gold plated connectors

is there a true difference between regular cables (hdmi, USB, internet) and those made by "boutique cable" manufactures that include gold plated connectors?

puzzles me is that the cable is still copper so would any benefits from the gold connectors be negated by the copper wire? im looking for scientific reasons (from a metallurgy, engineering perspective) for a difference or lack thereof. Im not looking for an audiophiles perspective, they tend to buy anything if its expensive. seems as though they are victim to the the placebo effect.

EDIT so it seems that gold plating prevent corrosion, makes sense, but is there any true sound difference between monster cable(and similar cables) and normal priced audio cable. one of the claims ive heard is that with monster cable you hear "more of the nuances of the music"


Thanks —Preceding unsigned comment added by 71.98.64.15 (talk) 10:13, 17 July 2009 (UTC)[reply]

Well, for digital signaling, there is no difference. All decent cables that follow the standards are plenty good enough to carry the digital signal. Boutique cables for digital signals are a scam. For analogue transmission, cable quality can make a difference. But even there, gold plating is not going to have a significant difference. If you can hear the difference between decent and overpriced cables, James Randy has ONE MILLION DOLLARS for you.[12] --Stephan Schulz (talk) 11:39, 17 July 2009 (UTC)[reply]
The gold plating isn't actually that expensive - you can get fairly cheap gold plated cables if you want.
The gold plating ensures that the connector will not corrode or rust, but a rust free alloy would do just as well.
There's and additional complication that a junctions between different metals have properties (eg think thermocouple) - which potential could actually degrade the signal - in practice there is not a difference.83.100.250.79 (talk) 11:49, 17 July 2009 (UTC)[reply]
It's probably a good idea to match the platings - ie if a device has tin plated connections then don't use gold plated connectors.83.100.250.79 (talk) 11:55, 17 July 2009 (UTC)[reply]
The theory to gold plated connectors is that gold resists corrosion better than copper, so gold plated connectors can wind up having a lower contact resistance than copper ones, particularly in a damp environment. I have had a 50-year-old copper phone connector corrode to the point that it made for a staticky phone connection, and perhaps that problem wouldn't have occurred had the connector been gold-plated. Of course, digital devices become obsolete way before they’re that old. Red Act (talk) 12:20, 17 July 2009 (UTC)[reply]
Gold plating is most useful on cables you will need to plug and unplug often. Gold is soft and will somewhat lubricate the connection as it slides in and out. For connectors that stay in place for most of their life - that are maybe unplugged when you move house or something...meh - any old cable of the appropriate standard will do. The most ridiculous thing is when audiophiles claim to be able to hear the difference between gold plated cables and the regular kind. Audiophiles have a problem. They are like most of us geeks who love to debate the merits of Linux versus Windows or MINI Cooper's versus Mustangs - to endlessly tweak the object of their joy to get another 1% of performance out of it...except that their field of interest is 'solved'. We can't buy the perfect car or the perfect computer - but we can all have essentially unlimited quantities of perfect (in the sense of 'better than human perception') audio at any time we want from a sub-$100 box that you can buy at any Walmart. There is nothing left to be geeky about...no conceivable reason to rush out and buy this years sexiest $10,000 stereo system beyond the shape of the knobs and the color of the LED's. Sadly, these people retreat into a world of their own where they claim they can tell the difference between two ethernet cables used to connect their PC to their media center (yes, really!) when it's a completely lossless digital connection with error recovery built in at the software level. These people will happily pay $1000 for a set of cables just so they can "one-up" their audiophile buddies. Check out these [13] nut-jobs who sell cables with silver conductors instead of copper at hundreds of dollars for what would normally be a $1 cable. Their advert reads: "These cables allow deeper insight into the textures of the critical mid-range, leading to improved intelligibility of vocals, greater resolution of acoustic spaces around the instruments, more real-sounding instrumental timbres, etc. It’s not only beneficial on acoustic instruments – it also gives greater insight into studio techniques, the timbres of synthesisers and guitars etc. Nuance and emotion in vocal and instrumental performances come to the fore."...completely ridiculous of course. It's actually rather sad. SteveBaker (talk) 12:30, 17 July 2009 (UTC)[reply]
The bullshit related to cables is legendary, but I'd say that nothing can top the $485 wooden volume control knobs they were pushing a few years back: [14]. Alas, these knobs are no longer being sold by the original manufacturer. TenOfAllTrades(talk) 14:16, 17 July 2009 (UTC)[reply]
I have restored telephones and telegraph equipment over 100 years old. Soldered connections still conduct. Tinned connections screwed together generally still conduct. Bare brass or copper may develop surface oxidation which insulates, even though no gross corrosion is seen. A goldplated connection should still work fine after that interval. Ordinary audio connectors, with copper inside and some silvery looking outer conductor (tin?) or the connection of vacuum tube pins in their sockets can become oxidized and noisy after ten years or more. Gold plating would likely help maintain continuity. Edison (talk) 14:39, 17 July 2009 (UTC)[reply]

Your 2nd question - See http://www.google.co.uk/search?hl=en&q=monster+cable+rip+off&meta=&aq=f&oq= the search terms are bound to bias the results - but you can see if you read the results that there is someting in the claim that they are shit.

In general there are some bad cables - in my experience once problem is that caused by metal connector shields (the plugs) - on analogue video I've found these to sometimes give noticeable (really noticable) noise. Only buy ones with a insulating plastic plug - these are often the cheapest.

Why not read Monster_Cable_Products - selectie quote fo fun:

one experiment, audiophile listeners could not distinguish between short Monster cables and ordinary coat hangers.

Use your common sense - nothing worth the money raises suspicions.83.100.250.79 (talk) 18:44, 17 July 2009 (UTC)[reply]

I have found that it makes a difference, probably because of oxidation resistance leading to less static etc, plus more expensive cables tending to generally be better-made than the cheapest ones. Cables with gold plated connectors cost a few bucks extra if you shop around and I think it is worth it to do so. I wouldn't get the scammy ones where they charge several times more for a lot of marketing crap. They're basically the same cables. 67.117.147.249 (talk) 00:50, 19 July 2009 (UTC)[reply]

REPORTING

Where and to who would you report a valid UFO Sighting? —Preceding unsigned comment added by Chromagnum (talkcontribs) 11:12, 17 July 2009 (UTC)[reply]

There’s no need to report a UFO sighting, as governmental and scientific organizations will just assume that it’s just yet another natural event that people have gotten overly excited about because they can’t immediately identify it. However, some of these UFO organizations will probably be happy to hear about the “sighting”. The Unidentified flying object article lists some phenomena that might help you make a guess as to what the thing actually was. Red Act (talk) 11:27, 17 July 2009 (UTC)[reply]
If you have a video or at least a picture of the thing, someone here might be able to help you identify it as being ball lightning or a meteor or whatever. Or was it all over too quickly to take a video or picture of it? Red Act (talk) 11:35, 17 July 2009 (UTC)[reply]
A UFO (in the original sense of "any flying object that you can't immediately identify") is a singularly irrelevant event. A UFO (in the more modern sense of "a flying saucer") is essentially certain to be a case of you failing to correctly identify a UFO (in the old sense of the initialism)! In either case is it no more than a passing curiosity. The curious thing about the modern use of "UFO" is that people almost always use in the sense of "A flying object that I've already identified as a flying saucer"...hardly 'Unidentified'! Unidentifiable flying objects might be more interesting! Anyway - as Red Act says - the old 1960's idea that these things had to be reported to the air force or something is long gone. Tell your friends - find a bunch of flying saucer nut-jobs and tell them - but nobody else gives a damn. SteveBaker (talk) 12:02, 17 July 2009 (UTC)[reply]
As my old father used to say (ex-RAF) any FO that is not I is a UFO (or any Flying Object that is not Identified is an Unidentified Flying Object) and it remains a UFO until it becomes I. --TammyMoet (talk) 19:23, 17 July 2009 (UTC)[reply]
My father was ex-RAF too - and he used to say the same thing...I suppose it was something they were taught. SteveBaker (talk) 00:08, 18 July 2009 (UTC)[reply]
I've seen two "flying saucers" recently, and both turned out to be USAF aircraft (one was a Blackhawk chopper, the other one a C-17 Globemaster). 76.21.37.87 (talk) 02:02, 18 July 2009 (UTC)[reply]
I saw a UFO once about 10 years ago, while I was out walking the dogs with my girlfriend one night after dark. A reddish-orange object made a rapid, curved arc through the sky. The UFO remained unidentified for about 5 or 10 seconds – until I got close enough to see that there was a house in that direction, and I saw the person sitting in the second-story window of that house, who I realized had just flicked a still-lit cigarette butt out the window. Red Act (talk) 02:30, 18 July 2009 (UTC)[reply]

A thermometer broke in a hot tub

I'm working at a hotel and a thermometer in the hot tub broke, leaking some blue, oily substance into the tub. It didn't look anything like mercury but I don't know enough about mercury to know whether it's possible to make it look blue and.. oily. I've heard they often use semiconductors in thermometers although I won't vouch for that.

What do you suppose the liquid substance is? What should I do? Should I clean the tub with water, soap.. should I close it indefinitely? --88.149.11.100 (talk) 12:17, 17 July 2009 (UTC)[reply]

It is an oil, nothing special. Mercury is silver and doesn't in any way behave like an oil. It rolls and, if spilled into a tub, will quickly roll down the drain. Use a cleaner for oils (most kitchen cleaners work well). -- kainaw 12:42, 17 July 2009 (UTC)[reply]
Are you sure it's an oil? I didn't know they could be used in thermometers. If you're sure then this should be all right but I would like to get a confirmation just to be sure, nothing personal Kainaw :) --88.149.11.100 (talk) 13:00, 17 July 2009 (UTC)[reply]
There are two common types of thermometers used in this sort of application. The cheap ones use a bimetal strip. Those are usually obvious because they have a dial display. The more expensive ones use an oil. They tend to have a digital display. Even if it is a bimetal strip, an internal lubricant oil is used to keep the strip from sticking to itself. Neither design uses mercury. In fact, it is rare to find mercury in thermometers. If you go look at them in the store, most are digital, many have a red oil, and a couple might have mercury (if you are lucky - I haven't actually seen a mercury one in a long time). Also, if it were mercury, it would behave like mercury. Mercury is very heavy. It quickly goes to the lowest point on any surface. By quickly, I mean that it appears like it has intelligence and is racing towards a destination. In a tub, the lowest point is the drain, so it would quickly roll straight down the drain. -- kainaw 13:09, 17 July 2009 (UTC)[reply]
According to Alcohol thermometer, "The liquid used can be pure ethanol or toluene or kerosene or Isoamyl acetate," with an added dye.--Shantavira|feed me 13:11, 17 July 2009 (UTC)[reply]
I've run into mercury thermometers every now and then in labs; but they're generally only used in certain applications where the temperatures get into hundreds of degrees celsius - it's generally not used for more mundane applications because of its now-recognized toxicity. I also agree with Kainaw that if it was mercury, you'd know it - mercury behaves very differently to the other liquids you encounter in everyday life. It was almost certainly some kind of dyed oil or alcohol that may be slightly poisonous but will do nobody any harm if diluted in a tub of water. ~ mazca talk 13:33, 17 July 2009 (UTC)[reply]
Mercury-in-glass thermometers can’t even be sold legally in some countries (as listed in the article), and in some states in the U.S.[15]. Red Act (talk) 14:05, 17 July 2009 (UTC)[reply]
My dad used to show me the mercury from mercury thermometers when I was younger, but it sounds like they're not as common any more. When I think about it it would probably sink if it were mercury but my first thought was to ask- I didn't really want to take any chance with this. Thanks for your helpful answers though :) I cleaned the tub and everything seems to be all right now, I'm more knowledgeable about thermometers now which is never a bad thing ;) User:BiT --88.149.11.100 (talk) 15:39, 17 July 2009 (UTC)[reply]

Dry mouth

Please explain what chewing xylitol-containing gum helps with dry mouth? Does the xylitol do something specific? --Reticuli88 (talk) 14:32, 17 July 2009 (UTC)[reply]

This is probably just marketing hype. Xylitol (q.v.) is merely an artificial sweetener. Chewing practically anything (apart from dry foods that absorb water) will stimulate saliva production.--Shantavira|feed me 15:09, 17 July 2009 (UTC)[reply]
(Not entirely hype - see below.)
The sweetness is designed to make you salivate. Xylitol is used because it is sweet, and less likely to have the problems associated with using normal sugar:
http://www.google.co.uk/search?hl=en&q=xylitol+saliva&meta=&aq=f&oq= http://www.google.co.uk/search?hl=en&q=xylitol+dental+caries&revid=1295609484&ei=Q5hgSt2NFuDTjAe-gbGtDg&sa=X&oi=revisions_inline&resnum=0&ct=broad-revision&cd=1 (obviously some of these results are marketting, and may be funded by xylitol manufacturers - disclaimer)
Xylitol#Dental_care
One possibility for this is that bacteria that can metabolise are rarer (or don't exist) compared to bacteria which can metabolised sucrose or glucose - reason being that xylitol is a 'new' sugar...83.100.250.79 (talk) 15:33, 17 July 2009 (UTC)[reply]
Xylitol is a sugar alcohol. The cooling effect described in the article may provide added relief. Since Xerostomia includes tooth decay as one of the symptoms. chewing gum with sugar is probably not a good idea. I don't know if the existing bugs have trouble with the "alcohol" portion of the sugar alcohols. Hand sanitizers are made from alcohol gel, but the "sugar" part may counterbalance any antibacterial effect. Lisa4edit (talk) 16:09, 17 July 2009 (UTC)[reply]
Sugar alcohols are not anything like hand sanitizer alcohol. It's not really the sugar alcohol acting like a poison. Cavities are caused by acid producing bacteria. These bacteria can't process Xylitol because it's not food so there isn't "alcohol's antibacterial effect is canceled by the sugar feeding effect".129.64.55.234 (talk) 16:39, 17 July 2009 (UTC)[reply]
Thanks. Do you know how they differ? I had thought the "alcohol" group at the end interferes with some part in the bacteria's membrane. Could you clarify the process and explain why it doesn't do what it does for sugar alcohols? Lisa4edit (talk) 17:50, 17 July 2009 (UTC)[reply]
Think you're just wrong. sugar, cellulose etc all have lots of alcohol groups, and they don't interfere with bacteria as far as I know. You should present some evidence that what you thought is actually true, rather than expecting people to provide responses based something that you haven't provided any evidence to be true.83.100.250.79 (talk) 18:22, 17 July 2009 (UTC)[reply]

looking for information on a type of water-powered hammer

I saw a type of water-powered hammer used for striking coins. It was water-powered, but not a trip hammer. It consisted of a big beam with a heavy weight on one end (the hammer) and a bucket on the other end. Water poured into the bucket, and when it would get full, it would sink down, lifting up the weight. Then, when it would get to the bottom, the water would pour out of the bucket and the hammer would drop. I thought it might be a trip hammer, but that's based on a cam and a rotating shaft, which this didn't have.

thanks, WhiteDragon (talk) 14:41, 17 July 2009 (UTC)[reply]

I wouldn't refer to that as a water-powered hammer. I would refer to it as a gravity-powered hammer. It doesn't matter what raises the hammer (water, rocks, a mule...), it is gravity forcing the hammer down to produce the desired result. Unfortunately, Googling for "gravity hammer" will not return any useful results. -- kainaw 16:39, 17 July 2009 (UTC)[reply]
The drive is under Water scoop (hydropower). There's no mention on specific names for the tools/equipment driven, though. Lisa4edit (talk) 16:53, 17 July 2009 (UTC)[reply]

Mercury again

When did it become common knowledge among reasonably well-educated people that mercury is highly toxic? When I was a child, a broken thermometer meant *FUN*, since my mother would let my brother and me play with the mercury that came out. Its behavior was so alien (the little spheres coalescing into larger spheres, etc.), that it provided hours of entertainment. Now when I hear of a mercury leak at a school they shut the whole thing down and send in the guys in hazmat suits. Should my mother have known better in the late 1970s? If it matters, she was Rhodesian/Zimbabwean, a nurse, and we lived in Brazil. Thanks. --Sean 23:11, 17 July 2009 (UTC)[reply]

It was already a big deal when a thermometer broke in our house in the late 50s early 60s. People have varying attitudes toward risks. Risk assessment is not just done in the documented orderly way described in out article, but happens in our heads pretty much constantly. Different people and cultures come to different results. We happily take our kids on trips in cars, but cringe or cry child abuse when we see someone hand a machete to a toddler in a stroller. The comparatively delayed and remote risk of mercury poisoning may have paled when compared to more immediate severe risks your mom encountered every day. You may still have fared better than some who made a big fuss, because one method of removal was vacuuming up the mercury, which of course made the problem a whole lot worse. Out of sight and out of mind wasn't out of the air. Lisa4edit (talk) 16:37, 17 July 2009 (UTC)[reply]
Years ago it was common for high school chemistry students to play with mercury, rolling it around in the hand. If a thermometer broke, kids played with the glob of mercury rolling it around on the floor until it disappeared into a crack. If it broke in a kids mouth, they just spat out the mercury and glass fragments. It was claimed (by a family doctor years ago) that the elemental mercury in the thermometer was not in a form easily absorbed by the body. On the Mr. Wizard TV show in the 1950's the kids learning about science were commonly exposed to mercury, as in the electromagnetic "jumping spring" where it was used for electrical contact. The splashing and sparking could have put a fair amount into the air. Many electrical experiments/demonstrations work vastly better with mercury than with brine or just metal to metal contact. It is hard to replicate some of the early electrical work of Michael Faraday without mercury for a friction-free highly conductive electrical connection. Probably in the 1970's some of these science demos became things the kids read about instead of things they saw or did. Likewise mercury manometers were previously used in high school physics and chem labs to measure pressure or to demonstrate barometer function. Perhaps dial or electronic pressure transducers replaced them. It was well known in the 1800's that chronic mercury exposure caused brain damage ("Mad as a hatter") but it was probably assumed that brief exposure did not have lasting effects. Mercury/silver amalgam fillings are still in millions of mouths. When did they stop putting mercury in tooth fillings? Edison (talk) 19:13, 17 July 2009 (UTC)[reply]
When I took high school chemistry in the early 1970s it was still the way Edison describes it. So mercury is toxic; big deal, you won't get enough exposure to matter. Today the attitude is to err much more on the side of caution. --Anon, 19:21 UTC, July 17, 2009.
I found an article from December 1963 about the U.S. Public Health Service banning a toy which contained drops of mercury. It says "If the toy should break and scatter mercury beads in hidden areas the fumes from the element could cause brain damage." A 1960 article behind paywall also cites a toy pulled from the market by authorities because it contained mercury.So I'd say it was "common knowledge among well informed persons " by then. But pretty much ignored by most for years after, like the dangers of smoking or the need to wear seat belts when driving. By 1970 there were numerous stories about the danger of trace amounts of mercury in fish and mercury vapor from spills. In the 1960's the U.S. used more mercury in pesticides than other countries, which were banning it after health problems were noted. Edison (talk) 19:31, 17 July 2009 (UTC)[reply]
In 24 BI (before internet), facts did not spread so easily. Whereas today we can google information on mercury, or other things to find information, back then it was much more difficult. Googlemeister (talk) 19:54, 17 July 2009 (UTC)[reply]
Which date exactly are you using as the start of the internet epoch? As far as I can tell, Unix epoch is the closest thing - the "internet" has no clean event which marks its beginning (first router? first TCP packet? first domain name lookup? first www server? first html render?) Any of these types of events can be counted as the start of the internet. Nimur (talk) 22:18, 17 July 2009 (UTC) [reply]
Your mention of car seats gave me a chuckle, since my mom also put us kids in "car seats" similar to old-skool drive-in trays, where we were just kind of hooked loosely onto the back of the driver's bench seat for reasons not of safety, but of getting a better view out the windshield. :) --Sean 23:11, 17 July 2009 (UTC)[reply]
I played with lawn darts! (Hint: when the other guy throws, it's good to be paying attention.) And had a chemistry set with dozens of poisonous substances. And rode a bicycle hundreds of times and never had a helmet or padded knees and elbows. And speaking of bench seats, remember when as a kid in the back seat you could roll down your window more than an inch and a half? Everybody's afraid of everything now. 69.245.227.37 (talk) 07:25, 18 July 2009 (UTC)[reply]
I also played with a chemistry set that had a whole bunch of poisonous substances, like NaOH pellets, 3M HCl, two different ferrocyanates, and copper sulfate (I remember wondering what copper sulfate tastes like, but my dad told me that it's poison, so I didn't try it). Also, I played with mercury when I was little (and I'm none the worse for it, I must say). And I actually raced my bike, downhill on a busy street, without any helmet (and then braked it with my sneakers, it had a really unreliable coaster brake that didn't always work). And once I rode on a train's rear buffers with a 50-pound pack on my back. And another time I flew over the mountains of Alaska in a tiny little single-engine plane (didn't do any piloting, though). And occasionally I've handled (dilute) HF with my bare hands -- no gloves, no goggles, no nothing! Must I add that I'm into martial arts? Reckless?! No, just taking calculated risks. 76.21.37.87 (talk) 07:58, 18 July 2009 (UTC)[reply]
As far as I know, organic compounds of mercury are highly toxic and accumulate in the brain. Metallic mercury or its ions are still toxic but much less and is a hazard mainly for the kidneys, as good or bad as most heavy metals. Many people living today have mercury in there teeth. Compact fluorescent lamps contain mercury, too. I guess the released amount from a shattered one is in about the order as the vapours from a shattered mercury thermometer. So I wouldn't panic when a thermometer breaks. The other thing is the mercury drop going down the sewer and finally distribute itself to the environment. 93.132.138.254 (talk) 08:22, 18 July 2009 (UTC)[reply]
I agree with you 100% that the hazards of metallic mercury are overrated and that it's not such a big deal if a thermometer or a light tube breaks. But I must warn you that dumping mercury down the sewer is a bad idea cause it'll go into the water and any fishes that swallow some will turn it into methylmercury and get poisoned (maybe even die) and also become poisonous for us people to eat. If I had a mercury thermometer (light tube, Ignitron, whatever) and it broke, I'd just gather up the mercury into a jar and turn it in for toxic waste disposal.

76.21.37.87 (talk) 23:51, 18 July 2009 (UTC)[reply]

Oh, and I'd wash my hands afterward with soap and water.  :-) 76.21.37.87 (talk) 00:24, 19 July 2009 (UTC)[reply]

ethyl acetate formation by cooking?

Is the pH of vinegar too high to attempt Fischer esterification with ethanol and vinegar over a hot frying pan? The yield prolly wouldn't be too good, but since what I'm aiming for is flavor detection, an equilibrium product of 0.1% (at room temperature) would do very well.

Or is this something to be done in barrels where you wait for years? John Riemann Soong (talk) 15:25, 17 July 2009 (UTC)[reply]

There is this story on some wikipedia page about a mother warning her kid about lots of things not to do while she's away including not to put jelly beans up his nose. He had never done that so he immediately had to try. So, I won't link the page and I hope you don't want to know this for any unhealthy reasons that came to my mind. Lisa4edit (talk) 16:22, 17 July 2009 (UTC)[reply]
I don't think pH has as much to do with it as Ka which measures the strength of the acid. The acid catalysts listed in Fischer esterification have very low Kas. Maybe you should try it. The reaction takes a long time in aging wines but if there's heat applied, the reaction might speed up and produce just enough for your needs. —Preceding unsigned comment added by Coolotter88 (talkcontribs) 16:57, 17 July 2009 (UTC)[reply]

You actually need to know the equilibrium constant for

Acetic acid + ethanol <> ethyl acetate + water

Acetic acid will promote this reaction - even Ammonium Acetate can promote this type of reaction - however the speed of the reaction may be slow - I recommend a pressure cooker - doing this you will get an equilibrium amount of ethyl acetate - whatever that is - the trick is to have as little water in as possible.83.100.250.79 (talk) 19:25, 17 July 2009 (UTC)[reply]

Don't try that at home. ...and Wikipedia:Don't stuff beans up your nose. 71.236.26.74 (talk) 22:22, 17 July 2009 (UTC)[reply]


Is Ka that important? See I thought it was H+ concentration that catalysed the reaction but possibly I guess acid strength would be important to help prevent a reverse reaction even in favourable pH. Also, it's ethanol and vinegar -- common edible solvents -- and I expect the majority of my yield would be my starting reactants (discounting evaporation) -- why would it be dangerous? I mean, I add vinegar to all sorts of ingredients with active essential oils and flavors all the time. If I added something that was pseudohygroscopic -- flour or some kind of starch -- I guess that would help absorb the water? (It would also fry, but hey.) John Riemann Soong (talk) 23:13, 17 July 2009 (UTC)[reply]

Acidic bitterness

How can lidocaine have been identified as the source of a bitter taste, when it comes in a solution of HCl, which is acid? I thought acid is sour and bitterness is associated with alkaline? DRosenbach (Talk | Contribs) 17:15, 17 July 2009 (UTC)[reply]

Hydrochloride should provide some insight. Coolotter88 (talk) 17:47, 17 July 2009 (UTC)[reply]
One way: the taste occurs when lidocaine is injected intravenously or subcutaneously. There need be no direct involvement of taste sensory neurons, as there might be if topically applied, and there is no change in pH when injected, as the HCl is quickly dissociated and buffered. - Nunh-huh 21:48, 17 July 2009 (UTC) (Addendum: pH of Lidocaine HCl injectable without epinephrine is about 5.0-7.0 in the bottle).[reply]
My response is a real answer to your question. Perhaps if you can tell me why you think it is not, I can explain it more clearly. Or perhaps the problem is my understanding of you question, in which case perhaps you can explain it more clearly. The pH of injectable lidocaine with epinephrine is about 3.3-5.0 - Nunh-huh 09:29, 21 July 2009 (UTC)[reply]
The article says it is a salt of Hydrochloric Acid. I'm not sure but I don't think there is any Hydrochloric Acid being injected in your mouth. The slightly less than 7 pH of Lidocaine HCl is probably because HCl is reacted with a weak base. Coolotter88 (talk) 16:52, 20 July 2009 (UTC)[reply]

colour of the moon

Hi there!! I would like to know what material on the moon's surface makes it white. Or is it just the strong light of the sun that makes it so? Please help me out.--Lightfreak (talk) 17:55, 17 July 2009 (UTC)[reply]

The moon is mostly Feldspar which is white. But there are also black rocks too.83.100.250.79 (talk) 18:16, 17 July 2009 (UTC)[reply]
This is an image of the moon showing how bright the bright parts are and how dark the dark parts are (But see comment at (XX))
The sliver of moon rock at the air and space museum

The moon is not white; it's black, or more precisely very dark gray. It only reflects about 10% of the light that hits it -- an object you would call white would reflect something more like 90%. If you visit the National Air and Space Museum, they have a slice of moon rock you can touch, and you can see the color.

However, human color perception is relative. An object is judged to be light or dark in comparison to the things around it. This means that when an object is seen against a much darker background, like a light in the sky, it is impossible to see it as having a "dark" color like brown or navy blue or dark gray. You see red or yellow or blue or, in this case, white.

--Anonymous, 19:33 UTC, July 17, 2009.

This picture shows you how appearances can be deceiving with regards to brightness. Vimescarrot (talk) 19:46, 17 July 2009 (UTC)[reply]
added image above 83.100.250.79 (talk) 20:32, 17 July 2009 (UTC)[reply]
You can also visit the article feldspar and look at an image of a lighter moon rock ! 83.100.250.79 (talk) 20:33, 17 July 2009 (UTC)[reply]

(XX) The image caption "showing how bright the bright parts are" is misleading. All large areas of the Moon are black to dark gray, it's just that some parts are even darker than others. The photo was made with an exposure sufficient to show detail on the surface -- the same sort of compensation that our brain does automatically, so it reproduces the way the Moon appears to the eye. If there was a true white object in the photo, large enough to see, it would look grossly overexposed. If the exposure was made correct for the white object, then the image would show how dark the Moon really is. --Anon, 22:48 UTC, July 17, 2009.

You probably want this [16] image .83.100.250.79 (talk) 00:18, 18 July 2009 (UTC)[reply]
And by the way - you are very weird - a picture of a whitish moon is no good, but a picture of someone touching a triangle is. Fucking wierdo.83.100.250.79 (talk) 00:26, 18 July 2009 (UTC)[reply]

So basically, it is the strong light of the sun as well as the dark background that makes it appear white. Thanks!!!--Lightfreak (talk) 11:12, 18 July 2009 (UTC)[reply]

If you ever visit the Exploratorium in San Francisco, California, they have an exhibit that consists of a picture of the moon that has accurate reflectivity. Compared to other objects around it, it looks very dark. But you can flip a switch that isolates the background to darkness and shines a bright light on the moon, and it looks bright, just as it does in real life. The difference is striking. (At least, this exhibit was there several years ago.) -- Coneslayer (talk) 15:42, 20 July 2009 (UTC)[reply]
If you want to know what the color of moon "really', what the colors of planets "really" is very tricky. Human perception changes depending on the enviornmental conditions. We have had this discussion in the past--69.228.145.50 (talk) 00:55, 21 July 2009 (UTC)[reply]
Moon article says there are spherical glass beads in moondust that make it brighter. Jay (talk) 10:40, 22 July 2009 (UTC)[reply]
Somewhere I've seen a recent snapshot of Earth and Moon together as seen by some space-probe; the Moon is brown. —Tamfang (talk) 02:21, 30 July 2009 (UTC)[reply]

green chlorophyll

Why is chlorophyll green? --Lightfreak (talk) 18:14, 17 July 2009 (UTC)[reply]

Our chlorophyll article has some info about the optical properties. The nature of any molecule is to absorb only certain wavelengths (colors) of light (affected by structure, pi bonds, non-bonded electrons, and metal atoms, among others). The nature of chlorophyll is to absorb several "non-green" colors, so what's left is green. DMacks (talk) 19:38, 17 July 2009 (UTC)[reply]
That still leaves the question of why chlorophyll absorbs those non-green colors and not green. (Why hasn't evolution "tuned" chlorophyll to absorb green light.) Turns out that short wavelength light (the blues and violets) contain a lot of energy, so it makes sense to absorb those. On the other hand, as a blackbody radiator, the sun puts out most of its radiation (number of photons) in the lower wavelength region (the reds, oranges, and yellows), so you'd want to capture that as well. What's left is the stuff in the middle (green), which isn't particularly energetic, nor particularly prevalent. While it would be best to capture those wavelengths too (making plants black), they don't give enough of a benefit to drive evolution to that point. This analysis changes with different the different emission profiles of different stars. I vaguely recall news reports about research which hypothesized what colors plants on extra-solar planets would be, given the above considerations. -- 128.104.112.87 (talk) 19:57, 17 July 2009 (UTC)[reply]
In respect to the many colors of the spectrum, chlorophyll, while being the most prevalent pigmented substance in plant cells, is not the only. Xanthophylls (yellow) and carotenoids (orange/red) also exist -- they are what cause dying leaves to appear other colors besides green once the chlorophyll begins to disappear at the end of the season. All together, they allow leaves to absorb and use light of all colors, albeit in low levels. DRosenbach (Talk | Contribs) 19:35, 19 July 2009 (UTC)[reply]
Actually I believe it's because of the constraint of the photoreceptors, and because chlorophyll evolved in the ocean (and established enzymes are hard to modify), and the ocean provides a color filter of its own. I believe there's an explanation that states that green light in sunlight was actually too intensive in the ocean for it to be useful -- in the sense that it would be overly damaging to the pigment itself, so chlorophyll evolved in a way *not* to absorb too much of it. If you look at http://en.wikipedia.org/wiki/File:Solar_Spectrum.png you can see that sunlight actually has a lot of green light relative to the other wavelengths. John Riemann Soong (talk) 23:08, 17 July 2009 (UTC)[reply]
This previous discussion relates and has some interesting points including a link to this article about an early "purple Earth" theory with the world dominated by retinal. - Draeco (talk) 04:23, 21 July 2009 (UTC)[reply]

Cigarettes against tear gas?

Per a news story[17], antigovernment protestors in Tehran were "lighting cigarettes and putting them in front of one another's faces to ward off the effects of the tear gas." Is there any merit to this belief? I was told that tear gas effects are worse if you rub the facial skin and especially the eyes, so swim goggles are useful to keep it out of the eyes, and that since it is a powder and not a gas, an N95 respirator mask provides some protection to the lungs, better than a bandanna over the mouth and nose but clearly not as good as a gas mask. I was also told that water to flush the eyes was useful. But what possible protective effect would come from cigarette smoke around the face? Edison (talk) 18:40, 17 July 2009 (UTC)[reply]

Vaseline on the skin is also an old recommendation from demonstration organizers, but about as many sources say it makes the teargas worse, sine the material gets dissolved in the petroleum jelly. Other demonstration organizers in the past recommended vinegar or baking soda[18]. Any reliable sources? Not much useful and referenced info in the Tear gas article. Edison (talk) 18:57, 17 July 2009 (UTC)[reply]

I don't know - but I know that heavy smokers can have a reduced response to some stimulants/allergens (tear gas stimulates a response rather than actually being harfull I think) - eg smoking can stop allergies such as pollen hay fever.. However I think you need to actually smoke some cigarettes for this to work.. Just a guess.83.100.250.79 (talk) 19:21, 17 July 2009 (UTC)[reply]

Conversely, smoking (or smokers...not sure if it's an effect of the smoking at the time or of the result of some long-term response to smoking) reportedly makes one more able to detect cyanide. Not that cyanide is being used on the protesters (I hope!). DMacks (talk) 19:41, 17 July 2009 (UTC)[reply]
WARNING, ANECDOTAL EVIDENCE FOLLOWS I used to be in the United States Air Force. Airborne Battle Management to be exact. Since we were flying around, directly above the battlefield they had us go through the same training the Combat Controllers (the USAF's version of Navy Seals, Army Rangers, etc). This training included frequent trips to the gas chambers. I believe the gas in the chambers was CS gas, but they also used tear gas on us occasionally. I am, and have been for a long time, a heavy smoker. Before I went in, I smoked a cigarette. When I came out, I smoked a ciggarette. My eyes watered. My friends were dripping with mucous.
I'd like to believe I was just a badass, but smoking probably does reduce the effects of the gases.Drew Smith What I've done 04:14, 18 July 2009 (UTC)[reply]
It could be that either (a) the cigarette smoke desensitizes the mucous membranes, making them less susceptible to the effects of the tear gas; or (b) the nicotine actually has a toxic effect that is opposite to that of the tear gas, and so acts as an antidote. I'm not really sure about that last part, I got to look it up. BTW, CS gas is tear gas.

76.21.37.87 (talk) 04:57, 18 July 2009 (UTC)[reply]

Just looked it up -- nicotine does stimulate the sympathetic nervous system, which would tend to suppress tear secretion -- so you could say that it acts like an antidote. FWIW

76.21.37.87 (talk) 06:01, 18 July 2009 (UTC)[reply]

What does sine (mathematical equations?) got to do with this?174.3.103.39 (talk) 06:11, 18 July 2009 (UTC)[reply]
Where did you see the word "sine"??? 76.21.37.87 (talk) 06:50, 18 July 2009 (UTC)[reply]
It's in Edison's first answer second question above, and is very obviously a trivial mis-type of "since." 87.81.230.195 (talk) 13:52, 18 July 2009 (UTC)[reply]
If you carry a sine in a riot they hit you on the head, besides throwing tear gas. I saw a cop do this, and he was obviously outdoors a lot because he was a tan gent. Edison (talk) 20:02, 18 July 2009 (UTC)[reply]
"If you carry a sine in a riot they hit you on the head, besides throwing tear gas." -- Especially if it's an obtuse sine... :-D 76.21.37.87 (talk) 23:58, 18 July 2009 (UTC)[reply]
"I saw a cop do this, and he was obviously outdoors a lot because he was a tan gent." -- Oh, I get it now (didn't at first)! HAHAHAHAHA! ROFLMAO! 76.21.37.87 (talk) 00:56, 19 July 2009 (UTC)[reply]
Someone should have to cosign before you make such an acutely horrible pun. --Sean 14:18, 20 July 2009 (UTC)[reply]
Good one! ROFLMAO!!! X-D 76.21.37.87 (talk) 04:56, 21 July 2009 (UTC)[reply]

airspeed indicators

Why do most airspeed indicators have the unit knots, even when they are not naval aircraft? Why not mph or kmph? Googlemeister (talk) 19:24, 17 July 2009 (UTC)[reply]

See the article Knot (unit), in particular the section on Modern-use. ny156uk (talk) 19:48, 17 July 2009 (UTC)[reply]

Ok, why was it changed from miles to knots? Googlemeister (talk) 19:51, 17 July 2009 (UTC)[reply]
From the article:
...its retention for nautical and aviation use is important for navigational reasons, since the length of a nautical mile is almost identical to a minute of latitude. As a result, distance in nautical miles on a navigational chart can easily be measured by using dividers and the latitude indicators on the side of the chart.
It's not clear to me that it was "changed" from mph, but I don't know the history. Tempshill (talk) 19:56, 17 July 2009 (UTC)[reply]
The article mentions that the US used to use miles and then changed, but that could well be an American aberration. Algebraist 20:34, 17 July 2009 (UTC)[reply]
In an era of cheap calculators and flight control radar, what is the benefit of something being "almost identical to a minute of latitude?" Does that "almost" keep you from smacking into a mountain in the clouds, or "almost" get you to a landing strip on some speck of land in the Pacific before you run out of fuel? It's hard to see why such approximate navigation would be very useful today. Edison (talk) 21:41, 17 July 2009 (UTC)[reply]
Useful? It's useful because it is a functional and standardized measurement of airspeed. Unless you are asking, "why doesn't the worldwide aviation system shift from knots to kph as a measurement of airspeed," which is a different question. Tempshill (talk) 22:15, 17 July 2009 (UTC)[reply]
It's basically just history - the seafaring roots of the nomenclature and the (then) convenience when using maps for long distance travel. Nowadays, it's just something that's way too embedded in expensive physical objects (planes, control towers, etc) and in the brains of their operators to change. Aircraft often remain in service for 30 years - and their pilots are around for a similar amount of time - any change-over period would take decades to achieve. Worse still, it would require approval internationally since aircraft spend a lot of time going from one country to another. They also measure altitude in feet rather than meters. It's hard to imagine what benefit there would be from such a costly and difficult change. SteveBaker (talk) 00:03, 18 July 2009 (UTC)[reply]

Stomach aches and depression

I recall reading something years ago about how because of the way your brain is wired, physical pain in the stomach can lead to emotional depression. sounds kinda fishy, but i remembered reading it because i'm profoundly hungover and also a bit bummed out. thanks 70.122.109.186 (talk) 19:55, 17 July 2009 (UTC)[reply]

Sorry, the Refdesk doesn't diagnose conditions or give medical advice. Please consult a doctor. Tempshill (talk) 19:57, 17 July 2009 (UTC)[reply]
Any continual pain or illness or disability can lead to depression potentially - but I'm not aware of anything specific about the stomach, and doubt it very much.83.100.250.79 (talk) 20:22, 17 July 2009 (UTC)[reply]
While we can't diagnose your specific condition, you are also asking if this link might be true. I doubt it. The problem (and one reason why we can't give such advice) is that depression can be caused by physical (brain chemicals) or psychological (ill family member, etc.) causes. The latter, of course, being more about the fear of loss, the empty part of your life if that person dies, etc.; yes, there is a "feeling in your gut" that can be related to concern over that family member, but it would be a symptom, not a cause, of depression.
Also, see our article on Memory - what you read might be coming to you in a different form than you read it. Maybe that's why it seems fishy; you're not recalling it the way it was.
Finally, if you are "profoundly hung over" from a drinking binge, for instance, you may be experiencing guilt - and feel you shouldn't have done that. That can cause pain in your gut, too. In which case, I will refer you to the only medical advice on our site."It hurts when I do this." "Then don't do that. :-) Somebody or his brother (talk) 00:11, 18 July 2009 (UTC)[reply]
Nietzsche wrote something to the effect of 'the Holy Spirit resides in the entrails'. I couldn't agree more. Though when you're suffering from indigestion and depression you probably aren't too interested in philosophy. I recommend cutting back on your meat intake until you feel better. It's not trivial to digest, and there are a lot of unscrupulous men out there who will raise profoundly unhealthy animals to be fed to undiscriminating people. It's an evil business sometimes but it's been the way of things for so long that nobody bats an eye. Vranak (talk) 23:58, 18 July 2009 (UTC)[reply]


July 18

Car incident types

Quotation: "Lane change accidents are noted as one of commercial vehicle fleet's top 3 accident types." Anyone know(or at least can guess) what the two others accident types? Vitall (talk) 04:39, 18 July 2009 (UTC)[reply]

Rear-enders are the most common kind of car crashes, I'm sure of that. WP:OR: I've myself been rear-ended twice within a year or so, once by an 18-wheeler (OUCH!!!) and the second time by a jeep. FWIW 76.21.37.87 (talk) 05:02, 18 July 2009 (UTC)[reply]

There would of course be more than one way to define "accident type". However, from page 31 of this document, it looks like rear ending is indeed the most common accident type, and the second most common is hitting a fixed object. The referenced document lists the third most common accident type as “side swipe (opposite or same direction)”, which does seem like it would include a typical lane change accident. Red Act (talk) 08:52, 18 July 2009 (UTC)[reply]
... which clearly indicates that roads are too short rather than too narrow ...  ;-) 93.132.138.254 (talk) 16:49, 18 July 2009 (UTC)[reply]

Miscarriage question

First, I am a regular user, but for obvious reasons I am not logging in to post this question. Second, I am not asking for medical advice, but for artices (WP or other) relating to the question.

A few years ago I was with a girl who was a frequent liar. The first time she claimed to be pregnant I was 16, and believed her. About a month later she claimed to have had a miscarriage. I believed her. The second time, I was probably 17. I wasn't so sure about it this time. Soon after she claimed to have had another miscarriage. This happened a few times more, and by the time I left her I didn't believe that she had ever been pregnant.

Now, my current girlfriend is going through a miscarriage. At about 4 weeks into the pregnancy.

Now I'm wondering if my first girlfriend may have been telling the truth, and the miscarriages are somehow related to me. Is there any connection between males and miscarriages?75.93.119.255 (talk) 05:45, 18 July 2009 (UTC)[reply]

I'm not an obstetrician, so please don't take this as medical advice, but the only way a male could present a risk factor for miscarriage is if he's a heavy smoker (if he smokes more than 2 packs a day, then his wife / girlfriend could be up to 3 times more likely to have a miscarriage). However, if you're not a heavy smoker, then there's no conceivable link between you and your current girlfriend's miscarriage. In this latter case, the current miscarriage is not your fault (or hers, for that matter), and your first (ex)-girlfriend is a damned rotten liar. FWiW 76.21.37.87 (talk) 06:47, 18 July 2009 (UTC)[reply]

From personal experience here: having had at least 5 misses myself, and talking to doctors about why, it seems that about 50% of all pregnancies end in miscarriage: sometimes the period is just a few days late but there is still a pregnancy which has ended. So don't blame yourself for this. It's a natural feature of being a woman. Oh - and you may wish to consider using contraception to prevent pregnancy and therefore miscarriage occurring in future. --TammyMoet (talk) 08:32, 18 July 2009 (UTC)[reply]
I'm also not an obstetrician but I would have to disagree with 76. There's no reason why a male can't be a factor in miscarriages. If the miscarriages are caused by genetic defects (e.g. chromosomal abnormalities), then that could be either the male or the female. E.g. [19] [20] [21] [22]. Also there's no way we can know that 'your first (ex)-girlfriend is a damned rotten liar' as 76 suggested. Even if you aren't a factor, it easily possible your ex-girlfriend had a fertility problem of her own. I woulld also emphasise TammyMoet's point. Miscarriages are hardly uncommon and probably more common at such a young age as your first relationship (although 4+ is fairly unlikely without some sort of problem, that could still go away naturally). And if you don't want to have a baby, you need to look at more efficient contraception since it sounds like whatever you've been doing hasn't been working. And finally, if you are concerned about your fertility, you should see a specialist. Nil Einne (talk) 10:14, 18 July 2009 (UTC)[reply]
OK, I take back the "damned rotten liar" part. However, if there are 5 miscarriages in a row, it's highly unlikely that genetic defects are a factor in all of them. No, 75 IP's genes can almost definitely be ruled out as a factor, so assuming that 75 IP is not a heavy smoker (which would be a factor) and that his first ex-girlfriend was telling the truth (which she might or might not have), then it would mean that she had an infertility problem of her own (as Tammy pointed out) and that the current miscarriage was simply a random misfortune. So, to recap: if 75 IP is a heavy smoker, he should quit smoking or at least cut back to less than a pack a day; if not, then he is not a factor in his girlfriend's miscarriage.

76.21.37.87 (talk) 00:12, 19 July 2009 (UTC)[reply]

Well, just to be a smartass, it doesn't appear that I need any contraceptives. But everything else makes sense. And it was 4 in 2 years. 6, if you count both sets of twins she claimed... Now you understand why I didn't believe her. Still don't, but I'm wondering if maybe one of them was real...75.93.119.255 (talk) 10:54, 18 July 2009 (UTC)[reply]
Carriers of a Balanced translocation can be at high risk to have a fetus with an unbalanced chromosome content thus leading either to frequent miscarriages or children with disability. If the information you were given by your partners is correct then I would follow Nil Einne's recommendation to see a specialist. --- Medical geneticist (talk) 11:53, 18 July 2009 (UTC)[reply]
I know you're being a smartass but believe me, miscarriages are not trivial, and indeed carry risks to the health of the mother. If you do have something wrong with you which causes your progeny to spontaneously abort, the least you could do is prevent the pregnancy occurring in the first place. And you'll not find out if there is something wrong until you see a specialist. Have some common decency man! --TammyMoet (talk) 12:35, 18 July 2009 (UTC)[reply]
Looks like he's one of those people who don't have to work themselves for the money there offspring needs, not to mention other care. 93.132.138.254 (talk) 15:43, 18 July 2009 (UTC)[reply]
This is 75.93.119.255. Just for the record, my second girlfriend already had a baby when we started dating. And I worked my ass off to provide for them both. Please remove the personal attack, it was highly uncalled for.Drew Smith What I've done 22:18, 18 July 2009 (UTC)[reply]
Twins are usually diagnosed by ultrasound, somewhere between 14 and 20 weeks. A miscarriage at that stage is serious, often requires hospitalisation, and would be hard to miss by anyone seeing the patient on a nearly daily basis. Girls whose periods are not totally regular may imagine themselves to be pregnant for a few days' lateness, then imagine a miscarriage when the period does start. An amount of wishful thinking may also involved. together with testing out the reactions of boyfriends and family.
There seem to be slightly more miscarriages of male foetuses where an earlier pregnancy (completed or not) was also male. - 220.101.166.151 (talk) 03:12, 19 July 2009 (UTC)[reply]
I don't see how you can rule it out. As MG has said, there is at least one condition, and probably more which can lead to frequent miscarriages (and of course if the child is born, it's likely to be suffering some sort of nasty condition). Perhaps you are under the mistaken presumption we're talking about 5 random events. Clearly we're not. We're obviously considering a defect that exists in the germline (whether because of a mutation that arose early on in development or whether it also occurs in the somatic cell line and the OP is a carrier of the condition is irrelevant) and therefore would be present in many or all sperm. It's also worth remembering we may not be talking about an all or nothing situation here. It's possible some of the miscarriages were genuine and some weren't (e.g. it's easy to imagine that if say the first two were and because of the effect they had, the person then made up the next few cases (obviously the person would still be a liar in that case). What case is the most likely is speculatory so not really for the RD but my point being there's ultimately a lot of possibilities. N.B. If you are uncertain if it's worth seeing a specialist, and you still know how to get in to contact with the ex, it may be worth explaining the situation to them making clear why you want to know and that you have no intention of telling anyone else etc to try and find out the truth about these previous alleged miscarriages. Nil Einne (talk) 17:31, 19 July 2009 (UTC)[reply]
It could still be that his first girlfriend had an infertility problem of her own, which would explain the first 4 miscarriages and leave only the last 1 as a "random event". Or maybe smoking was to blame after all -- I know that the OP smokes tobacco (see "Cigarettes against tear gas"), don't know how much. (BTW, it's only likely to be a factor if it's more than 1 pack a day.) Or perhaps his first girlfriend WAS a dirty rotten liar and lied all 4 times about having a miscarriage, which would also explain a lot. FWiW 76.21.37.87 (talk) 01:20, 20 July 2009 (UTC)[reply]
Yes, I smoke alot now, but not when I was with the first girl. I guess it's still more likely to be a coincidence. If it happens with the next girl though, I'm seeing a doctor.Drew Smith What I've done 05:20, 20 July 2009 (UTC)[reply]

I’m rather shocked this question hasn’t been deleted yet.

Some pertinent information from this page: Repeated miscarriages are one of the signs that a miscarriage may have been due to genetic defects passed on by a parent. The source defines “repeated” miscarriages as being three or more miscarriages. Only 1 in 200 women have repeated miscarriages. Another sign that a miscarriage may have been due to genetic defects passed on by a parent is if a parent has a child or relatives who have birth defects. Genetic testing (I presume meaning of the parents) can help the doctor identify the problem.

In my opinion, what we should really be saying here is “See a doctor.” Red Act (talk) 11:03, 20 July 2009 (UTC)[reply]

"Second, I am not asking for medical advice, but for articles (WP or other) relating to the question."Drew Smith What I've done 09:22, 21 July 2009 (UTC)[reply]
In that case, the article "Miscarriage" should have the info you're looking for. 76.21.37.87 (talk) 23:26, 21 July 2009 (UTC)[reply]

Can all animals store fat?

And if so, is it possible for any animal to become obese? ScienceApe (talk) 07:20, 18 July 2009 (UTC)[reply]

All vertebrates can store fat, and cats in particular can become obese (I don't really know about other animals). FWiW 76.21.37.87 (talk) 07:41, 18 July 2009 (UTC)[reply]

Of course they can, labradors (dogs) tend to fattyness, bears store fat for hibernation, etc etc.83.100.250.79 (talk) 11:18, 18 July 2009 (UTC)[reply]

I don't know about "of course". I don't know if you can find obese locusts, for example. Or overweight sponges. Vimescarrot (talk) 11:53, 18 July 2009 (UTC)[reply]
Sorry, my mistake - I didn't notice it said "any animal".83.100.250.79 (talk) 11:58, 18 July 2009 (UTC)[reply]
Also, bears might not hibernate. The Book of Animal Ignorance says it's "torpor"; our article on that says torpor is hibernation, but there's still some debate as to whether black bears do it. Though you're right, they do layer up fat before they do it. </trivia> Vimescarrot (talk) 11:57, 18 July 2009 (UTC)[reply]
Fish etc don't become obese as mammals do. (Actually some sites say fish can become obese - but I'm not sure - they can develope poor morphology due to lack of exercise - but I'm not sure about 'obese' )83.100.250.79 (talk) 12:15, 18 July 2009 (UTC)[reply]
Ants cannot become obese because their exoskeleton prevents them from doing so.83.100.250.79 (talk) 13:01, 18 July 2009 (UTC) However they can store fat [23] 83.100.250.79 (talk) 13:04, 18 July 2009 (UTC)[reply]
Hmm... seeing how much fat ants can actually store would make an interesting research project for some entomology graduate student. I can see the paper now: "subjects were fed Snickers (Mars Incorporated, Hackettstown, NJ) three times daily..." – ClockworkSoul 15:19, 18 July 2009 (UTC)[reply]
Even the humble Amoeba can slow the signs of obesity

If specimens of Amoeba proteus are fed exclusively on colpidia, they become very large and extremely fat and sluggish ..[24]

They can store energy as glycogen or fat:

Fat and glycogen are two distinct types of storage material present in Amoeba proteus[25]

83.100.250.79 (talk) 13:40, 18 July 2009 (UTC)[reply]
Amoeba are protists, not animals. DRosenbach (Talk | Contribs) 19:46, 19 July 2009 (UTC)[reply]
There are some fish whose flesh contains hardly any fat at all -- haddock and cod, for example, are less than 1% fat. So, although I don't know for sure, I doubt that those animals would be able to store fat to any significant degree. Looie496 (talk) 05:36, 19 July 2009 (UTC)[reply]
What about Jellyfish? -- penubag  (talk) 06:25, 19 July 2009 (UTC)[reply]
The answer would be no -- obesity does not manifest in all types of animals. Unless by any, you did not meant to specify a universal generatlization that all animals can present as obese, but rather can any (at all) animals (i.e. non-humans) become obese, then the answer would certainly be yes. DRosenbach (Talk | Contribs) 19:46, 19 July 2009 (UTC)[reply]
So what happens when a Jellyfish or ant consumes a large amount of food or overeats so to speak? ScienceApe (talk) 03:55, 21 July 2009 (UTC)[reply]

Irreducibility of multivariate polynomials

I cannot find anything about that. I can think of two reasons for this: either it's utterly complicated and not well understood or it can be so easily reduced to the question of irreducibility of polynomials of a single variable that no one cares to tell. What is it? 93.132.138.254 (talk) 09:35, 18 July 2009 (UTC)[reply]

Wikipedia:Reference_desk/Mathematics#Irreducibility_of_multivariate_polynomials

You really should ask on the maths desk - I've copied your question to there.83.100.250.79 (talk) 11:20, 18 July 2009 (UTC)[reply]

I haven't ever got a useful answer there... 93.132.138.254 (talk) 14:46, 18 July 2009 (UTC)[reply]
Then ask for more explanations and details. - on the maths desk - because it's a maths problem.
Also try waiting a bit.83.100.250.79 (talk) 16:45, 18 July 2009 (UTC)[reply]
It's a math problem, yes. But on the IT desk and on the math desk are the people who are posing questions --- here on the science desk are the people who read questions: for fun, to broader their horizon, and, in regard of the previous, as kind of a nearly effortless side effect, help with ideas and answers. That's the difference I can't ignore. But when there is no more of an answer than "go and learn some math" then I see there's no place to put those kind of questions and it's only a waste of time. And I won't ask about what is meant be "leading term" in the article of Gröbner basis, not here nor there. 93.132.138.254 (talk) 17:29, 18 July 2009 (UTC)[reply]
  1. Yes, multivariate factoring is complicated.
  2. Yes, the math desk is the right place to ask.
  3. Wikipedia's coverage of symbolic algebra is actually not all that good (compared with other math subjects), but a google search for "multivariate factoring" may help you get started. 67.117.147.249 (talk) 00:59, 19 July 2009 (UTC)[reply]

Gravity

Did Newton have a theory for gravity, or did he just create a law for it? If Newton didn't, does that make Einstein the only one with a theory for gravity? --wj32 t/c 09:47, 18 July 2009 (UTC)[reply]

There isn't any difference between what you are saying. There isn't any difference between a 'law' and a 'theory' in science, there are only mathematical relationships that can be used to accurately predict outcomes. —Preceding unsigned comment added by 58.111.132.76 (talk) 09:55, 18 July 2009 (UTC)[reply]
No, a law is a statement whereas a theory is an explanation. If those aren't the definitions, then that's what I'm asking anyway. --wj32 t/c 11:03, 18 July 2009 (UTC)[reply]
If you think of a law as a mathematical framework to compute things, say F=Gm, and of a theory as a deeper (whatever that is) framework that helps to understand (whatever that is) how an effect comes to exist (say, you describe chemical bonds using the laws for the electrical field), then neither Newton nor Einstein did have a theory. The first puts in the mass, not understanding what this really is, the other puts in curvature of space-time, not explaining where this comes from. But the distinction is rather artificial and subjective. 93.132.138.254 (talk) 10:22, 18 July 2009 (UTC)[reply]
I get what you're saying, but then is there any possible way for gravity to have a "theory" behind it, since it's a fundamental force? --wj32 t/c 11:06, 18 July 2009 (UTC)[reply]
The distinction between a scientific law and a theory has rather vague, subjective boundaries, and part of the difference is just that the term “law” was more popular historically, and “theory” is more popular now. However, Einstein’s general theory of relativity is arguably more of a theory than Newton's law of gravity, in that there’s more of explanation given as to what is going on. Newton’s law of gravity, to modern eyes at least, is not much more than one equation, with a definition of what each of the terms in the equation is. General relativity consists of not only Einstein’s field equations, but the explanation that those field equations are describing how the stress-energy tensor causes a curvature in spacetime. But part of that difference is just a subjective matter of perspective. The fact that spacetime is curved is very unobvious, so Einstein’s explanation seems quite important. However, in Newton’s day, the explanation that every two point masses might attract each other was also a very unobvious observation. So they are arguably both theories. Red Act (talk) 11:23, 18 July 2009 (UTC)[reply]
OK, I guess it is a subjective thing, then. Thanks for the explanation. --wj32 t/c 11:28, 18 July 2009 (UTC)[reply]
All that people mean when they call gravity fundamental is that we don't currently explain it as a consequence of something else, not that we won't ever do so. Newton famously refused to speculate about the mechanism of the gravitational interaction (hypotheses non fingo), but I think he believed there was some mechanism because he didn't believe in direct action at a distance. In a way general relativity provided that mechanism by explaining Newton's action-at-a-distance law as a consequence of a (rather different looking) local law. It's not clear we should use the name "gravitation" for both. Newton's law of gravity wasn't the first either. The first law of gravity was the observation that things fall to the ground of their own accord, which must predate recorded history and probably even H. sapiens. Newton's discovery was universal gravitation, which is the fact that the same force that makes things fall to the ground is also responsible for the motion of the planets. In a sense by carrying the name "gravity" from one phenomenon to another we've decided by fiat that there will always be something fundamental called "gravity". -- BenRG (talk) 11:34, 18 July 2009 (UTC)[reply]
(After edit conflict) Newton deduced the law of universal gravitation and used it to explain and connect a variety of other phenomena, including the motion of the Moon, tides and Kepler's laws of planetary motion. He also proposed at least two different explanations for the origin of gravity itself - see Mechanical explanations of gravitation#Streams - but ultimately realised that these explanations were unsatisfactory. In 1713 he added the famous Hypotheses non fingo statement in an appendix to the second edition of the Philosophiæ Naturalis Principia Mathematica: "I have not as yet been able to discover the reason for these properties of gravity from phenomena, and I do not feign hypotheses". We still do not have a satisfactory "theory of gravity" in the sense of a deeper explanation that connects it with the other fundamental forces - although there are various candidates for the position. Grand unification theory successfully combines the other three fundamental forces within the Standard Model, and the search for a "theory of everything" that incorporates gravity within this framework is one of the biggest unsolved problems in physics. Gandalf61 (talk) 11:41, 18 July 2009 (UTC)[reply]

See Universal_gravitation#Newton's_reservations

Principia: "I have not yet been able to discover the cause of these properties of gravity from phenomena and I feign no hypotheses... It is enough that gravity does really exist and acts according to the laws I have explained, and that it abundantly serves to account for all the motions of celestial bodies

The same section also metions that he did "..invented two mechanical hypothesis in 1675 and 1717" - though it seems he wasn't happy with them - so he had guesses at theoretical origins of gravity, but nothing he was satisfied with.83.100.250.79 (talk) 11:47, 18 July 2009 (UTC)[reply]

It's obvious to me that the questioner is asking if newton had a hypothesis for the origin of the gravitational force (and by extension the form of the force/distance equation) - as such "Newton's aether-stream hypothesis" seems to be one of them - it doesn't seem (though I may be wrong) that he accepted gravity as a fundamental force, and would have wished to explain it in terms of some motion (ie an explanation via the laws of motion )at least that's what I think is the case83.100.250.79 (talk) 11:52, 18 July 2009 (UTC)[reply]

I can't find much more on it though this article [26] might help a little.83.100.250.79 (talk) 11:55, 18 July 2009 (UTC)[reply]
Newton's theory of gravity was that there was an force called Gravity. He had no explanation of what caused it on a physical level. He postulated that there was a thing called Gravity, it existed, and it could be understood with his equations. He was roundly criticized in his time for postulating an "occult" force, something that wasn't based in other known natural properties. Over the years of course people assimilated the force of gravity into their understanding of the natural properties of the universe, which is why we find it so unproblematic today, even though the Einsteinian explanation gets rid of the force altogether and makes it all a question of the path of least resistance in spacetime. --98.217.14.211 (talk) 14:15, 18 July 2009 (UTC)[reply]


The way I always distinguish: A scientific law is the simple equation, like V=IR or U=GMm/r. When you plot your data, you get a cluster of points around the "law", and a least-squares fit will re-derive the law. A "scientific theory" is the technique you use to pick which law you need to plot. Nimur (talk) 00:40, 22 July 2009 (UTC)[reply]

infertility

how infertility occurs? —Preceding unsigned comment added by Shrekul (talkcontribs) 11:24, 18 July 2009 (UTC)[reply]

Infertility#Causes will get you started.83.100.250.79 (talk) 11:40, 18 July 2009 (UTC)[reply]

How far can people shout?

How far can the average human shout? I've checked the Vociferation article and it says the maximum audio levels but I'm not sure how to work out the distance that people could hear them, nor how loud the average person is. 92.233.244.115 (talk) 12:21, 18 July 2009 (UTC)[reply]

I can't find anything about it online but in one of my From Our Own Correspondent it talks of people in (I think) Spain that had a language that they'd shout across mountains to each other. It has be to a number of miles, but the environment will be the biggest factor - plus it depends on whether you count using 'nature' to amplify you or not? After all in some building in London (St Pauls Cathedral? You can whisper along the edge of the wall and it will move round the build and the person on the other side will hear you. ny156uk (talk) 17:00, 18 July 2009 (UTC)[reply]
Infact we have an article Whispering gallery. Nothing to do with 'furthest' but shows how important the environment is to the range. ny156uk (talk) 17:02, 18 July 2009 (UTC)[reply]
From [27] you can see Australia's loudest shout (Done by a crowd) can do 115.8 decibels. When you double the distance you reduce the intensity by 1/4, but since decibels is logorithmic, this works out to be ~6 decibel reduction for each time the distance is doubled. 0 Decibels is considered the threashold of hearing, so the distance can be doubled 115.8/6.02=~19.234. This gives us a distance, in perfect conditions, of 2^19.234=~616,595 meters (since I'd assume the originally measured amount of decibels is at a length of 1 meter, but I couldn't verify this). So we get 616 km or 383 miles. Obviously this could never occur in the real world since you can't hear something of 0 decibels over even the faintest noises of wind. 100 miles may be more reasonable for a real world max. I've seen reports of people shouting across lake Michigan (can't find source now) which is 60 miles across in most areas, but I'm not sure the distance where the guy shouted was. Anythingapplied (talk) 17:48, 18 July 2009 (UTC)[reply]
My somewhat dated (1990) copy of the Guinness Book of World Records, on page 33, says

"The normal intelligible outdoor range of the male human voice in still air is 200 yards. The silbo, the whistled language of the Spanish-speaking Canary Island of La Gomera, is intelligible across the valleys, under ideal conditions, at 5 miles. There is a recorded case, under freak acoustic conditions, of the human voice being detectable at a distance of 10 ½ miles across still water at night. It was said that Mills Darden (see Heaviest Men) could be heard 6 miles away when he bellowed at the top of his voice."

As far as shouting across Lake Michigan, I think you’re thinking of being able to see across Lake Michigan under unusual circumstances, as in the article quoted in this post[28]. (I never thought I’d use the Flat Earth Society as a reliable source, but they’re copying a now-unavailable Holland Sentinel article verbatim.) Red Act (talk) 21:37, 18 July 2009 (UTC)[reply]

There were people whose duty it was to wake up those who were on duty in the Temple in Jerusalem [Mishnah: Shekalim 5:1]. The noise that they made was so loud that King Agrippas said it could be heard as far away as 32 kilometres [Jerusalem Talmud: Shekalim 5:1] or according to another version 12 kilometres away [Babylonian Talmud: Yoma 20b]. Simonschaim (talk) 11:42, 19 July 2009 (UTC)[reply]

Oh, that’s nothing. When Athena was born by emerging fully grown and fully armed from Zeus’ head (after Zeus had swallowed Athena’s pregnant mother), she gave out a shout that echoed throughout the whole world.[29]. That beats the heck out of those Jerusalem shouters! Red Act (talk) 15:11, 19 July 2009 (UTC)[reply]
I don't know about Athena, but from what I gather (WP:OR), Valkyries can shout really loud too... :-) 76.21.37.87 (talk) 00:45, 20 July 2009 (UTC)[reply]

Are these vehicles visible on the moon through Earth-based telescopes? —Preceding unsigned comment added by 76.195.3.170 (talk) 16:41, 18 July 2009 (UTC)[reply]

Nope. -- Aeluwas (talk) 17:53, 18 July 2009 (UTC)[reply]
They're not on any of the recently released LRO images http://www.nasa.gov/mission_pages/LRO/multimedia/lroimages/apollosites.html, but those look to be able to resolve them. I don't know if they were parked so close to the lander that LRO can distinguish them. 87.114.153.140 (talk) 18:13, 18 July 2009 (UTC)[reply]
Maybe they were carjacked and taken to a chop shop. Edison (talk) 19:54, 18 July 2009 (UTC)[reply]
ROFLMAO! X-D 76.21.37.87 (talk) 00:16, 19 July 2009 (UTC)[reply]
The LRO images that were released a few days ago are said to be only about a third of the resolution that the LRO will achieve when it's fully operational - from the look of the present photos - I think that spacecraft will be able to resolve the rovers - but you aren't going to see much - one or two fuzzy pixels maybe. Earth-based telescopes don't have the resolving power to image even the landers - the rovers are quite a bit smaller than the landers. SteveBaker (talk) 23:41, 19 July 2009 (UTC)[reply]

How well known was the gravitation of the moon?

The article gravitation of the Moon states that the acceleration due to gravity near the surface of the moon is about 1.63 m/s^2. When was this determined? Specifically, before the moon landing, had the moon's gravitation been correctly predicted? --bdesham  19:07, 18 July 2009 (UTC)[reply]

Interestingly, you don't need to know much to calculate an approximation: Newton's law of universal gravitation says gravity near a massive object only depends on its mass and the distance from it.
  1. If you know the earth's gravity and diameter, you can calculate its mass.
  2. If you know the mass of the earth and the orbit of the moon, you can calculate the moon's mass: Planetary_motion is also governed by the same gravitational laws.
  3. If you know the moon's mass and diameter, you can calculate the approximate gravity at its surface.
Now this all assumes the moon is approximately homogeneous, which we know is only sorta-approximately true (the article you linked talks about variations and their causes, and the very first sentence talks about mapping from orbit...did we send orbital probes before landing missions?), but if we're talking just a decimal-place or two and "average" "near the surface", seems a reasonable guesstimate. DMacks (talk) 19:34, 18 July 2009 (UTC)[reply]
Please help me out here. You state "If you know the mass of the earth and the orbit of the moon, you can calculate the moon's mass." OK, consider a satellite in a circular orbit 10000 miles above the Earth. What is its mass? Is there more info in "orbit of the moon" than I am seeing? Edison (talk) 19:53, 18 July 2009 (UTC)[reply]
I meant to ask "What is the satellite's mass?" There can be a satellite in orbit at the same height as the International Space Station with 1/1000 of its mass. The orbital height of the satellite tells you nothing about its mass, does it? Edison (talk) 01:28, 19 July 2009 (UTC)[reply]
Knowing the earth's gravity and diameter doesn't get you the earth's mass, it gets you the earth's mass times the universal gravitational constant (which is itself hard to measure). And how does the orbit of the moon allow you to calculate the moon's mass? You can use it to get GMearth, but that's all I can see. Algebraist 20:12, 18 July 2009 (UTC)[reply]
The distinction between the case of an artificial satellite and the moon is one of mass. While for very nearly all intents and purposes an artificial satellite has an utterly negligible mass when compared to the Earth, the moon masses a little bit more than 1% of the mass of Earth. For a circular orbit, the orbital period (T) is related to the orbital radius (r), the gravitational constant (G), and the two bodies' masses (M and m) by the relation
T2 = 4π2r3 / G (M + m)
(See also Standard gravitational parameter.) For artificial satellites, M >>> m, so we usually just drop the mass of the satellite and assume a constant orbital velocity at a given altitude. For the moon, the apparent orbital period will be just a trace shorter than it 'ought' to be under that assumption; the discrepancy can be accounted for by the moon's mass. TenOfAllTrades(talk) 20:26, 18 July 2009 (UTC)[reply]
Of course, I'd forgotten that trick. That gives you the moon's mass in terms of the earth's mass, and you can calibrate the earth's mass with earthbound measurements of G. Algebraist 20:53, 18 July 2009 (UTC)[reply]
Indeed (@Algebraist), "Knowing the earth's gravity and diameter[...]gets you the earth's mass times the universal gravitational constant", which if we assume is a constant we just look up in some well-refereed publication that is reputed as being usable for the types of calculations at hand. If we use the same constant in all calculations, seems like we'll be good for comparison to earth's gravity (the original question) even if the absolute intermediate values (e.g., "mass of earth")aren't as correct. DMacks (talk) 20:42, 18 July 2009 (UTC)[reply]
Actually, I'm fairly sure all of the steps above can be done knowing only the products GM and Gm. For the question being asked I don't think it is necessary to know G or M by itself. One of the reasons G is so hard to measure is that you can only get at it when comparing gravitational forces to non-gravitational ones. Dragons flight (talk) 05:00, 19 July 2009 (UTC)[reply]
Okay, so we can determine GMEarth from surface gravity on Earth and radius of the Earth, both of of which are known very accurately. And in principal we can determine G(MEarth + MMoon) from the period and semi-major axis a of the Moon's orbit. Then we subtract GMEarth fromG(MEarth + MMoon) to find GMMoon. Problem in practice is that GMMoon is only about 1% of GMEarth, so to find GMMoon to an accuracy of, say, 1% you need to know G(MEarth + MMoon) to an accuracy of 0.01%, and so you need to know the semi-major axis a of the Moon's orbit to an accuracy of about 0.003% (because G(MEarth + MMoon) is proportional to a3). In other words you need to know the semi-major axis of the Moon's orbit to within about 10 km. In a pre-spaceflight era, that seems like a big challenge. Gandalf61 (talk) 09:14, 19 July 2009 (UTC)[reply]
Yep, that's a tough measurement, and I can't think of a clever way to get it. (I can't guarantee that they didn't have one, but it seems unlikely.) I did a bit of poking around, and found that Michelson (of Michelson-Morley) and Gale did some experiments around World War I that aimed to directly measure the tidal force of the Moon from the Earth's surface. Morley did some painstaking measurements with some cast-iron pipes and microscopes, and was able to derive an estimate of the Moon's mass: [30][31][32]. The number he came up with was about 30% low, but it was a remarkable achievement given the magnitude of the effect he was measuring.
Another approach would be to try to locate the barycenter (center of mass) of the Earth-Moon system. The ratio of the Earth-barycenter and moon-barycenter distances tells you the ratio of the two bodies' masses. Unfortunately, the wobble that puts into the Earth's orbit is only about 3000 miles/5000 km to each side, and the Earth orbits the barycenter at only about 10 meters per second. I'm not sure if there's any clever way to make those sorts of measurements with pre-spaceflight technology.
Per Dragons flight below, there were of course several missions that orbited or flew by the moon years before there were actual landings; highly precise measures of the moon's mass would have been easily derived from those flights. TenOfAllTrades(talk) 13:58, 19 July 2009 (UTC)[reply]
With Newton's insights into the nature of gravity, the rough radius of the moon, and an assumption that the moon is made of rock, an 18th century scientist could get the right answer within 20% or so. More directly there were several lunar orbital missions before the lander missions, so the gravity would have been well known before any human set foot on the moon. Dragons flight (talk) 13:35, 19 July 2009 (UTC)[reply]

Cute bug

Can anyone recognize the the bug shown [33] here? Photo was taken a couple of days ago in Connecticut. RJFJR (talk) 20:09, 18 July 2009 (UTC)[reply]

It appears to be the larva of the Spicebush Swallowtail, Papilio troilus.CalamusFortis 20:24, 18 July 2009 (UTC)[reply]
(after ec) Yes, Papilio troilus larva it is. --Dr Dima (talk) 20:28, 18 July 2009 (UTC)[reply]

Thank you. RJFJR (talk) 21:14, 18 July 2009 (UTC)[reply]

As a matter of interest, are the caterpillar's markings intended to mimic a snake's head/face? --Kurt Shaped Box (talk) 00:15, 19 July 2009 (UTC)[reply]
Yes, most likely a head of a predator. It doesn't really matter whether it looks more like a snake, a tree-frog, or a lizard; it reduces a chance of being attacked and eaten all the same. Big eyes + big mouth = danger = do not approach. --Dr Dima (talk) 00:48, 19 July 2009 (UTC)[reply]
TBH, I'd be wary to approach that if I saw it in my garden without knowing what it was. --Kurt Shaped Box (talk) 02:05, 19 July 2009 (UTC)[reply]
Some research was performed on the larval markings of this species, and it was determined that the false eyes deter predators because, no matter what angle they are viewed at, like the eyes of the Mona Lisa, they appear to follow you.CalamusFortis 03:08, 19 July 2009 (UTC)[reply]

WQ Test

What kind of questions would be asked on a multiple-choice Wisdom Quotient (WQ) Test? - GlowWorm. —Preceding unsigned comment added by 174.130.249.97 (talk) 22:21, 18 July 2009 (UTC)[reply]

Wisdom might be a place to start.71.236.26.74 (talk) 02:47, 19 July 2009 (UTC)[reply]
Given that the concept of "wisdom" is even more nebulously defined than the concept of "intelligence", and Intelligence Quotient tests are widely regarded as very suspect indicators, I'm not sure what would be gained from such a thing. --98.217.14.211 (talk) 15:48, 19 July 2009 (UTC)[reply]

July 19

Who was the first man to masturbate in space?

Strange question, I know - but this came up (no pun intended) during a discussion of the mechanics sex in space with some cow-workers today.

Presumably, the guys on long-term space station missions have jerked off in space (probably though boredom as much as anything else) - but is it on record as to who the first guy to 'boldly cum where no man has cummed before' actually was? I'd be very surprised if NASA or the Russians hadn't asked one of their spacemen to try having a wank during a mission to see if it worked up there.

I have a feeling that this has been asked before on here but I can't find anything in the archives. --Kurt Shaped Box (talk) 02:12, 19 July 2009 (UTC)[reply]

What moo-ved your "cow-workers" to consider this question? Edison (talk) 02:46, 19 July 2009 (UTC)[reply]
FWIW, there was an article about the moon landings in a newspaper that someone was reading. Someone else glanced at it, frowned and asked "I wonder if anyone's ever had sex in space?". Just one of those random things, I guess. Once the talking point had been raised (again, no pun intended), it went on from there... --Kurt Shaped Box (talk) 00:48, 20 July 2009 (UTC)[reply]
This is slightly off-topic, but why would ejaculation in weightlessness be any different than on Earth? I mean, the "internal ballistics" should be entirely the same - the neurons fire, the muscle contracts, the cosmonaut comes. There is no "gravity assist" involved :) . As for, erm, who came first - I really don't know. Sorry about that. --Dr Dima (talk) 03:02, 19 July 2009 (UTC)[reply]
There is the school of thought, is there not, that it is impossible to get/maintain an erection in zero gravity? --Kurt Shaped Box (talk) 00:49, 20 July 2009 (UTC)[reply]
Why would it be impossible? And how would you know -- have you actually tried this in zero gravity? :-D 76.21.37.87 (talk) 01:32, 20 July 2009 (UTC)[reply]
I'm not claiming that it's impossible! ;) --Kurt Shaped Box (talk) 02:38, 20 July 2009 (UTC)[reply]
These [34] [35] suggest, as I personally expected, if it has happened we don't know who or when. Nil Einne (talk) 03:42, 19 July 2009 (UTC)[reply]

Is it too much to ask for a little decorum here? Phrases like 'boldly cum' chafe my sense of decency, and I don't mind telling you. Vranak (talk) 04:41, 19 July 2009 (UTC)[reply]

It's a fair enough point. My apologies if you were offended by my choice of words. --Kurt Shaped Box (talk) 00:48, 20 July 2009 (UTC)[reply]

While you're at it, I'd like to know who was the first woman to masturbate in space. —Preceding unsigned comment added by 82.43.91.128 (talk) 08:10, 19 July 2009 (UTC)[reply]

Mostly likely, anyone who has masturbated in space (and I admit it's not unlikely) did so in a private, unmonitored location and does not discuss it. That said, considering that zero gravity has been simulated for films by using a jet in free fall, it is possible that someone has or could produce public pornography in a zero-gravity environment - but this would be extremely expensive. Dcoetzee 08:16, 19 July 2009 (UTC)[reply]
It's been done, albeit on a limited scale. The Uranus Experiment (safe-for-work description) features a brief scene that was filmed aboard a Russian parabolic flight. TenOfAllTrades(talk) 13:24, 19 July 2009 (UTC)[reply]
It would take a brave couple to try to perform pornographic acts on the Vomit Comet. However, there are people who have been filmed having sex while skydiving, so I guess anything is possible. Looie496 (talk) 16:50, 19 July 2009 (UTC)[reply]
A plaque left on the Moon by the first two Apollo astronauts to spend time there said "We came in peace for all mankind." I never before considered whether it might have been literally true. Edison (talk) 00:08, 20 July 2009 (UTC)[reply]
Just to link to it, we do have a sex in space article. Nothing on solitary sex in space, alas. Having seen at the National Air and Space Museum the monstrosities that are space toilets, I can hardly think it would be a very, err, convenient place to do such a thing. --98.217.14.211 (talk) 01:14, 20 July 2009 (UTC)[reply]

Bioavailability of calcium

Is it true that calcium in dairy milk is less bioavailable to humans than calcium in leafy greens? NeonMerlin 03:16, 19 July 2009 (UTC)[reply]

No, it's the other way round. See Phytic acid. 95.112.134.175 (talk) 07:32, 19 July 2009 (UTC)[reply]
Huh? According to our article, at least, phytic acid is found in the hulls of nuts, seeds and grains. And all of the listed sources of phytic acid are grains, legumes, or seeds. And this source referenced by the phytic acid article gives lettuce and spinach as examples of foods that are devoid of phytic acid. The question was about milk vs. leafy greens, and neither one of those contains phytic acid. Red Act (talk) 08:21, 19 July 2009 (UTC)[reply]
Sorry I only scanned the article very briefly. I was told by a doctor that phytic acid was found in lettuce. 95.112.134.175 (talk) 08:34, 19 July 2009 (UTC)[reply]
This article says “even when it comes to calcium, the complexes [(salts)] found in leafy green vegetables such as bok choy are absorbed more readily than those in dairy foods.” This yogurt manufacturer, not surprisingly, has a contradictory point of view. This looks like it may be one of those questions where it can be difficult to get a clear-cut, unbiased answer. To get a good answer, it will probably be necessary to stick purely to research articles, and even then check to see who’s funding the research, as it apparently is a contentious issue. Red Act (talk) 09:14, 19 July 2009 (UTC)[reply]
I don’t want to pay to see the articles I find from Google Scholar, so I’m basing a lot of this post on the little snippets I can see in the search results. But it looks like calcium absorption from most leafy greens (e.g. kale, collards, bok choy, turnip greens, mustard greens, basically anything from the brassica family) is considerably better (40-59%) than calcium absorption from milk (32%). Spinach and watercress, however, are exceptions, since most of the calcium in those is bound up as calcium oxalate. The yogurt manufacturer above, not surprisingly, only lists calcium absorption rates from spinach and watercress, and completely ignores calcium absorption from all those non-oxalate leafy greens. Red Act (talk) 10:01, 19 July 2009 (UTC)[reply]
Bizarrely, our calcium in biology article doesn’t list the calcium content of a single leafy green, even though calcium absorption from most leafy greens is better than from milk, and the article lists lots of foods with far less calcium per serving than leafy greens, according to the reference the article cites. E.g., according to the reference, a cup of cooked collards has more calcium than a cup of skim milk (357g vs. 306g)! I guess that article’s editors have an anti-leafy-green bias. Red Act (talk) 11:34, 19 July 2009 (UTC)[reply]
The article doesn't list how much calcium there is in milk either (only milk powder, cheese and human milk) so I guess you could say it also has a milk bias Nil Einne (talk) 14:57, 19 July 2009 (UTC)[reply]
Yeah, that whole section needs some serious work. Horse meat? I’ve started working a little on a more carefully chosen list, but it’s tempting to just delete that section, since that haphazardly chosen list isn’t a whole lot more valuable than just a link to the very thorough reference. And that list takes up an unpleasantly large fraction of what is otherwise a reasonably well-written article. Red Act (talk) 15:29, 19 July 2009 (UTC)[reply]
Not only has that list never even listed regular milk, the list originally contained “vaccine”! Clearly there’s some kind of pro-vaccine, pro-horse-meat bias going on! Red Act (talk) 16:16, 19 July 2009 (UTC)[reply]
Horse meat? I think the author must've been a Cossack.  :-) 76.21.37.87 (talk) 00:41, 20 July 2009 (UTC)[reply]
I believe the issue with milk and calcium is that there isn't as much magnesium in milk as is required for healthy human functioning. You can absorb all the calcium you want from food but without enough magnesium it doesn't profit you. Vranak (talk) 19:39, 19 July 2009 (UTC)[reply]
One obvious consideration is the quantities a person is likely to consume daily. I can easily imagine someone drinking 250g of milk a day, but I suspect far fewer people take in 250g of bok choy (or whatever), even if it may be healthier in a number of ways if they do. In other words, regardless of the concentration and bioavailability, it doesn't mean advocating something else as the best source of calcium in the real world is a good idea. Depending on why you want to know this may or may not be relevant Nil Einne (talk) 20:25, 19 July 2009 (UTC)[reply]
My statements above that that list is “biased” were mostly in jest. That list isn’t really “biased”, so much as just being very poorly chosen. However, this statement seriously is showing an unintentional cultural bias.
In most parts of the world, most people don’t generally drink milk as an adult, in large part because they lack the mutation on chromosome 2, that’s present in most people of northern European descent, that makes a person lactose tolerant. For example, in China, despite recent growth in milk consumption due to things like “school milk” plans, the per-capita daily consumption of milk only comes to about 37g. (Take the 17.5 billion liters here, and divide by the 1.3 billion people in China.) Furthermore, that 37 g/day is presumably disproportionately consumed by young children, who have not yet lost their ability to produce lactase. Bok choy, meanwhile, is a popular dish in China. So it’s quite likely that in China (which includes more people than all of Europe and North America combined), more adults consume 250g of bok choy on any given day than drink 250g of milk. Red Act (talk) 01:55, 20 July 2009 (UTC)[reply]
I'm not personally convinced that many people consume 250g of bok choy in China. There are a lot of different vegetables in China and while bok choy may be popular it's not the only one (250g would be say 150g for dinner and 100g for lunch which is a fair amount). Although it depends whether you include stuff such as choy sum etc too I guess, I personally don't. They may not consume 250g of milk either but that doesn't mean they consume 250g of bok choy. On the other hand you're right that they probably get a greater percentage of their calcium from sources other then milk, particularly vegetable source and I apologise for neglecting to mention this. Remember however that the 37 g/day if 'disproportionately consumed by young children' is not going to be 37 g/day if the ratios are disproportionate. It could easily be 100 g/day for the the children but 10 g/day or less average for the adults. And remember that adequete calcium is particularly important at that stage of life. But still it's fair to say milk is unlikely to be the primary source of calcium. Ultimately though, my main point it still accurate. The bioavailability issue is somewhat irrelevant when it comes to public health consideration IMHO. If for genetic and cultural reasons milk is a significant source of calcium then the fact that calcium in bok choy is more bioavailable in bok choy (or whatever else) is probably irrelevant. Similarly if milk consumption is low then advocating it as the best source of calcium is not necessarily the best idea and the bioavailability is still largely irrelevant.
On the more general point of lactose intolerance, it's actually an interesting issue. For example our article says "Some studies have found that most Japanese can consume 200 ml (8 fl oz) of milk without severe symptoms (Swagerty et al., 2002)" and "Lactase persistence, allowing lactose digestion to continue into adulthood, is a dominant allele, making lactose intolerance a recessive genetic trait." and "For healthy individuals with secondary lactose intolerance, it may be possible to train bacteria in the large intestine to break down lactose more effectively[57] by consuming small quantities of dairy products several times a day over a couple of weeks" and "Some studies indicate that environmental factors (more specifically, the consumption of lactose) may "play a more important role than genetic factors in the etio-pathogenesis of milk intolerance",[11] but some other publications suggest that lactase production does not seem to be induced by dairy/lactose consumption.[58]". In other words, it's a lot more complicated then something like '90% of Chinese can't drink any milk'. I don't know about in China (although I believe there is great growth in dairy consumption, e.g. [36] and [37] albeit this has probably been very negatively affected by the melamine scandal), but while growing up in Malaysia I found there wasn't actually much discussion about lactose intolerance despite the fact it's obviously must be fairly prelevant (and proven by [38]). Milk consumption is rather low compared to many Western countries but there is advertising and an increasing consumption, and milk is sold in most supermarkets and many convenience stores and also sometimes in secondary schools. Most doctors will I presume quickly diagnose any lactose intolerance related problems and there is likely general awareness but it's not actually some that gets great consideration. However cultured milk drinks are very common and probably one of the key reasons for an increase in milk consumption and these would I presume generally have significantly lower lactase.
Nil Einne (talk) 05:49, 20 July 2009 (UTC)[reply]
I’m surprised you think that there wouldn’t be too many people in China who would eat 250g of bok choy in one day. 250g doesn’t seem like all that much bok choy to me. I happen to have some baby bok choy in my fridge right now. It’s just 4 of those little heads, they weigh 278g, and I will almost certainly eat them all in one meal. In fact, pretty much every day I eat a bowl of vegetables more than twice that mass (567g) in one sitting. I guess it’s a matter of perspective; I’m 193cm tall, and I have a different idea of what a “lot” of food is than a smaller person would.
Just to clarify something, it looks like you might possibly be thinking that I thought that there would be a lot of people in China who eat an average of 250g of bok choy per day. Clearly, Chinese cuisine is so wonderfully broad that it’s hard to imagine too many people choosing to narrow their diet to bok choy to that extent. My point was just about the events on any given day, not about an average daily bok choy consumption rate. E.g., my presumption is that the number of people in China who eat 250g of bok choy on July 20, 2009 will be greater than the number of people in China who drink 250g of milk on July 20, 2009. Red Act (talk) 08:09, 20 July 2009 (UTC)[reply]
"However cultured milk drinks are very common and probably one of the key reasons for an increase in milk consumption and these would I presume generally have significantly lower lactase." -- Another factor to consider is that they contain bacteria that digest lactose and so, once consumed, will actually help the person digest lactose and thus overcome lactose intolerance. FWiW 76.21.37.87 (talk) 03:57, 21 July 2009 (UTC)[reply]

Reflecting light vs color

I'm shining a green laser onto flat surfaces. I know from experience that the light that reflects back into the eye is a lot dimmer on a black or dark surface. What color would reflect the green laser the best? White seems like a likely answer but what about a green surface of the same hue of the laser? -- penubag  (talk) 04:50, 19 July 2009 (UTC)[reply]

If you want a very non-quantitative, crude, rule-of-thumb answer, then white would be your best bet. White reflects all or most of the visible spectrum quite well, so you have little (however, still nonzero) chance to hit an absorption spectral line with your laser wavelength. You are right assuming that green surface reflects green light better than red or blue surface; but "better" does not mean "well", and green is not a single wavelength. So I'd choose white over green if I were you. Quantitatively, on the other hand, your question is unanswerable as posed. The problem is, we only have three types of color receptors (cones) in our eye, but infinitely many distinct wavelengths in the visible part of EM spectrum. Thus, infinitely many different combination of wavelengths produce the exact same color percept in a normal healthy human. In other words, many different surfaces with different absorption spectra will look the same shade of green under the same light source. And this tells you very little about the reflection coefficient at the precise wavelengths that your laser emits. Hope this helps. --Dr Dima (talk) 05:17, 19 July 2009 (UTC)[reply]
IMPORTANT: please NEVER experiment with reflecting a laser into your eye. A reflected laser light can still damage the retina very efficiently and permanently, and some of the damage does not become apparent until much later. Just don't do it. --Dr Dima (talk) 05:23, 19 July 2009 (UTC)[reply]
Thank you very much for that answer, you explained it very thoroughly.
Is reflected light from a laser onto sheet of paper dangerous? My particular laser is a Class 3B. From the Laser_safety article, it says, "A Class 3B laser is hazardous if the eye is exposed directly, but diffuse reflections such as from paper or other matte surfaces are not harmful." My friend's laser is 10x more powerful and can pop balloons and burn skin under exposure but I never thought anything of it. -- penubag  (talk) 05:48, 19 July 2009 (UTC)[reply]
You are welcome. And, regarding laser safety, Class 3B can definitely cause permanent eye damage. Reflection from a surface is not necessarily much less dangerous than the direct exposure, for two reasons. First, even when the surface looks matte, it is hard to judge by eye how much light is reflected into any direction; a significant portion of the beam may still undergo specular reflection, depending on the local surface conditions. Second, it is impossible to control all reflections; not even in the lab, and definitely not at home. In any home there always are reflective surfaces around (mirrors, windows, cups, jars, polished metal handles, your wrist-watch, etc.) and, inevitably, laser will be reflected off those surfaces and into your eyes every once in a while. So doing this kind of home experiments with a Class 3B laser is not recommended to say the least. However, it may be a very good idea for a science project, provided (1) a proper eye protection is used whenever required, (2) laser beam travels in a horizontal plane that is substantially lower than the eye-level of the shortest of the participants, and (3) no-one EVER lowers his head to the level of that plane when the power is or may be on. The reflected light level can - and should - be measured with a photodiode or a similar device, and NEVER with an eye! By the way, using a photodiode you can perform the measurements with and without an Integrating sphere to measure total reflection and specular reflection, respectively. --Dr Dima (talk) 07:18, 19 July 2009 (UTC)[reply]
Maybe the laser saftey article should be written -- penubag  (talk) 21:18, 21 July 2009 (UTC)[reply]
It exists! Laser safety (note spelling). 70.90.174.101 (talk) 05:57, 22 July 2009 (UTC)[reply]

The color of a surface is a direct indication of the colors it will reflect best. A leaf is green because it's absorbing the other colors. Your light would be best-reflected by a material that's exactly the same color as the laser. Of course there are other factors, such as the smoothness of the surface (a rough surface would scatter light in many directions). IIRC, few materials will reflect 100%. It's not an easy question to answer, and it can't be answered completely without intense quantum calculations, if at all. Twang (talk) 22:39, 19 July 2009 (UTC)[reply]

White is probably better than a surface the same color as the laser - mainly because it's exceedingly difficult to make a material that has that exact color - where there are plenty of naturally occurring materials that are very bright white. The best answer is a surface that's a really good mirror - which has no "color" (in the conventional sense) at all...but it somewhat depends on what you need the answer for. SteveBaker (talk) 23:14, 19 July 2009 (UTC)[reply]
I know, but I said what if the exact color could be attained. -- penubag  (talk) 21:18, 21 July 2009 (UTC)[reply]

firefly

Recently while walking by a graveyard a friend noted that the fireflys looked like souls rising from the graves. It did look like they moved in a generally upwards direction. Tonight I observed the flight paths of fireflys around dusk at a cook out in northern Virginia. As I watched I noticed that in general they would light up as they moved upward at least nine out of ten times. It seemed that their flight paths would involve an intentional drop in elevation so that they could rise from a near standard elevation at each lighting. To be more exact they would hover fly laterally to a point in space dip at an angle, hover, than light as they moved up ending well above their flash point and sometimes hovering before moving laterally to a new location. I don't remember fireflys doing this when I was a child in Maine; in fact I remember many fireflys lighting up as they moved in lateral directions looking like zig zags. But that was a few years back and my memory may be bad. I was wondering if anyone knew anything about firefly behavior and could explain the correlation between moving up and lighting up?--OMCV (talk) 04:57, 19 July 2009 (UTC)[reply]

The most common firefly species in eastern North America is the Big Dipper (Photinus pyralis) [39], which flies in a J-shaped trajectory, lighting on the upswing, making it look like they're all flying upwards. Acroterion (talk) 15:27, 19 July 2009 (UTC)[reply]
Link to the Washington Post article on why this is a good year for fireflies (lightning bugs in the mid-Atlantic) [40] and a graphic that describes several species' patterns [41]. Presumably, the Big Dipper wasn't common in Maine and a species with a different pattern prevailed. Acroterion (talk) 15:42, 19 July 2009 (UTC)[reply]
Sorry I don't have a scientific explanation, but this reminds me of Manichaeism#Cosmogony. In some other article that I can't find now I have read that (according to this believe) the light is collected by the moon and transfered to the sun. A more darwinistic explanation (well, it's a guess) is that when trying to find a mating partner many kinds of animals try to show vitality by doing costly things like buying a luxury car (on credit) of flying steeply upwards. It's only natural that they shine a light on the ups and not on the downs. 95.112.134.175 (talk) 15:29, 19 July 2009 (UTC)[reply]
Each species exhibits both distinct flight patterns as well as blinking patterns. Only the males fly -- the female climb stalks and trees and blink to the males to attract them for mating, or lunch. It has been documented (from a Tuesday Science section, that's all I can remember) that females sometimes blink the wrong way to catch males from other species for lunch. DRosenbach (Talk | Contribs) 19:54, 19 July 2009 (UTC)[reply]
How do you know they "looked like souls" - you know what a "soul" looks like? SteveBaker (talk) 23:09, 19 July 2009 (UTC)[reply]
I'll have to ask my friends how she knows what souls look like Steve. Until then thanks for all the info and links every one, much appreciated.--OMCV (talk) 01:52, 20 July 2009 (UTC)[reply]
In his magisterial Introduction to the History of Science, George Sarton, in the course of recording his amazement that Dante appears to have been the first imaginative writer in European history to mention fireflies (Vol. 3, Part 1, pp. 487–88), remarks that "one would expect them to impress any poetical and imaginative mind". He goes on to recount an exchange he had with D'Arcy Wentworth Thompson on the subject, in which the latter suggested that there may have been, in ancient times, a superstitious aversion to mentioning both butterflies and fireflies, they both having been viewed as dead men's souls. (A butterfly was ψυχή, psyche, in ancient Greek.)
One of my own favorite "poetical and imaginative" lightning-bug passages is in Vladimir Nabokov's Ada:
The males of the firefly, a small luminous beetle, more like a wandering star than a winged insect, appeared on the first warm black nights of Ardis, one by one, here and there, then in a ghostly multitude, dwindling again to a few individuals as their quest came to its natural end. Van watched them with the same pleasurable awe he had experienced as a child, when, lost in the purple crepuscule of an Italian hotel garden, in an alley of cypresses, he supposed they were golden ghouls or the passing fancies of the garden.
Deor (talk) 03:23, 20 July 2009 (UTC)[reply]

Needles on the skin

What is the effect of sticking a needle on my skin? Will I release any hormone? --Mr.K. (talk) 10:58, 19 July 2009 (UTC)[reply]

Possibly Endorphins. -- Aeluwas (talk) 12:20, 19 July 2009 (UTC)[reply]
Depending upon the damage you do, you may also trigger Hemostasis (including Coagulation) and the release of the chemicals involved. Shower of Jagged Steel (talk) 14:32, 19 July 2009 (UTC)[reply]
Well, one possible effect is that unless you are very careful about sterility you can get a really nasty infection -- puncture wounds are the worst kind. Looie496 (talk) 16:54, 19 July 2009 (UTC)[reply]
They are? Why? Vimescarrot (talk) 22:07, 19 July 2009 (UTC)[reply]
For superficial wounds, even if they look nasty, bacteria usually don't thrive all that well, and it is relatively easy for the body to flush any contaminants to the surface. Puncture wounds, if they get infected, tend to form abcesses. If an abcess gets large, the immune system has difficulty keeping bacteria in it from growing, which can ultimately result in sepsis, which is generally fatal if not very aggressively treated. Looie496 (talk) 22:28, 19 July 2009 (UTC)[reply]
As an aside, is it the same with gunshot / shrapnel wounds? (I'm pretty sure that those tend to get infected too.) 76.21.37.87 (talk) 00:34, 20 July 2009 (UTC)[reply]
Bullets and shrapnel tend to be pretty clean, so the only bacteria that get into a wound tend to come from the clothing or body surface, but even so, yes, those sorts of wounds can easily produce life-threatening infections. Looie496 (talk) 00:44, 20 July 2009 (UTC)[reply]
The above is why animals which fight with domestic cats (other cats, mostly) get a lot of abscesses: the cat's teeth are like needles. --Sean 14:58, 20 July 2009 (UTC)[reply]

What's the resolution of our eyes?

Is there anyway to measure the resolution of the human eye in the same way we do for a TV? --Cameron Scott (talk) 11:33, 19 July 2009 (UTC)[reply]

Eye#Acuity has information relevant to your question. Dauto (talk) 12:06, 19 July 2009 (UTC)[reply]
Better to compare the resolution of the human eye to some camera or eyesight of other animals than to a display. Edison (talk) 15:21, 19 July 2009 (UTC)[reply]
The reason it's hard to find numbers for this is because there really isn't any region with constant resolution for the eye. The highest resolution occurs at a point at the center of the fovea, and the resolution drops off pretty sharply as you move away from that point. Looie496 (talk) 16:38, 19 July 2009 (UTC)[reply]
Also, there is a trade between speed of aquisition and resolution. When we stare at something, our eyes vibrate in order to sample the image at positions between the "pixels" produced by the rods and cones of our eyes. We also see monochrome information (brightness) at about four times the linear resolution at which we can see colors. Hence you hear numbers in the 2000 to 8000 pixel region - with 4000 probably being about right. SteveBaker (talk) 23:06, 19 July 2009 (UTC)[reply]
The foveal resolutiojn is the relevant number. We automatically look at and area of interest or saliency.I seriously doubt the assertion of "eyes vibrating to sample positions between pixels." Edison (talk) 00:06, 20 July 2009 (UTC)[reply]
Hmm. The two facts Steve mentioned are correct, but it's unclear whether they're causally related. It is definitely true that our eyes vibrate, or at least jiggle, constantly, in what are called microsaccades. It's also a fact that there are numerous tasks where we are capable of detecting spatial structure considerably finer than our smallest receptive fields -- this phenomenon is called hyperacuity. But as far as I know it hasn't been established that microsaccades are involved in hyperacuity -- unless I've missed something, which is always possible. Looie496 (talk) 05:46, 20 July 2009 (UTC)[reply]
Well, correlation isn't causation - but I'm sure of one part of this: if the muscles that perform the microsaccades are slowed down (eg because your eyes are tired or because you've recently been aneasthetised or something) then your vision is blurry. Also, it's hard to imagine any other means by which we could have visual acuity that's better than the underlying resolution of the rod/cone cells - and it's exceptionally difficult to imagine any other reason why we'd perform these microsaccades. FWIW: I've actually built a high-resolution camera by taking many pictures with a rapidly vibrating still-frame digital camera and over-sampling and averaging the in-jittered pictures together - the results can be pretty impressive. SteveBaker (talk) 02:33, 21 July 2009 (UTC)[reply]
Super-resolution is a common image processing technique. Vibrations, or dithering, are a class of methods that assist in sub-sample resolution. This is a way to "beat the Nyquist rate" - but what is actually happening is that time-domain signal is being transformed into spatial-domain signal (no physical laws are violated, of course). I would not be surprised at all if there were physiological, biological/psychological versions of these techniques. They are certainly very common in software and hardware signal processing. Nimur (talk) 00:52, 22 July 2009 (UTC)[reply]

Long-term storage of water

If one was to fill a 20 litre container with water for long-term storage for use in an emergency which rendered tap water unavailable or unsuitable, would there be a noteworthy risk of something growing in it? Would it be advisable to change the water contained periodically? Would it be advisable to add anything to the water prior to storage and/or immediately before use? ----Seans Potato Business 19:02, 19 July 2009 (UTC)[reply]

Try it. Experiment. Smell the water. Taste the water. The ultimate arbiter of whether something is good for you or not is your own body. Vranak (talk) 19:37, 19 July 2009 (UTC)[reply]
Well it's a bit problematic if there's a major disaster and you find then the water is not "good for you". Anyway without trying to provide a definite answer, I would consider boiling the water before use, if that's practical. In most cases, energy may be limited so it may not be a good idea to use what you have for boiling water when you can avoid it. You may want to keep some water sterilisation/purification tablets in your emergencies supplies if you can't boil the water. If your household water is chlorinated then if you get fresh water and close and keep the lid tight that may help. Don't use water that has been sitting exposed to the air. Chlorinating the water further may help, but you'd need further research to find an appropriate level so you don't end up with such a high level that if you need it in 1 year you'll have to wait 1 week before you can drink it. P.S. I came across this [42] which may be of interest but I'd look thorough the sources thoroughly before I trust it Nil Einne (talk) 20:13, 19 July 2009 (UTC)[reply]
Why a 20 liter container? I find that many juices come in smaller plastic bottles. They are probably good for storage of water for drinking purposes. They might prove more easily transportable. Storage space taken up might be comparable. If some get spoiled for some reason, others might be still potable (and not to mention portable). Bus stop (talk) 20:23, 19 July 2009 (UTC)[reply]
If you're concerned about something like a hurricane or earthquake, I think filling a few 5-gallon (= 20 liters) containers with water is a very reasonable idea. As long as they're tightly sealed, I don't see why you would have to change the water more than once a year or so -- nothing is going to grow in an airtight container of pure water. Looie496 (talk) 20:48, 19 July 2009 (UTC)[reply]
A google search for "emergency water storage" yields several sites with some good information. I would tend to trust official sites more than the various ask-me sites, since the latter may have been written by non-experts. The FEMA site for example may be a good starting point. Key points include periodic rotation or replacement of your emergency water supply and proper preparation of water storage containers (such as sanitizing containers with bleach). If you are reusing containers, soda bottles are recommended over fruit juice or milk containers because of difficult-to-remove residual milk protein and fruit sugars in the latter. -- Tcncv (talk) 21:01, 19 July 2009 (UTC)[reply]
Ignore the "taste it" guy, above; you can't taste cryptosporidium or any number of other waterbourne pathogens, and getting that wrong will kill you. If you must store the water in a large vessel, then you should count on using purification tablets to make sure it's safe. Even if the water is good when you open the vessel, over the days and weeks following that, when you're returning to it (with hands you haven't washed properly) you'll inevitably contaminate it long before it's finished. Don't boil water - what little fuel you have should be used to keep you warm and warming food; boiling every drop you drink will multiply your fuel needs greatly. Rather than a home-filled demijohn, I'd seriously consider stockpiling smaller supermarket-bought bottled water (the cheapest kind, obviously; here in the UK 2 litres costs 13p) which guarantees it's safe and uncontaminated (and being in smaller containers means you don't have all your eggs in one basket). Plus those modest-sized empty bottles have a bunch of re-uses, from storing captured rainwater to being a kind of primitive fuel. -- 87.113.21.118 (talk) 22:11, 19 July 2009 (UTC)[reply]
Are you familiar with how rats eat? I imagine not. They take a little bite to eat, wait a while, and if they don't feel bad they go back for more. This approach works well enough for humans as well. Vranak (talk) 01:48, 21 July 2009 (UTC)[reply]

Yes, it's a good idea to change it now and then (say, yearly). And you *might* want to add something to it to discourage anything that might grow in it. You can use tincture of iodine or household bleach for that. Some more advice here and here. Twang (talk) 22:45, 19 July 2009 (UTC)[reply]

Original research: If you fill clean glass jugs with tapwater, in a few months they may have green things growing in them. This is not to say either that you might not drink it is thirsty enough, or that you would not get ill if you did. Bottled purified water would go far longer and still not have visible things growing in it. I expect that half gallons of water could be "canned" as could fruit juice, by canning processes in the Ball Blue Book or per recommendations of State Departments of Agriculture in the U.S., and be consumed long afterwards. So: ordinary tap water? Very short shelf life, measured in days or weeks at most. Supermarket bottled water in large containers is pretty cheap, and storing a supply for a week or two drinking needs will not bankrupt most people. Shelf life should be a year or so. If you turn off the main water intake, to prevent introduction of contaminated water if the water plant goes off line, and turn off the gas to the water heater, to avoid explosion if the water level drops too low, you have an emergency water supply of 40 gallons (160 liters or so) in many homes. You should be able to drink this and survive for quite a long time, acknowledging that the little green fuzzies will grow in it as well eventually. Edison (talk) 00:02, 20 July 2009 (UTC)[reply]
Why would you trust "Supermarket bottled water" but not tap water? Most cheap brands (and some expensive ones) of bottled water are literally tap water, often not even filtered. APL (talk) 02:39, 20 July 2009 (UTC)[reply]
Personal OR here. I've stored tap water for extended periods in large plastic containers Malaysia and New Zealand and haven't gotten green stuff growing in it. I also tend to end up with a lot of water bottles in my room and if I happen to drink one after several months the water tastes odd but I haven't yet gotten sick (not that it's a good indicator). However this is not fresh water but rather water that I've been drinking so is probably fairly contaminated. However the water doesn't tend to have green stuff growing in it. So I'm not really sure what's up with Edison's water supply. (The water supply in Malaysia is sometimes a bit dirty, probably due to poor piping but is usually chlorinated.) Also I suspect the problem is partially as a result of unclean bottles as the bottles tend to keep the taste after you wash them. And you tend to get the same problem if you keep reusing a bottle for several months. As someone mentioned above, this is a big issue if you reuse fruit juice containers, cleaning them out properly. In more general terms, again complete OR here, re-filling a 20L bottle is likely to be a lot easier then ~7 3 L fruit juice bottles. If the refilling difficult causes issues then this is an important considering. If you keep more then 20L, e.g. 40L - 60L then ~13 or 20 of these 3 L containers it likely becomes an even bigger issue. If you would refill a 20L container every 6 months but the 3 L every 2 years, I would suspect the 20L is better. Indeed I would stay the same about pre-bottled water. In terms of durability etc, while you do have redundancy I would expect there's a good chance most of the bottles will go off so I don't see it as necessarily a big benefit. Also as anyone who's ever used one of those large containers would know, and as may be obvious, they are quite strong things. I would expect them to be much more puncture resistant, and also resistant to things falling on them. If you store the 3L containers in durable box this may make up for that of course. This may be significant since if it's e.g. an earthquake it's easy to imagine things falling on the water bottles. On the other hand, you could store the smaller bottles in different locations. I'm not sure about resistance when dropped, the large containers obviously are stronger but also a lot heavier when full. You could try dropping them both from a defined height when full (it may be easier to fill up the 20L after you've taken up to this height) but please take very careful care about where you do this, you could easily kill someone or cause major property damage. However this may not be such a big issue, you shouldn't be storing your bottles that high up anyway. One final issue, if a water tanker is sent during this emergency, it'll be a lot harder to fill up the 3 L bottles then the 20 L bottles on the other hand, it'll be harder to fill up the 20L bottles from say a stream (but you could use a small bottle or something to help). Personally I would suggest something like e.g. one 20L and perhaps a few smaller bottles intended for the storage of water may be best Nil Einne (talk) 06:25, 20 July 2009 (UTC)[reply]
How long is this 20L going to last you? for how many people? how long do you expect the emergency to continue? in total breakdown of piped supplies but with civil order (eg pumping station failure), they'd start bringing in tankers in under a day (and shops would be open to buy bottled water). With disorder (eg natural or nuclear disaster) 20 litres won't last you long enough to get out of there, nor will you find clean supplies in a reasonable distance. For some unspecified short-term (2 or 3 day) shortage, the contents of the hot water heater would be the best, though it might not taste very good because of deposits in the tank. - KoolerStill (talk) 09:47, 20 July 2009 (UTC)[reply]
OR here, but I used to live in an area where the water was technically safe to drink, but was a dark gray from mineral deposits so we would bring in drinking water in 1 gal milk jugs from someone we knew who had a well (water was not chlorinated). This water never grew any green things although we would only store it for 1-2 months at a time. Googlemeister (talk) 13:36, 20 July 2009 (UTC)[reply]
The recommended daily intake of water for an adult male is 3.7 liters...more than that in a hot climate. So if all you do is drink the stuff, 20l will last you about a week if you're alone - much less time if you are a family. You may be able to economise a little in an emergency - but it's not going to last a month no matter what. Of course a typical water consumption rate in the USA is 50l per person per day! However, this fact points the way to maintaining your 20l emergency supply. Simply install a 20l tank between the water main and your home let your household water flow in at the top and out at the bottom - so it's continually replenished. The water in that tank will be replaced 'automatically' in just a few hours of typical domestic use - so it won't have time to turn nasty until after whatever catastropy you are worried about comes to pass. At that point, you'll have 20l of completely fresh water in your tank. It's not going to turn bad in a week - and that's probably about as long as it's going to last you. SteveBaker (talk) 02:24, 21 July 2009 (UTC)[reply]
How is the proposed tank better than my present water heater, which holds about 40 gallons or 150 liters, which should have a longer "shelf life" since it has been previously heated to 130 degrees F or 54 C? In a pinch the toilet tank is a good short term source of pure water. Older ones hold about 4 gallons, newer ones considerably less. Just remember not to flush! Edison (talk) 03:44, 21 July 2009 (UTC)[reply]
If you have foreknowledge of the coming disaster (say a hurricane) people will fill their bathtubs to have a short term supply of available water. Seems to me like it would not be easy to keep it from getting contaminated though. Googlemeister (talk) 13:40, 21 July 2009 (UTC)[reply]
I agree that in an emergency, the hot water tank, toilet cisterns, swimming pools and (when possible) pre-filled sinks and bathtubs are a better source of drinking water...which is why most people don't go putting 20 liter tanks out in their backyards. I guess it all depends on how long this hypothetical disaster lasts. If it's a planetary scale problem - you'd better find a nice lake or have a decent well. But if help is likely to arrive in a week - you should be OK with the hot water tank contents and the toilet cysterns. I'll confess to having 50 one liter bottles of mineral water stashed away for emergencies. I bought them several years ago - and the water is still crystal clear...I presume that means it's still OK to drink. SteveBaker (talk) 15:10, 21 July 2009 (UTC)[reply]

The usual advice is to change out the water every 6 months, and put in a drop or so of household bleach per gallon of water to kill bacteria. equipped.org and its forums have a lot of info on water storage. 70.90.174.101 (talk) 06:00, 22 July 2009 (UTC)[reply]

Coal liquefaction economical at how many dollars per barrel?

I already asked on Talk:Coal liquefaction but I am in a hurry, I need to calculate economic projections, and I really need to know. Please answer wherever is more appropriate and I will make sure a summary appears in the other place. 99.60.2.8 (talk) 21:45, 19 July 2009 (UTC)[reply]

According to this article from China (2006??) "International indicators show that if the cost of liquefied-coal oil ranges from US$22 to US$28, the process is still profitable.... Presently, it costs around US$25 per barrel to produce one ton of coal-liquefied oil with three to five tons of coal used in the production." (Does that cost include the environment and transportation?) Here's another, 2007, Bloomberg article (ref'd in the WP Synfuel article). Twang (talk) 23:09, 19 July 2009 (UTC)[reply]
Well, if it's only $25/barrel and oil is $40/barrel, then WHY THE BLOODY HELL AIN'T WE DOING IT?! This technology could EASILY gain us energy independence and stop our money from going to the Arab terrorist savages and oil sheikhs, so why not start liquefying coal and start doing it TODAY?! 76.21.37.87 (talk) 00:31, 20 July 2009 (UTC)[reply]
According to those articles (which are a bit incoherent), those prices are based on a coal price of under $10 per ton. In the U.S. right now, high-quality coal gets $40-$50 per ton, and the price was much higher before the economic downturn. Looie496 (talk) 02:44, 20 July 2009 (UTC)[reply]
Oh, I see, it's the [regional] price of coal driving up the production costs. On the other hand, coal liquefaction don't need high-quality coal -- it works just fine with low-quality subbituminous coal, lignite and even peat if need be. The total efficiency is lower of course, but it'll still work. 76.21.37.87 (talk) 03:08, 20 July 2009 (UTC)[reply]
BTW, I've also read (in some research paper, don't remember the name off the top of my head) that they found a way to liquefy solid waste along with coal. The way that it works is they mix the coal with some waste tar (from steel foundries) or plastic or old tires or even garbage and then liquefy it in the same way as if it was just coal Coal liquefaction. If that's true, it can also greatly reduce the raw material cost (cause only half the raw material is coal, the other half garbage), and also have a big benefit for the environment (whatever...) by getting rid of all that garbage and old tires. FWiW 76.21.37.87 (talk) 03:16, 20 July 2009 (UTC)[reply]
Coal liquefaction also has large capital costs, and it becomes economical only at quite large volumes. (building that plant would cost a lot, take several years, during which price of crude oil might change almost several times) -Yyy (talk) 18:50, 20 July 2009 (UTC)[reply]
The price of crude will be going mostly up during those several years. 76.21.37.87 (talk) 04:50, 21 July 2009 (UTC)[reply]
@76.21.37.87 Similar things have been suggested and tried with other "waste materials". (turkey waste, cooking oil etc.) The problem is economics. Once you find a use for waste it becomes a commodity instead of useless trash. All of a sudden you'll get market dynamics driving up the price. 71.236.26.74 (talk) 19:42, 20 July 2009 (UTC)[reply]
So if you are smart and have invented a way to turn turkey poo into barrels of oil, the first thing you should do is go to all the turkey farms and offer them a very long term contract to buy the stuff for next to nothing. Googlemeister (talk) 20:02, 20 July 2009 (UTC)[reply]
It's usually turkey carcasses - and the real big turkey producers have their own plants...83.100.250.79 (talk) 22:21, 20 July 2009 (UTC)[reply]
No one has mentioned it, but coal to liquids is also dreadful from a global warming perspective. You release far more CO2 per barrel than if you were just working with petroleum. Dragons flight (talk) 20:05, 20 July 2009 (UTC)[reply]
Yes, indeed. More to the point, the idea that this will be a great technology once the oil runs out neglects the problem that we can't possibly use all of the oil we've got without flooding the planet to a depth of 20 to 30 feet of ocean. We don't need to swap one kind of hydrocarbon for another kind - anything with carbon in it is pretty soon going to have to be off-limits as a fuel source. Coal, natural gas and oil are all going to have to be strictly off-limits. Worse still, because it takes a lot of energy to turn coal into oil - the destruction of the environment caused by doing that will be worse than either burning coal or burning oil! Hopefully, the world will realise this pretty soon (the USA is MAYBE doing something halfway useful) and tax the shit out of fossil fuels of all kinds. At that point, coal liquifaction will be just about the most useless technology imaginable. SteveBaker (talk) 02:11, 21 July 2009 (UTC)[reply]
And then how the hell are we gonna get around, how the hell are we gonna get raw materials and finished products where they're needed most?! For that matter, how the hell are we gonna trade with other nations if we ain't got no ships (that run on fossil fuels)?! Your idea to "tax the shit out of fossil fuels of all kinds" is tantamount to throwing away almost 200 years of technological progress (and our American birthright of prosperity and economic leadership) for some crazy environmentalist agenda of limiting global warming, and it's recklessly irresponsible to say the least!!! Well, now that you've exposed your real agenda of having the civilized nations go back to a pre-industrial level of development (which you've disguised up to this point with all that crooked talk about "environmental issues"), maybe we can have a straight, honest conversation about whether such a course of action is good or bad for the USA... 76.21.37.87 (talk) 04:10, 21 July 2009 (UTC)[reply]
For the benefit of all others who want to weigh in on this: Note that SteveBaker wrote that "hopefully the WORLD will tax the shit out of fossil fuels", not "the USA will tax the shit out of fossil fuels". I wonder if he's also secretly advocating some supranational system of world governance (either under the auspices of the UN, or as a single "world government")... 76.21.37.87 (talk) 04:16, 21 July 2009 (UTC)[reply]
Cargo ships can be run on nuclear power. Railroad trains can be run on electricity. Airplanes and cars will not work without fossil fuels, though. Nuclear ships currently are more expensive than fosil fuel powered ones. Railroad electrification is also more expensive than using diesl power. -Yyy (talk) 05:15, 21 July 2009 (UTC)[reply]
Yeah, what about airplanes and cars, SteveBaker? Are you proposing that we all walk to work (oh so many miles in my case, and also in many other people's cases) instead of driving, and that we travel coast-to-coast by train (3 days at the very least) instead of by plane (a few hours)?! And also remember that "cars" also include the delivery trucks that transport merchandise from the regional warehouse (harbor, airport, train station, whatever) to nearby stores; what do you propose as a replacement, horse-drawn buggies?! Obviously no purely electrically-powered vehicle can offer us the same flexibility as a diesel lorry. Plus, keep in mind that electrifying every single railroad line in our country, and replacing all of the steam-powered / diesel-powered ships with nuclear-powered ships will cost hundreds of billions if not trillions of dollars, while creating only a marginal economic improvement (if any). This kind of expenditure with hardly any return on investment could literally sink our economy!!! Now you're getting some idea of what it would mean for our prosperity if fossil fuels are outlawed!!! And also, last but not least, nuclear-powered ships (or indeed any kind of nuclear-powered transport) have their own environmental / safety problems (like, what could happen in case of a shipwreck)... 76.21.37.87 (talk) 05:44, 21 July 2009 (UTC)[reply]
I'd say it is a pretty safe bet that by mid century ground transportation will be mostly electric. We already have a proliferation of consumer hybrids. The logical jump to plug-in hybrids will almost certainly be popular in the next decade, which reduces fuel consumption by ~80% for typical commuter use. From there I'd expect to also see a proliferation of light duty all electrics and some form of fuel cell vehicle for heavy duty uses (e.g. replacing diesel trucks). Hybrids are close to economical already, so it wouldn't take much of a refinement in technology or an increase in gas prices to make them a natural replacement for traditional cars. I suspect that similar concerns will also make the other technologies economical in time (especially with either a large carbon tax of a supply driven spike in oil prices). Once it becomes cheaper to go electric than not, it would just be a matter of waiting for electric vehicles to replace traditional ones as cars and trucks naturally get replaced due to old age. However, I am skeptical of nuclear ships, and planes will probably run on hydrocarbons for a long time. That said, we don't need to reduce emissions to zero. If one can remove most of the transportation problem and replace most power stations with renewables and/or nuclear, then we'd buy ourselves many more decades to come up with other solutions. Dragons flight (talk) 06:23, 21 July 2009 (UTC)[reply]
And how do you propose to "remove most of the transportation problem", eh?! Besides, you just admitted that the other technologies are supposed to become "economical with a large carbon tax" -- how do you think a "large carbon tax" will impact our economy and our industrial development?! It'll just raise transportation (i.e. gasoline, airfare, freight S&H, etc.) costs to unthinkable levels without making it any cheaper to "go electric"; as matters stand, it's the next-worst alternative to mandating electrification! And speaking of airfare, I repeat my question: WHAT ABOUT AIR TRAVEL?! Do we limit air travel (an indispensable part of our national economy)?! Cause that's what'll happen if a "large carbon tax" is put in place! 76.21.37.87 (talk) 06:43, 21 July 2009 (UTC)[reply]
Even for ground transportation, it's probably gonna be cheaper to liquefy coal than to go electric -- all it'll take is to build some coal liquefication plants (those could even be co-located with existing oil refineries and use those to refine the syncrude), which is a lot cheaper than replacing all of the energy-transportation infrastructure, trading in every car and truck in the country, and putting every railroad line under the wires. 76.21.37.87 (talk) 06:51, 21 July 2009 (UTC)[reply]
And as for hybrids, remember that they still use gasoline -- just a lot less of it. 76.21.37.87 (talk) 06:57, 21 July 2009 (UTC)[reply]
A good plug in hybrid, if regularly charged, is not going to use petrol for commuter transport which is what probably 90%+ of car travel is. Nil Einne (talk) 09:14, 21 July 2009 (UTC)[reply]
So how is the progress going to make a plug in tractor trailer with cross country range? Googlemeister (talk) 13:38, 21 July 2009 (UTC)[reply]
We call those "trains". They do require some initial investment in infrastructure, but are otherwise entirely possible and reasonably efficient. APL (talk) 20:03, 21 July 2009 (UTC)[reply]
They also run on rails, which limits the places they can go (to say the least). For cross-country freight transport, trains are useful, no doubt; for delivery to local retail stores, not so much. 76.21.37.87 (talk) 23:32, 21 July 2009 (UTC)[reply]
Well, we have a pretty clear choice - allow most of the major cities of the world to wind up under 20 feet of water, have most fertile land in the world covered by ocean, massive species die-off, etc...but still be able to drive to work in gasoline powered cars...or we fix it. Fixing it isn't going to be either cheap or easy - but it certainly doesn't mean that we have to revert to a pre-industrial society. (I sure as hell hope not - I make computer games for a living - I don't think I'm well-suited to an agrarian life-style!).
We have to wind up with an electric/hydrogen economy. We need all the windmills, tidal, geothermal, solar and (regrettably - but of necessity) nuclear and hydro power we can scrape together - and fill the inevitable gaps with plant-based (but preferably not corn-based) ethanol and other 'renewables'. We need to agressively pursue energy saving measures where we can. We need to work harder on fusion reactors and consider space-based solar energy plants.
Cars will (by preference) be smaller than they are now and probably plug-in electric - but where longer range is required, hydrogen or perhaps plant-ethanol driven. Not many people need to drive more than 100 miles per day - so commuter cars can be electric - and we can either rent or own hydrogen/ethanol cars for the occasional road-trip. Airline travel needs to be drastically scaled back - we can cut out almost all business travel using the Internet. We need to re-vamp the rail network (which is easy to electrify) and use it to replace trucks and planes where-ever we can. Long distance shipping will become much more expensive - so "cheap" goods from overseas won't be anything like as cheap - but hey - the flip side of that is that outsourcing becomes a lot less attractive when shipping costs more than labor.
Houses, offices and factories need to be better insulated and tools and appliances should be carefully designed to save energy.
I don't think our lifestyle has to change as much as the scaremongers think - and many of the changes are for the better. But the sooner we do it - the less painful it's going to be. When our crops start failing and our cities start to be deluged - it's going to be a LOT harder to fix.
The tricky part right now is to get the whole world to pull together. Europeans are definitely sold on this. There are promising signs that China and the USA are beginning to get active about it - but India is still a major problem.
SteveBaker (talk) 14:09, 21 July 2009 (UTC)[reply]
While it would be optimal from a high altitude perspective to do all the things you suggest above - and many of them are or will be done - there are significant difficulties fitting it into the US government budget without a serious carbon tax, which both parties seem to have decided is unacceptable. Instead they have decided to go with a "cap and trade" system in which they give away 75% of the emission credits, completely defeating the purpose and handing the ball to lobbyists and otherwise politically connected companies.
Barring some direct value placed on not emitting CO2, it seems green energy will have to compete in the free market based on economics alone and unfortunately at the moment it does not fare well against established technology - largely coal and oil (wind advocates will tell you different, but they often use selective mathematics). The real impetus to act in this non-carbon tax scenario would be a sustained high oil price. It is perhaps ironic that the companies most hurt by oil's collapse are the green energy startups.
I would also urge you to consider carbon capture. I know it's an unproven technology and it has some real flaws. But there is also potential there to allow us to use the world's massive coal reserves, without irresponsible emissions, for decades if not centuries to come. I'd say it is at least as promising as some of the more outlandish green energy proposals (including a complete "hydrogen economy"). TastyCakes (talk) 14:31, 21 July 2009 (UTC)[reply]
The trouble is that we can't bank on carbon-dioxide sequestration to actually work. We also can't bank on fusion power or space-based solar working either. It's also really too late to bet the farm (literally!) on a completely theoretical possibility like that. We MUST proceed on the basis that neither fusion nor carbon sequestration will work - and if either or both DO happen to come through with the goods - then we'll be a much happier planet.
However, let's consider what you say. Suppose for one moment that CO2 sequestration on an industrial scale were to work (not even the most crazed advocates claim you could do it on a small scale). You can't run cars on coal - and even if you could, you couldn't sequester the CO2 on that scale - you can't run airplanes or trucks on coal either. You MIGHT (maybe) be able to do it for very large ships - but not for trains or individual homes.
So even if we HAVE CO2 sequestration at some time in the future - we STILL have to have: Electric cars, all-electric homes and factories, etc, etc. You can't dig up coal, turn it into oil (as our OP suggests) put that into a car and THEN sequester the CO2! So every single one of the things I describe above has to happen at the exact same pace of development and at the exact same rate of funding and commitment whether CO2 sequestrated "clean coal" eventually works or not. So - by all means lets invest in the research - but that cannot, MUST NOT stop us from switching over to an electric/hydrogen/plant-ethanol economy.
FWIW, I think CO2 sequestration is a ridiculous concept - where on earth we put all of the CO2 even if we could sequester it is a completely un-thought-out problem! SteveBaker (talk) 21:14, 21 July 2009 (UTC)[reply]
(ec)You just finished advocating electric cars and trains that essentially could run on coal! Further, power plants comprise about 40% of CO2 emissions in the United States (and probably significantly more in places like China). This number would only increase in your proposed electric future. If the goal is to get off of oil, that's one thing, but if the goal is to reduce emissions that's quite another. I hold no illusions over the former, but carbon sequestration could certainly address the latter. Further, I think you misunderstand the problems with CO2 sequestration. There are massive depleted natural gas reservoirs in the United States, indeed throughout the world, which formerly held hundreds of TCFs of natural gas. Huge amounts of CO2 could be injected into them without going anywhere near the fracture pressure which would allow it to escape, and it could be monitored very easily with conventional oilfield equipment. Gas has been in these geological formations at pressure for millions of years, I see no reason to expect it won't stay there long enough for human purposes. The problem is you'd need a massive CO2 collection system since the powerplants are for the most part nowhere near these reservoirs. Perhaps worse, you have to process the gas to separate out other problematic things before you can inject it, and then when you inject it your compression requirements are not insignificant. CO2 is already injected in many oil fields as a type of Enhanced Oil Recovery to maintain reservoir pressure and reduce viscosity. But barring that motivation there is no financial incentive for companies to do it - unless someone (ie the government) is willing to pay them to do it, or to punish them for not doing it.
I am not saying that we shouldn't do all the things you have said. But personally, I'd bank on geological sequestration long before I bet on nuclear fusion or Space Based Solar. And this isn't a game of absolutes - no one is going to click their fingers one day and have us all driving fuel cell cars. Even if only a portion of emissions can be sequestered, it is still more breathing room with regards to the problem as a whole. The question is whether that is the most economically effective way it can be done. TastyCakes (talk) 21:43, 21 July 2009 (UTC)[reply]
I'm actually an optimist about carbon sequestration eventually working (though it still needs to be demonstrated). As a society we currently remove 100 trillion cubic feet of natural gas from the Earth each year. Most of that gas has been safely sequestered in the Earth for millions of years. For comparison, we need to be able to store 600 trillion cubic feet per year of CO2 in the ground to totally eliminate all current emissions. That's only 6 times larger than the flux of gas we currently take out. The comparison to natural gas shows that it is possible for the Earth to store massive quantities of gas for geologically relevant intervals. Figuring out how to take advantage of that on a practical level is a massive undertaking, but I don't think it is an impossible problem. Most of the estimates of the potential storage capacity of deep reservoirs suggest that we'd run out of usable fossil fuels before we run out of places to store CO2, though without large scale demonstrations it is hard to know how accurate those estimates are. And of course, potential reservoirs may not exist at all the locations where one might want to have them. Dragons flight (talk) 21:32, 21 July 2009 (UTC)[reply]
PS. I realized my comment might be misinterpreted. Steve is right about needing to pursue various options to address the various sources, etc. My comment is meant to focus just on the feasibility of large scale carbon capture and storage when you have point sources to work from. Dragons flight (talk) 21:45, 21 July 2009 (UTC)[reply]
My point is that even if CO2 sequestration does eventually work and is 100% effective - all that buys us is 'green' electricity. We STILL have to make the transition to electric cars, hydrogen fuelled ships, trains, airplanes, etc. It makes absolutely ZERO difference to the strategy we should be pursuing right now - at this very minute. If we don't start phasing out gasoline-powered cars/trucks/ships now - then there will still be an alarming number of them still on the roads in 10 to 15 years time. But the risk that this technology doesn't pan out is just too high. We can't rely on it - the stakes are just too high. So by all means let's spend some money to research it - and let's spend some money on fusion and other possibilities too...but we can't let the idea of "Clean Coal" be the answer to all of the problems. Even if it's 100% effective and does everything the enthusiasts claim for it - we still have to overturn the entire fossil-fuel based economy into something based around electricity/hydrogen. SteveBaker (talk) 23:20, 21 July 2009 (UTC)[reply]
Electricity won't run cars / trucks / planes, as Yyy correctly pointed out. And as for hydrogen, remember, it don't happen naturally -- it's made using fossil fuels (either natural gas, or in some cases coal gasification. 76.21.37.87 (talk) 23:49, 21 July 2009 (UTC)[reply]
Assuming that global warming is actually happening, is actually cause by humans, and can actually be prevented it might make sense to spend the trillions of $ to do something about it, if you can get everyone on board. Change will not be from carbon taxing, it will happen from high oil prices caused by supply shortage. Saw that last summer when the total number of miles driven by Americans dropped off a good bit. Googlemeister (talk) 16:37, 21 July 2009 (UTC)[reply]
I agree with your skepticism, I too am not entirely convinced of the global warming story. My supporting a carbon tax is based on the assumption that it is true and is as pressing an issue as presented by its proponents. If it is, a simple flat percentage carbon tax seems the most fair, transparent way to make energy users and producers value any process that prevents CO2 emissions. It gives them the latitude to do whatever is most effective at reducing those emissions, whether that be building a nuclear plant or a wind farm, building more transmission lines to hydro dams or sequestering carbon in some way. Under such a system, the market would determine the most effective means of avoiding emissions. I would take such a market over a group of politicians deciding where to apply subsidies and other market manipulations any day, and it would have avoided throwing huge amounts of money into things like corn-based ethanol, in my view a technology that is utterly pointless except as a tool of political pandering. TastyCakes (talk) 17:09, 21 July 2009 (UTC)[reply]
I really think that at this point, if you're still skeptical, it can only be because you haven't read or understood enough about it. There is massive evidence for global mean temperature rise, melting ice - and worse...melting methane clathrates, migration of plants and animals further north than they've ever been, vanishing glaciers...how can those NOT be due to a general warming of the planet?
Even if you don't believe humans are at fault - the consequences will still be cruelly felt by everyone - and it behooves us to try to prevent it if we want anything approximating our present civilisation to survive. I don't care about pointing the blame at nature or at humans. If some theory about the end of a mini-ice-age or the sun producing more energy or a bizarre coincidence of volcanic activity makes you feel happier about yourself - then fine. But even if it IS one of those things - we still need to cool the planet or all of the consequences will still happen. The only safe way we know to do that is to cut down the amount of greenhouse gasses. So - OK - maybe I buy your viewpoint...but we still need to do something about it - that doesn't change a thing!
Worse still - if you think an oil shortage will save us - you really need to crunch the numbers (I've done it right here on this very Ref Desk on several occasions). What those numbers say is that by the time we've burned enough oil to start pushing the prices up to the level where burning the stuff becomes uneconomic - it's already FAR too late. The amount of CO2 produced by burning (say) 50% of the world's present reserves would be more than enough to bring about the worst of the predicted catastrophies. Worse still - if you are completely honest - you'll have to admit that we won't simply burn all of the easy-to-get oil and then stop and switch to something better - because it'll then become economic to mine currently uneconomic oil-shales - and to use that fancy new BP technology that makes liquid fuels from natural gas - or (as our OP suggests) from the liquifaction of coal. So the price of producing CO2 won't get really prohibitive until all of the oil, coal AND natural gas gets used up - and the world's coal reserves are simply massive - so that's death to most of the life on the planet - it's so much CO2 that we'd die of CO2 poisoning, let alone global warming!!
The entire problem with stopping this catastrophy is that simple market forces won't help us at all if the amount of CO2 required to wreck the planet is less than what we'd produce by burning the remaining oil stocks...and it is...MUCH less! So the approach you suggest simply won't work.
If unmodified market forces won't do it - we're left with simply banning the stuff - or taxing it into extinction - or artificially limiting it's use via stuff like cap & trade or some other clever fiscal trick. I'm neither a politico nor an economist - and I honestly don't know (or particularly care) which of those we do...so long as we pick one that works. No matter which it is, the consequences are the same. If we're not going to force people to stop using fossil fuels by simple rule of law - we can artificially push the prices up to the point where other alternatives are cheaper - or we can limit the output of CO2 with a cap and let the market decide who gets to economize. For sure the level of the presently proposed cap won't help much - but a gradual ratchetting down of the size of the cap (how much each CO2-production "credit" represents) is a possible solution. Cap and trade (done right) would entice the open market to spend money on researching and constructing alternative energy sources and ways to cut energy usage. A flat out carbon tax would give the government enough funding to do that research and construction. If you are on the right wing of the political spectrum then you should advocate cap & trade and keep the government out of the business of making green products - if you are on the left then a carbon tax makes sense because it guarantees that the money to do the work will appear and be directed in one coherent direction. As I said - that's a political choice, I don't much care which it is...but we need one or the other and we need it yesterday. SteveBaker (talk) 23:00, 21 July 2009 (UTC)[reply]
You may be right about global warming. But there lies the rub: I'm not an expert on the matter and haven't studied it academically, but I consider myself an educated person, and I have read a reasonable amount on the subject, yet I've never seen the argument broken down to the point where I buy it hook line and sinker. I think it is clear that the climate is changing - I have been to the Athabasca ice fields where there's a little sign where the glacier was 50 years ago, and it's a long way from where it is now. And I've been to Joshua Tree National Park, where areas that ranchers used to graze their cattle on are now far too barren to consider that. But as you point out I am not convinced that humans have been the dominant factor with this, and even less so that it's carbon dioxide emissions that have done the most harm. But perhaps where I diverge from you the most is that I'm not convinced that we can do anything to stop the rise in temperatures even if we really tried, although it seems quite clear we could beggar ourselves in the process (or to be more precise, force the third world to remain beggared rather than develop their societies). We know the Earth's climate is not a constant and that it has been far warmer in the past. We know we're in an interglacial period of gradual warming. I won't dispute that temperature rise could have devastating consequences. But I am not convinced that this hasn't been in the cards all along or that our puny efforts to curb it aren't the height of arrogance. I hope I'm wrong and that humans really are masters of our own domain and that our domain now constitutes the entire planet, able to control its climate to our liking. I suppose time will tell. Like I said before, I believe measures should be taken, including a carbon tax (which I do not believe is the "left option", and I'm not sure I follow the logic that leads you to that conclusion), but only because, as you point out, perhaps we can halt the temperature increase and the damage it entails. But at this time I do not see it in the same all or nothing, do or die light as yourself, mainly because I think there is reasonable possibility that we can't do anything to stop it no matter how hard we try. TastyCakes (talk) 03:18, 22 July 2009 (UTC)[reply]
And how are we gonna make sure we do this without destroying our economy (which is what'll happen if we start artificially raising the price of fossil fuels)?! Remember, our American way of life is our birthright and is NOT to be thrown away for some notion of "saving the environment"! 76.21.37.87 (talk) 23:54, 21 July 2009 (UTC)[reply]
Why would it destroy "our" economy? We can do a gradual, predictable shift from other taxes (especially sales taxes) to carbon emission taxes, keeping the overall tax quota constant. It's a simple system that fully leverages the ability of the market to provide cost-effective solutions. Last years oil crunch has done more for green energy in the US than any number of direct subsidies, and we are all still alive. Keep the oil price at 150 US$/barrel for a few years, with other fossil fuels taxed correspondingly, and we will see a massive shift towards less energy-intensive technologies and carbon-neutral energy sources. --Stephan Schulz (talk) 11:43, 22 July 2009 (UTC)[reply]
Too late, Steve. Now that 76.21 has skillfully exposed your hidden agenda to create a One World Government with the goal of reverting us to a pre-industrial society we'll never believe your 'crooked talk' of so called Global Warming again!
Also, besides relying on video games for your livelihood, iirc you're also an advocate of space exploration. You clearly haven't thought this pre-industrial business through. You would do well to learn from 76.21's logical and rational thought process. APL (talk) 20:15, 21 July 2009 (UTC)[reply]
I'm humbled by the razor sharp edge of his/her crazed ramblings cogent arguments. SteveBaker (talk) 21:14, 21 July 2009 (UTC)[reply]
So, let me get this straight here. On the one hand, SteveBaker is advocating space exploration; on the other hand, he advocates an outright ban on fossil fuels?! Doesn't he understand that all spacecraft require fossil fuels to get into space in the first place?! Clearly he needs a refresher course in logical thinking. 76.21.37.87 (talk) 23:44, 21 July 2009 (UTC)[reply]
This is nonsense. Ariane 5 has purely cryogenic (H2/O2) main stages. I don't know what's in the propellant for the SRBs, but this can certainly be synthesized without use of fossil fuels. Anyways, you are committing the frequent fallacy of argumentum ad consequentiam. The effect global warming will have on the world or the economy or our life style or Steve's ability to fly to Titan has not the least effect on the degree of correctness of the theory. --Stephan Schulz (talk) 23:58, 21 July 2009 (UTC)[reply]
For the record, hydrogen is made from natural gas or from coal, so fossil fuels are used in its manufacture. I'm not sure what the SRBs are made of (have to look it up), but I'm pretty sure that fossil fuels are used at some point during their manufacture. As for your statement about "appeal to consequences" -- for the record, I'm not disputing the correctness of global warming here (although I'm not convinced of it), only stating that the course of action SteveBaker advocates will lead to unacceptable consequences for the American economy and way of life. So don't try to go putting words in my mouth cause that's not gonna work. 76.21.37.87 (talk) 00:51, 22 July 2009 (UTC)[reply]
One day I want to get a job working in a Liquid Oxygen mine, digging up fossil rocket fuels. APL (talk) 00:43, 22 July 2009 (UTC)[reply]
That's not even funny. 76.21.37.87 (talk) 00:56, 22 July 2009 (UTC)[reply]
Just looked up Space Shuttle Solid Rocket Booster: it uses ammonium perchlorate as an oxidizer (which requires hydrogen for its manufacture, the hydrogen being derived from natural gas or from coal, as I've mentioned two or three times already), and also requires butadiene and epoxy resin, which are both made from natural gas or petroleum. Stephan Schulz, your assertion that SRBs can be made without fossil fuels is completely wrong. 76.21.37.87 (talk) 01:25, 22 July 2009 (UTC)[reply]
The Space Shuttle does use ammonium perchlorate. So? Many space craft launch on substances that could theoretically be synthesized in an entirely green way. In fact, I don't see any reason Ammonium Perchlorate couldn't. Just because they aren't currently doesn't imply a logical contradiction between supporting both space travel and a fossil-free world. In fact Space-based solar power could, in theory, be one of the technologies that allows us to reduce our reliance on fossil fuels. (Though closer to home technologies like renewable energy sources and nuclear are far more likely in the near future.) APL (talk) 02:59, 22 July 2009 (UTC)[reply]
"Many space craft launch on substances that could theoretically be synthesized in an entirely green way." -- I challenge you to give an example of a spacecraft that does not require any fossil fuels for fuel, and then maybe I'll believe what you said. And as for the synthesis of ammonium perchlorate, I already told you that it absolutely requires hydrogen, which in turn must be made either from natural gas by autothermal reforming, or from coal by the water-gas reaction. (I'm a petroleum chemist, I know how these things are made.) And regarding "space-based solar power": it's absolute BS, it'll cost trillions of dollars in capital cost while still being more expensive in terms of operating cost than coal / natural gas / hydroelectric / nuclear power. You might as well take ten billion benjamins and launch'em into space instead! Renewables might be better in terms of economics than space-based solar, but there won't be enough of'em to meet all our energy needs, so nuclear is the way to go (as far as electricity production is concerned). As for transportation, we'll still have to use hydrocarbons for that for the foreseeable future (which is why a carbon tax is a bad idea, it'll cripple our transportation system that our economy depends on). 76.21.37.87 (talk) 03:36, 22 July 2009 (UTC)[reply]
It's technically trivial to make hydrogen by electrical hydrolysis of water. It takes more energy than using coal/natural gas as a source, but if you have clean electricity (from whatever source) you can certainly make hydrogen without fossil fuels. Dragons flight (talk) 03:59, 22 July 2009 (UTC)[reply]
You're certainly right about electrical hydrolysis taking (much) more energy than autothermal reforming. This also means that it'll cost much more to make hydrogen in this way than by reforming; in fact, this process is hopelessly uneconomic compared to natural gas reforming or coal gasification. And higher price of hydrogen will mean a much higher price for launching stuff into space, which will greatly hinder any kind of space exploration. 76.21.37.87 (talk) 04:09, 22 July 2009 (UTC)[reply]
I'll take reasonable bets the that price of the (raw) fuels are a negligible part of the the overall cost of a space mission. Apparently, hydrogen prices are less then US$1/kg for large quantities. The Shuttle uses about 100000kg. The marginal cost of a Shuttle launch is about US$ 60 million, the total program cost is about US 1.3 billion per launch. At the moment, making H2 by electrolysis is about 3 times more expensive than via methane reformulation. So that's US$ 200000, about 0.3% of the marginal cost, and 0.015% of real cost. --Stephan Schulz (talk) 10:03, 22 July 2009 (UTC)[reply]

July 20

Green things growing in water

One of the answers to the question above reminded me of something I heard on the radio whilst half-asleep this morning (it was the early morning fishing phone-in on TalkSport, if anyone is interested). According to the host, if you were to fill a pop bottle or a jar with ordinary tap water, seal the lid down tight, then leave it outside in a place that catches the sun - within a couple of weeks the vessel will be completely filled with algae and similar-looking slimy green stuff.

Supposing that this is true - how on earth does it work? --Kurt Shaped Box (talk) 00:58, 20 July 2009 (UTC)[reply]

Algae, Algal bloom, Seaweed, Phytoplankton, Bacterioplankton and Cyanobacteria may help. Basically you are creating an environment that promotes growth for plant/bacterial species without the animals that would keep them in check in the open water. 71.236.26.74 (talk) 02:26, 20 July 2009 (UTC)[reply]
Nuts. It's simply not true. You can't have any of those things without sources of carbon, nitrogen, sodium, and potassium, among other things, and none of those are present beyond trace amounts in tap water. Looie496 (talk) 02:31, 20 July 2009 (UTC)[reply]
I don't believe it. Where would it get nutrients? Tap water is pretty much a lost cause, so you're stuck with the soda residue?
People building Algae reactors put an awful lot of effort into making "green stuff" grow. Just putting some tap water in the sun is far too easy. APL (talk) 02:35, 20 July 2009 (UTC)[reply]
Yeah, that's what my high-school knowledge of algae was telling me. Guy also said that distilled or boiled water definitely doesn't work... --Kurt Shaped Box (talk) 02:37, 20 July 2009 (UTC)[reply]
Actually tap water isn't a lost cause - depending on where you live, there's likely to be enough micronutrients in your tap water to support algal growth. Again, depending on where your tap water, the nitrate content can be fairly high. There's also going to be CO2 in the water - it's fairly soluble in water, and depending on your water source, the water may contain a significant amount of carbonate/bicarbonate derived from mineral sources (think lime scale).
It's also important to realise how little mass there is in a thin film of algae. Guettarda (talk) 06:15, 20 July 2009 (UTC)[reply]
Ok, I can believe that, but he said "completely filled with algae". APL (talk) 12:39, 20 July 2009 (UTC)[reply]
True. But "completely filled" is in the eye of the observer. I really should try it... Guettarda (talk) 13:07, 20 July 2009 (UTC)[reply]
It really depends on pot-luck what's in the water. There's this species of algae that form long threads a couple of those and the bottle may look to some as though it's "filled". I think Zygnematales may be what I'm thinking of. They can use hydrogencarbonate in water as well as CO2. If the show host's tap water supply is lake water that has undergone comparatively little filtering that might work. They would grow a whole lot better, though if he left the lid on only loosely. The fact that he wants the lid on tight makes me think that he might be trying to breed Cyanobacteria. Our article says there are freshwater species. Kurt and his show host are in the UK in Europe. The EC have a "directive on the quality of water intended for human consumption" ((98/83/EC) [43] and related documents. Based on those member states have issued lists of required mineral content and maximum levels for other stuff. European countries generally don't chlorinate their water as heavily as US utilities, unless they have to. So the experiment suggested may yield the observed results only in certain locations. If you use water from a pond rather than tap water it should work in most places. 71.236.26.74 (talk) 19:11, 20 July 2009 (UTC)[reply]

How much energy remains in the spent nuclear fuel produced yearly from US nuclear reactors?

Each year America's current 104 LWR reactors produce 2000 tons of spent nuclear fuel.

America's LWR only burn a tiny portion of the Uranium fuel in their fuel rods (about 3% or less) before the fuel rod is considered expended and is changed out.

What is the energy value left in 2000 tons of spent fuel rods after they have been removed from US LWRs, assuming complete burning of all fissile (U-235) and fertile (U-238) Uranium in an ideal breeder reactor that would extract all of the energy from the Uranium fuel?


source: IDB Reference Characteristics of LWR Nuclear Fuel Assemblies from the 1996 Integrated Database Report provides the following reference data about PWR and BWR fuel assemblies: http://www.nucleartourist.com/basics/hlwaste.htm —Preceding unsigned comment added by Friend2all (talkcontribs) 02:58, 20 July 2009 (UTC)[reply]

Prob'ly not much, at least in terms of usable energy. If all the uranium is burned up, what's left is just the fission products, and they (mostly) decay too quickly to be of much use for energy production. Besides, until the fission products decay, the spent fuel is lethally radioactive so there's no conceivable way to extract the energy from it. FWiW 76.21.37.87 (talk) 03:22, 20 July 2009 (UTC)[reply]

I must add that an "ideal breeder" is impossible to construct -- even the most advanced breeders will become poisoned by fission byproducts when their concentration in the fuel reaches 15-20% or so (during which time an equivalent amount of U-235 will be burned up, and maybe 20-25% of U-238 will be converted to fissile Pu-239), so this is obviously not a realistic scenario. (Although you could reprocess the fuel multiple times using advanced nuclear reprocessing methods to separate out the fission products and reuse the uranium and plutonium, which would give you more-or-less the same effect.) FWiW 76.21.37.87 (talk) 03:30, 20 July 2009 (UTC)[reply]

How much energy does a typical LWR reactor produce from a fuel rod in the current design and change-out schedule (i.e., what's the yield when 3% is used)? Assuming you could get up to the full 100% via the same decay processes (i.e., completely consume the partially-consumed isotopes using the same reactions), my hand-waving damn-all-practical-considerations answer is "an additional 97/3 times what 3% gives". DMacks (talk) 08:54, 20 July 2009 (UTC)[reply]
Sorry, I misunderstood the question. (Thought the OP asked how much energy remains in the fuel rods after 100% of the uranium's burned up.) 76.21.37.87 (talk) 06:23, 21 July 2009 (UTC)[reply]
It seems like lots of potentially useful fuel is to be expensively entombed for millenia over political objections of the state where the burial is to be. Breeder reactors could provide much more energy from the "spent" fuel. The problem is that bomb materials would be part of the fuel cycle and it is hard to tell "rogue" states not to emulate the present world powers if they built breeders. Edison (talk) 03:38, 21 July 2009 (UTC)[reply]
I agree, Edison, breeder reactors and reprocessing technology are the way to go (unfortunately, I don't have a whole lot of knowledge about these technologies, except for a little bit on the reprocessing end). As for your concern about proliferation, there's a pretty simple way to prevent that: Simply make a trade embargo on all breeder parts / materials / technologies, and strictly enforce it!!! That will put a stop to all those terrorist nations building breeders and making bomb material! It will require eternal vigilance of course, but it can be done. 76.21.37.87 (talk) 04:24, 21 July 2009 (UTC)[reply]
Everything needed to make a basic breeder reactor is already public knowledge. An embargo could slow things down, but any nation large enough to have the appropriate raw materials and industry could build one from scratch eventually. Dragons flight (talk) 04:54, 21 July 2009 (UTC)[reply]
Most terrorist (i.e islamic) nations don't have the industrial capacity to build a breeder from scratch at this time, so we should concentrate on making sure it stays this way. 76.21.37.87 (talk) 05:23, 21 July 2009 (UTC)[reply]
Besides, those people are too retarded to build a breeder reactor without Western (or Russian, or Chinese) nuclear experts doing all the design work. (All they're good at doing is blowing people up.) Keep the expertise out of the terrorist nations, and that will stop them from making the bomb. 76.21.37.87 (talk) 06:09, 21 July 2009 (UTC)[reply]
Can you tone down the anti-Arab/Muslim rhetoric a little? It's irrelevant, unhelpful, and offensive. -Elmer Clark (talk) 19:19, 21 July 2009 (UTC)[reply]
O ho, so now look who's sticking up for terrorists! For the record, Islamic nations are behind (almost) all the terrorism that's been happening in the past 30 years; they're sold on the idea of global jihad to force their ideology on others; there's widespread support for terrorism in most Islamic nations (in particular, in the Arab nations at least 60% or more of the total population supports terrorism, just look at the Gallup polls that's been conducted there); while at the same time the Islamic nations are the most oppressive totalitarian dictatorships in the world (just look at what's happening in Iran, for example)! And as for them being retarded, statistics show that the Arab world "produces virtually zero scientific papers and patents" (as shown by a 2002 UN report on the condition of the Arab world), which is proof enough that Islam rots your brain more than anything else. Must I add the Islamics' extreme mistreatment (to say the LEAST) of women, which is well-documented in many different sources and which is yet more evidence that they're nothing more than savages and barbarians?! 76.21.37.87 (talk) 00:24, 22 July 2009 (UTC)[reply]
Your vast oversimplification of very complicated subjects is the very definition of bigotry. It's also unrelated to the question. Please keep it civil and at least vaguely on topic. TastyCakes (talk) 02:43, 22 July 2009 (UTC)[reply]
What's so complicated about terrorism, I would like to know?! They've been fighting their jihad against the Western world since time immemorial, and we must fight back until we win the final victory, IT'S REALLY THAT SIMPLE. Besides, their so-called "culture" is responsible for more violence worldwide than any other culture (both toward other nations and toward their own people), while at the same time creating very little of anything worthwhile. I agree that this discussion is not directly related to the question, but I was giving my views on how to stop terrorist nations from acquiring nuclear weapons technology (which is COMPLETELY related to the original question), and it was Elmer Clark who brought up this discussion in the first place. And BTW, why would I "keep it civil" with regard to the savages who BRUTALLY MURDERED three THOUSAND of my countrymen/women on 9/11?! 76.21.37.87 (talk) 03:51, 22 July 2009 (UTC)[reply]
Regardless of whether there's any merit to what you're saying, there's no need to bring up your hatred for Arabs and Muslims practically every time you make a comment. Frankly you're coming off as a right-wing jingoist character account. -Elmer Clark (talk) 13:06, 22 July 2009 (UTC)[reply]
Ya, I'm starting to think this guy is just a troll. TastyCakes (talk) 14:14, 22 July 2009 (UTC)[reply]

Calorie/Kilocalorie

Hello, I am asking this question as I am confused over it. I noticed several sites mention the daily calorie requirement for an adult male in calories, for example someone needs 2400 calories per day. Is it correct? I mean will it be calorie or kilocalorie (i.e. 2400 Kcal)? For example a Cucumber has a food value of 20 kcal, then taking only one cucmber means taking 20,000 calories which is far more than daily calorie requirement. --AquaticMonkey (talk) 03:16, 20 July 2009 (UTC)[reply]

No-one uses the original calorie when talking about nutrition. Calorie always means the same as kilocalorie. Algebraist 03:18, 20 July 2009 (UTC)[reply]
See Calorie#Kilogram and gram calories for a discussion of this. The article says “in the context of food energy the term calorie generally refers to the kilogram calorie”, but I think that’s a severe understatement. I don’t think I’ve ever seen “calorie” mean “gram calorie” in a food context. Red Act (talk) 03:41, 20 July 2009 (UTC)[reply]

You also should know that daily intake to maintain an adult's wt varies enormously. 2400 may be average, but some people need half that and some twice that. alteripse (talk) 16:22, 20 July 2009 (UTC)[reply]

For example, if you're performing hard physical labor (working in a furniture warehouse, etc.), you can easily require 5000+ calories a day. 76.21.37.87 (talk) 04:27, 21 July 2009 (UTC)[reply]

Little greedy bastard

I have just been savaged by one mosquito. I have a simple question. Why did the little bugger, per their typical MO, feed on me four or five times in just a few minutes, rather than just once? I scanned our article on them it does not explain this feeding habit.--Fuhghettaboutit (talk) 04:24, 20 July 2009 (UTC)[reply]

Because you're delicious. --jpgordon::==( o ) 06:02, 20 July 2009 (UTC)[reply]
Are you positive it was the same mosquito?CalamusFortis 06:33, 20 July 2009 (UTC)[reply]
My guess is that it probably has something to do with the size of the blood meal. Either your skin is hard to get through, your blood clots too quickly, or you're chasing the poor mosquito away before she's finished feeding. All she wants is enough protein to produce a batch of eggs. Guettarda (talk) 13:13, 20 July 2009 (UTC)[reply]
I am quite delicious, and I Guettarda makes a good point about my androcentric selfishness, but after some thoughtful self-examination, I find I still hope she choked.--Fuhghettaboutit (talk) 22:09, 20 July 2009 (UTC)[reply]
There was a TV commercial for Tabasco sauce (or some similar product) where guy was eating a bunch of it, mosquito bit him, and moments later the critter burst into a little fireball. DMacks (talk) 00:48, 21 July 2009 (UTC)[reply]
I read once that mosquitoes go for overweight men more than any other demographic. Read into this what you will. Vranak (talk) 01:30, 21 July 2009 (UTC)[reply]
My guess would be simply that the mosquito knows it only has a limited time before you notice it biting you. It takes a short while between being bitten to when the swelling and itching starts, the mosquito doesn't want to be around when that happens or there is a much greater chance you'll swat it. It makes even more sense if the mosquito's prey is not humam. Vespine (talk) 04:13, 21 July 2009 (UTC)[reply]
You may be tightening your skin when you notice them. They notice the tightening and fly away because they expect you to swat them. Try to keep your skin relaxed if you're going to try to swat. 70.90.174.101 (talk) 06:03, 22 July 2009 (UTC)[reply]

Beetles nesting in old disks

While going through storage to throw things out I came across a number of old SyQuest 44mb removable hard disks that probably haven't seen the light of day in almost 10 years. Inside each one I found a sizable number of beetle larva (probably A. unicolor), both live and shed casings. Is this a common occurance? Is there something in the disks that makes them attractive? Horselover Frost (talk) 05:26, 20 July 2009 (UTC)[reply]

Dust? --Dr Dima (talk) 07:35, 20 July 2009 (UTC)[reply]
Did those disks have Beatles songs recorded on them? Maybe that's why those beetles found them disks to be so attractive. ;-) 76.21.37.87 (talk) 04:30, 21 July 2009 (UTC)[reply]

Fusion of vertebrae with age - painful?

I understand that as people age, more vertebrae of the lower spine fuse together. When two vertebrae fuse, is this painless and unnoticed by the subject? 78.146.249.124 (talk) 11:04, 20 July 2009 (UTC)[reply]

Remember that Wikipedia does not give out medical advice, but you might find information at Spinal fusion, which describes the deliberate fusion of vertebrae as "to relieve pain" not cause ir. SGGH ping! 11:17, 20 July 2009 (UTC)[reply]
I'm not quite sure what medical advice is being sought here, seems like a straightforward enquiry about a condition. Let's assume good faith. Ankylosing spondylitis is a condition where fusion of the vertebrae occurs. The article contains information you may find helpful. Richard Avery (talk) 15:14, 20 July 2009 (UTC)[reply]
Fusion of the vertebrae in the Sacrum is a part of normal development, see [44]. It is not painful, and is unnoticed by the subject. As far as I know, fusion of the lumbar, or of other vertebrae, does not occur in a healthy subject. --NorwegianBlue talk 19:15, 20 July 2009 (UTC)[reply]
The original poster may be referring to bone spurs that commonly form on vertebrae as people age. They don't typically "fuse" vertebrae together, but spurs from one vertebra may touch spurs from an adjacent one, and they may impinge on nearby spinal nerves which certainly can cause pain or paresthesia. - Draeco (talk) 03:53, 21 July 2009 (UTC)[reply]

Capillarity

I was reading a book in soil science, expressing the capillarity in tubes. It is suggesting that the pressure in the raised water inside the tube in negative, which I don't feel well that. I'd like to know more about capillarity specially from molecular point of view. Could anyone give some help and sources? Re444 (talk) 11:30, 20 July 2009 (UTC)[reply]

Xylem and Capillary action Surface tension would be a start.71.236.26.74 (talk) 03:56, 21 July 2009 (UTC)[reply]

Temperature X-Treme

From Extremes on Earth, the hottest measured air temperature was 58°C, measured in the Sahara desert. This got me thinking - on a hot day, ground or object temperatures (eg inside a car, or at the surface of an asphalt road) may be higher than the air temperature. Is this true, or is this an illusion based on the way humans perceive temperature? If it's true, I can build a contraption (I was thinking of a box made of metal painted black, and one side covered by a glass sheet, facing the sun) which would become hotter than the air temperature. Is there a theoretical limit to how hot I can make something by heating it with solar radiation alone, and about how hot would that be? — QuantumEleven 12:08, 20 July 2009 (UTC)[reply]

Yes, that is true. In theory (and unless I'm mistaken) you can make something as hot as the emitting surface of the Sun - about 6000 K, although that is technically implausible for more reasons than I can think of ;-). See Solar thermal energy. Solar box cookers reach cooking temperature without explicit concentration of solar light, large solar installation use mirrors to reach much higher temperatures. --Stephan Schulz (talk) 12:22, 20 July 2009 (UTC)[reply]
Why would the surface of the Sun be the limit? Tempshill (talk) 17:14, 20 July 2009 (UTC)[reply]
The short answer is the second law of thermodynamics. Heat does not flow (without assistance) from a cooler to a hotter body. The long answer causes me headaches ;-) --Stephan Schulz (talk) 21:06, 20 July 2009 (UTC)[reply]
Again, heat!=temperature. In theory, you could concentrate the received thermal energy into smaller and smaller masses, increasing the temperature drastically higher than 6000K. See the "solar furnace" comment below. -RunningOnBrains(talk page) 21:47, 20 July 2009 (UTC)[reply]
Our solar furnace article says that temperatures of 3000 oC have been achieved by concentrating sunlight. Gandalf61 (talk) 12:42, 20 July 2009 (UTC)[reply]
A classical thermal solar collector consists of a thin box with the bottom painted black and a glass top, tilted to face toward the equator. You run cool water in at teh top and get warm water out at the bottom. This works because the glass lets light in but does not let infrared out very well. -Arch dude (talk) 20:43, 20 July 2009 (UTC)[reply]
If you put two objects - one hot and the other cold - into an enclosed, perfectly insulated environment, they'll eventually equalize in temperature - right? If one of those objects is the sun and the other is your box - then they'd eventually become equal in temperature...if they were insulated from the environment. However, in any practical setup, your box can radiate heat off into space - heat can be conducted away by the air, etc. But if you were to perfectly insulate your box from the rest of the universe and let it heat up - it would eventually match the temperature of the sun. This happens on the sun-facing side of spacecraft - which have to be rotated to allow parts to face away from the sun and cool off. Apollo 8 span at a rate of one rotation per hour which resulted in a spacecraft that was at +200C on one side and -100C on the other! Our Solar cooker article suggests that it's fairly easy to get temperatures up to 150C with relatively simple home-made equipment. SteveBaker (talk) 01:59, 21 July 2009 (UTC)[reply]
No; equilibrium means no net energy transfer, *not* equal temperature. If you placed the sun and a box into a perfectly insulated environment (no energy loss) with no convection/conduction and all radiation exchanged (impossible in practice), the blackbody radiation energy transfer can be estimated by the equation , where A is the surface area of the object and T is the temperature. For all but ridiculously large boxes, Abox will be significantly smaller than Asun. Thus for equilibrium Tbox will necessarily be greater than Tsun. With numbers: Asun = 6.0877×1018 m2, Abox = 6 m2, Tsun = 5,778 K, Tbox = 1.83*108 K (interesting perhaps, but useless since one of the premises is faulty). Of course, practicality insists that no such perfectly reflective and insulating environment exists, the equations above are rather simplified, and (thankfully) much of the solar radiation is lost to space. – 74  04:33, 21 July 2009 (UTC) (edited 74  06:09, 22 July 2009 (UTC))[reply]
"with no convection/conduction" is an important limitation not explicitly mentioned prior to 74.137.108.115's comment here. If the only energy transfer is by radiation, then equilibrium is when the rate of loss and gain is the same, and it makes sense that loss of a black-body object would depend upon temp and surface area. However, if there's actual thermal contact (convection or conduction) I can't see a way to escape the "heat flows from warmer to cooler" basic law of thermodynamics. I don't think you can have a "one-way insulator", except you can get some similar effect if you control the rate of radiation separately from the rate of conduction/convection (as described in the solar-oven box). DMacks (talk) 07:07, 21 July 2009 (UTC)[reply]
No, convection/conduction is irrelevant - I was assuming the pure radiation case. Please not that both bodies, the sun and the box, are absorbing and radiating, and both processes are proportional to the surface area. Assuming both are black bodies, they will reach equilibrium at equal temperatures. Otherwise I could build a perpetuum mobile: Put a large and a small sphere into an isolated box, wait until the small one is warmer, open the box, use the heat difference, close the box... --Stephan Schulz (talk) 07:33, 21 July 2009 (UTC)[reply]
Eh, it works as far as the overly simplified equations are concerned. The devil is in the details, of course. Specifically, the assumption that all the radiation is exchanged. It appears I have reinvented a perpetual motion machine: the Energy Sucker. So sorry, no free lunch here. – 74  06:09, 22 July 2009 (UTC)[reply]
Limit is dependent on energy received by the Earth from the Sun. Sun's surface temperature is 6000K, but Corona is much hotter at one to three million Kelvin. By theory at least, Sun's surface temperate should not be limit. Practically archived temperature in solar furnace is 5430K, suggesting that 6000K may indeed be the limit. - manya (talk) 04:46, 21 July 2009 (UTC)[reply]
There is no fundamental reason (but there are sure practical ones) temperatures above 6000K should not be achievable by using concentrated sun light as described by gandalf. (even if we forget about the Corona for argument sake and simplicity). Arguments based on thermodynamic equilibrium neglect the fact that there is no equilibrium in the situation described. Dauto (talk) 05:46, 22 July 2009 (UTC)[reply]

Influenza A epidemic

Does it make any sense trying to brake the epidemic? At least in the Northern Hemisphere, where is summer now, wouldn't it be a better approach to let it spread, before the winter, and treat the sick?--Quest09 (talk) 16:19, 20 July 2009 (UTC)[reply]

Are you referring to the H1N1 pandemic? I do not know of an Influenza A epidemic, and did not see anything about it on the CDC website, so I suppose that it is possible that there is such an epidemic outside of the US. If you are in fact talking about the H1N1 pandemic, most medical treatment is minor, identical as far as I can tell to treatment for normal flu types. Only a small percentage of those who become infected require hospitalization. Googlemeister (talk) 16:37, 20 July 2009 (UTC)[reply]
H1N1 is a subtype of influenza A virus, as is described in our article 2009 flu pandemic. --NorwegianBlue talk 16:53, 20 July 2009 (UTC)[reply]
There is no way to break the pandemic. It is unlikely that we have any way to even reduce the spread at this point. The expected vaccine is still months away and the virus is too widespread for isolation-based control measures. Luckily, the virus is not much worse than ordinary flu, so far. Rmhermen (talk) 17:19, 20 July 2009 (UTC)[reply]
That's exactly the point. The government can't brake the pandemic, however, some governments keep trying to isolate people. Wouldn't it be easier to just let it run and concentrate on treating the ill? Quest09 (talk) 18:09, 20 July 2009 (UTC)[reply]
Even well intentioned well informed government officials are likely to take irrational measures when it comes to infectious diseases just to feel (or make the public feel) that they are doing something about it. Dauto (talk) 21:25, 20 July 2009 (UTC)[reply]
PS. same reasoning applies to the stupid security we nowadays have to go through every time we travel by plane. Dauto (talk) 21:27, 20 July 2009 (UTC)[reply]
You can blame that on the filthy terrorists who crash planes into buildings, or blow them up. I agree that (by analogy) it would be better to concentrate the security on stopping potential terrorists rather than checking everyone; however, all those evil Arab organizations and crooked "civil liberties" groups aren't letting us implement a meaningful profiling system... 76.21.37.87 (talk) 04:38, 21 July 2009 (UTC)[reply]
I don't have a problem with security. My beef, as I pointed out earlier, is with stupid security. Profiling is a member of the latter. Dauto (talk) 05:28, 22 July 2009 (UTC)[reply]
I think this is difficult to answer. While you can't break the pandemic, it doesn't mean you can't slow the spread. Whether the current measures some governments have implemented have slowed the spread is IMHO difficult to answer but I would say it's definitely possible. Slowing the spread has several advantages. 1) It avoids overloading the health system (particularly important for countries in the southern hemisphere, yes I know the OP referred to the northern hemisphere) 2) You buy time for a vaccine 3) It avoids panic 4) It will hopefully reduce the negative effect of the outbreak on the economy (many areas including tourism). Nil Einne (talk) 09:25, 21 July 2009 (UTC)[reply]

Breast implant's rejection

Why don't breast implants get rejected by the body? Quest09 (talk) 18:14, 20 July 2009 (UTC)[reply]

Because they're composed of inert silicone or saline and plastics. Transplant rejection is a function of the body's immune system attacking the biological material of the transplanted organ. Similarly, implanted metal (plates, screws, what have you) is not subject to rejection. — Lomn 18:24, 20 July 2009 (UTC)[reply]
All that stuff eventually gets encapsulated with 'stuff' (yeah, I know - my knowledge is epic), which can cause problems in its own right though, can't it? --Kurt Shaped Box (talk) 21:47, 20 July 2009 (UTC)[reply]
Scar tissue does form around the implant, which can harden to cause capsular contracture.[45] Red Act (talk) 00:27, 21 July 2009 (UTC)[reply]

Explain about Prion Diseases

I know what a prion is, but I can't find a completely understandable explanation of why prions start duplicating. Could someone please give me an understandable explanation why? Dogposter (talk) 18:20, 20 July 2009 (UTC)[reply]

For inherited prionic diseases, see PRNP and PRNP#Disease linkage. Quoting from Prion

To date, all [prions] that have been discovered propagate by transmitting a mis-folded protein state; the protein does not itself self-replicate and the process is dependent on the presence of the polypeptide in the host organism.

Intelligentsium 18:49, 20 July 2009 (UTC)[reply]

Okay, thanks! Dogposter (talk) 18:54, 20 July 2009 (UTC)[reply]

Prions are autocatalytic. That is, the presense of a prion in the cellular interior stimulates the formation of additional prions from the precursor unfolde proteins that would otherwise form the normally-folded form of the molecule. -Arch dude (talk) 20:33, 20 July 2009 (UTC)[reply]
That sounds really scary! Is there any way that the immune system can deal with all them prions? Also, is there any treatment that's effective against them? 76.21.37.87 (talk) 04:41, 21 July 2009 (UTC)[reply]
No, prion diseases are currently untreatable and universally fatal, though it may take many years for symptoms to develop in some cases. Dragons flight (talk) 04:46, 21 July 2009 (UTC)[reply]
Mein Gott! 76.21.37.87 (talk) 05:25, 21 July 2009 (UTC)[reply]
May be that phrase "Although CJD is the most common human prion disease, it is still rare, occurring in about one out of every one million people every year" taken from the Creutzfeld-Jakob disease article will offer you some confort... Dauto (talk) 17:10, 21 July 2009 (UTC)[reply]
Good, now if we can just keep it that way... 76.21.37.87 (talk) 00:31, 22 July 2009 (UTC)[reply]

So basically your body thinks that since a prion is there, it must be correct? Dogposter (talk) 13:50, 21 July 2009 (UTC)[reply]

That's one way to look at this. Or, you can picture a prion's action in the body as the prion binding to some protein and twisting it another prion like itself, which then goes on to twist another protein into a prion, and so on... (that's the way I think of this). FWiW 76.21.37.87 (talk) 00:30, 22 July 2009 (UTC)[reply]

Rabies vaccine

Why is the rabies vaccine not given routinely, as are the DPT vaccine, Varicella vaccine, etc.? I understand it is extremely unlikely that any one person will contract rabies, but upon contracting rabies, it is very unlikely that the person will survive without prior or immediate vaccination, and many rabies victims are unaware that they have been infected until it is too late. Is it not better to err on the side of caution? Intelligentsium 19:08, 20 July 2009 (UTC)[reply]

I do not know if it is applicable to the rabies vaccine, but most vaccines have a very small probability of giving the disease. Even if that rate is 1 in 1,000,000 it would probably cause more rabies deaths then it would prevent. It is also not free to make or distribute the vaccine. Googlemeister (talk) 19:22, 20 July 2009 (UTC)[reply]
There are many possible reasons, but I expect the most significant are directly related to just how unlikely rabies is. The cost of vaccinating a population is quite high, in terms of dollars, resources, and possible consequences of side effects. The US sees approximately two cases of rabies in humans per year. By contrast, there are approximately four million births in the US annually. A "one in a million" chance of complications from a vaccine would double the number of incidents, and do so in a segment of the population most susceptible to illness (the young). Additionally, our article on the vaccine notes that, even vaccinated, humans require treatment after a bite. Put together, world health authorities have concluded that this is an ineffective way to combat rabies. Rather, the US, Canada, Europe, and others are primarily combating rabies via oral vaccines of carrier populations, which seems to be more effective at far less cost. — Lomn 19:24, 20 July 2009 (UTC)[reply]
(EC) Speculation: except for tetanus, the diseases you listed are contagious, which rabies isn't really (not enough people biting people to support an outbreak). Vaccination policy's cost-benefit analysis probably has more to do with preventing epidemics than protecting individuals from rare diseases. --Sean 19:27, 20 July 2009 (UTC)[reply]
My wife received the vaccine, and experienced side effects that were not dangerous, but were unpleasant (pain and soreness). So combine that with the reasons others have given, and it's kind of a hard sell if you're not in a high-risk group. -- Coneslayer (talk) 19:35, 20 July 2009 (UTC)[reply]
In my understanding, routine vaccinations are highly effective for contagious diseases (smallpox, measles, polio, diphtheria, etc.) because they not only protect the person being vaccinated, but also the others, by reducing the incidence of the disease. Rabies disease, however, have a huge animal carrier pool, but is extremely seldom transmitted between humans; so vaccinating a person only protects that particular person. Vaccinating the animals that may infect each-other as well as humans makes more sense, and is indeed carried out routinely in some countries. AFAIR there is a bait (oral) vaccine for foxes and jackals available, but there well may be other types, too. --Dr Dima (talk) 20:02, 20 July 2009 (UTC)[reply]
Right, that's the catch. You vaccinate the domestic dogs and cats, not the humans. That probably takes care of 90% of potential cases; humans aren't bitten by wild animals all that often in the Western world. (I got stung by a bee last year, I think that was as close as I've come in a long time!) --98.217.14.211 (talk) 23:02, 20 July 2009 (UTC)[reply]
I don't know if this is still the case but the vaccine used to be delivered by injection into the stomach muscles - a very painful procedure - this would decrease it's uptake as well. Exxolon (talk) 01:41, 21 July 2009 (UTC)[reply]
I think that it was the rabies *treatment* that they used to inject into the abdominal muscles. Several times over several days. --Kurt Shaped Box (talk) 02:28, 21 July 2009 (UTC)[reply]
As I understand it, the vaccine is the treatment. The incubation period of rabies is long enough that the patient has time to build up a protective immune response before the virus causes symptoms. Though I wouldn't be surprised if a more aggressive vaccination is used for patients who are believed to have been exposed, than for those who are simply at risk to be exposed at some later time. --Trovatore (talk) 03:10, 21 July 2009 (UTC)[reply]

Earth's Net Charge

I learned in my physics class that Earth has a net negative charge to the tune of about 500,000 C. I found this interesting since we were also discussing how you can generate a charge by friction (ie. combing your hair) and the idea occurred to me that maybe the negative charge came from the hypothetical collision between the Earth and Theia, creating the moon. So basically my question is, has the net charge of the Moon ever been measured? If not, is it possible to? Would be kind of cool if it turned out to be net positive. And also if I'm overlooking a really obvious reason for the Earth's net negative charge, please let me know. Thanks in advance. Ginogrz (talk) 22:48, 20 July 2009 (UTC)[reply]

I think you’re misunderstanding the 500kC charge. The conducting layers of the ionosphere and the earth’s surface essentially form a capacitor, that’s charged by about 500kC.[46] That’s presumably just due to atmospheric effects. But that doesn’t mean that the planet as a whole, including the ionosphere, has a net charge. I would think that a planetary net charge like that would be extremely difficult to determine, since we have no way of measuring charge density below the earth’s crust. Red Act (talk) 00:56, 21 July 2009 (UTC)[reply]
Google tells me that Earth has a net charge of near zero because the solid earth has a charge of -500,000 C and the atmosphere has a charge of about 500,000C. As to your charge about the moon I don't know but in it's doubtfully very large- most things in such as planets aren't so charged I thought. In general strong nuclear force holds together atoms(i.e holds together the protons in the nucleus where the electric repulsion trumps the gravitational attraction by many orders of magnitude), electric force holds together things on a microscopic scale (compare the relative strengths of the electric field and gravitational field between two electrons) and on a macroscopic scale (i.e the universe) its gravity that holds things together because the net electric charge on objects is so much less then the mass of the objects. Most objects aren't like an electron with a charge that is appreciable as compared to the mass. Computeridiot34 (talk) 01:00, 21 July 2009 (UTC)[reply]
It looks like whether the earth has a net electric charge is a matter of scientific debate. See this and this. Red Act (talk) 01:14, 21 July 2009 (UTC)[reply]
The question of the earth’s net electric charge was discussed on this reference desk a few months ago.[47] Red Act (talk) 01:18, 21 July 2009 (UTC)[reply]
Yeah, I most certainly was misunderstanding it. The capacitor scenario certainly makes a lot more sense than a massive body having a significant net charge, as Computeridiot34 pointed out. Some interesting reads as well, I didn't know it was such a debated topic, but such is science. Thanks. Ginogrz (talk) 01:58, 21 July 2009 (UTC)[reply]

North and Suth America

Is there any evidence N and S America is currntly moving north? this site is tough to folow along. To what direction is Antartica plate moving. The slide shows, the ice (in white) will melt green in about 50 million years.--69.228.145.50 (talk) 22:56, 20 July 2009 (UTC)[reply]

plate motions. Dragons flight (talk) 01:16, 21 July 2009 (UTC)[reply]
No - to the contrary. A westerly or south-westerly trend seems more likely. Our article North American Plate says "For the most part, the North American Plate moves in roughly a southwest direction away from the Mid-Atlantic Ridge." but South American Plate is not so helpful. It kinda suggests a westerly motion since the ocean floor is spreading to the east of the continent - and it's being subducted to the west. The Antarctic Plate article says that it's moving in the direction of the Atlantic ocean...which is in a northerly direction...but then ANY motion of the antarctic plate could be described as "northerly"! SteveBaker (talk) 01:40, 21 July 2009 (UTC)[reply]

July 21

Why don't birds ever get rabies?

Question as topic. Is it the case that their immune systems have no problem destroying the virus as soon as it is detected? --Kurt Shaped Box (talk) 02:42, 21 July 2009 (UTC)[reply]

I doubt it. A lot of viruses are very specific as to what kind of cells they can infect. If I recall correctly, the rabies virus can infect only mammalian nerve cells.
Think of it as, you can't run a Windows virus on a Linux box (usually, anyway). --Trovatore (talk) 03:04, 21 July 2009 (UTC)[reply]
Technical answer: The best MEDLINE hit I found (Seganti et al. "Susceptibility of mammalian, avian, fish, and mosquito cell lines to rabies virus infection." Acta Virologica 1990 Apr; Vol. 34 (2), pp. 155-63.) says the rabies virus can bind to bird cells (an early step in infection), but subsequent "events following virus adsorption" inside the cell nix the infection. "Events following virus adsorption" glosses over a ton of stuff. - Draeco (talk) 03:41, 21 July 2009 (UTC)[reply]
Birds can definitely develop rabies infection, although they are not nearly as susceptible as mammals. This paper: [48] is very interesting. It is also rather surprising. I cannot possibly imagine why 5 out of 125 starlings tested had positive rabies titres... I mean it would be mind-boggling to imagine that 4% (or more) of starlings in the wild were at some point bitten by a rabid predator and survived. Returning to the question of bird susceptibility to rabies virus, chick embryo cells are used to make some types of rabies vaccine, and, if I remember right, duck embryo cells are or have been used at some point, too; so they are definitely susceptible at least at that developmental stage. --Dr Dima (talk) 06:29, 21 July 2009 (UTC)[reply]
Interesting, thanks. It would seem that the paper you found actually supports my immune system suggestion too. --Kurt Shaped Box (talk) 08:12, 21 July 2009 (UTC)[reply]
As for why Starlings might test positive for rabies - well, it has been known for them to scavenge a bit too. I don't know how well the virus survives in carrion though. --Kurt Shaped Box (talk) 08:12, 21 July 2009 (UTC)[reply]
Hmm, I've seen them go through garbage, but I've never seen them feed on carrion. Do they do that? That can explain the positive titres alright. Or it may have been bad or misread titres :) . Anyways, to your question on how well (or how long) the virus survives after the death of the host - it depends. In some cases the virus is no longer detectable at the time of death (immune system eliminated it, but the damage has been done); in others it is present and persists for quite some time. If it is hot outside, rabies virus will persist for hours or days AFAIK. However, if it is freezing, it may persist for much longer. Rabies virus survives freezing. --Dr Dima (talk) 17:13, 21 July 2009 (UTC)[reply]
FWIW, I've (very) occasionally seen Starlings pecking at roadkilled <whatever> and the odd pigeon carcass. Not to any great extent, mind - but I suppose that if it's there, split open, available and they're hungry... --Kurt Shaped Box (talk) 23:03, 21 July 2009 (UTC)[reply]
OK, that may explain the positive titres. --Dr Dima (talk) 23:55, 21 July 2009 (UTC)[reply]
Wow, no shortage of original research today on the Science RefDesk! --Scray (talk) 01:07, 22 July 2009 (UTC)[reply]

Is Attachment Resistance Disorder the same as Reactive Attachment Disorder?

Someone recently told me a friend had "attachment resistance disorder," but a Google search turned up nothing. Would this be another way to say RAD, as above, or some other general attachment disorder?209.244.30.221 (talk) 09:24, 21 July 2009 (UTC)[reply]

Yeah, it sure looks like the person who used the term "attachment resistance disorder" just misspoke. It's impossible to know for sure from the information given, but "reactive attachment disorder" does seem like a good guess as to what the person meant to say. Red Act (talk) 14:39, 21 July 2009 (UTC)[reply]
Thanks; it makes sense. I was thikning, too, my friend may have have made a logical leap; whatever one of those substitutions is called. They may have thought, "It's attachment and something beginning with 'R'...oh, as children people with attachment disorder resist attachment, so i bet it's 'resistance.'" (The person referred to was an adult, but it's still a logical step.)209.244.30.221 (talk) 11:57, 22 July 2009 (UTC)[reply]

Flash memory and chargeable devices

In case of flash memory, I undertood that a charge is trapped during the write data status, and then a relatively high voltage has to be applied in reverse polarity in case of erasing (in order to release this trapped charge). I was wondering how much charge in coulombs can be trapped in a 1 GB flash memory in maximum, and if it were possible to trap large amounts of charge to utilize later as a battery (i.e. battery flash technology)?--Email4mobile (talk) 12:21, 21 July 2009 (UTC)[reply]

You just basically want a capacitor; there's no value in using something as fancy as flash memory for this. Looie496 (talk) 16:48, 21 July 2009 (UTC)[reply]
True. The total gate capacitance of a flash memory is tiny. As an example, a 3-volt 4 Mbit NOR flash chip from 2006 had a gate charge of 10^3 to 10^5 electrons [49]. That gives a maximum charge of 64 nC, or a capacitance of 21 nF. Scaling that up to 1 Gbit and ignoring the likely drop in the charge per gate, that still gives only 5.5 uF, which wouldn't power your iPod for very long. --Heron (talk) 18:12, 21 July 2009 (UTC)[reply]
Thank you very much for clarifying this information to me :).--Email4mobile (talk) 18:44, 21 July 2009 (UTC)[reply]

Two identical piece of tachyon stuff hitting each other

If you have one piece stuff traveling at half the speed of light in one direction and another identical piece of stuff traveling at the same speed in the other direction. When they smash into each other, they can stick together and come to a standstill (aka velocity of zero).

But what happens when one piece of tachyon stuff traveling at twice the speed of light, hits an identical tachyon piece of stuff traveling at the same speed but in the other direction. Do they stick together and come to a standstill? 122.107.207.98 (talk) 12:54, 21 July 2009 (UTC)[reply]

How can a tachyon exceed the speed of light? Googlemeister (talk) 13:10, 21 July 2009 (UTC)[reply]
Read Tachyon. I'm no expert, but its given that they can't travel at subluminal speeds. Rkr1991 (talk) 13:16, 21 July 2009 (UTC)[reply]
Well, given that they are hypothetical, I don't think this has a useful answer. But one question comes to mind: Would they even interact? Maybe they just fly straight through each other... --Stephan Schulz (talk) 13:26, 21 July 2009 (UTC)[reply]
Tachyons don't exist - hence there is no answer to this question. Even if they did exist, we'd not have done any science on them so we wouldn't be able to answer the question anyway. It's like asking whether the tooth fairy gives Santa's reindeer a quarter whenever they lose a tooth. SteveBaker (talk) 13:33, 21 July 2009 (UTC)[reply]
(rolls eyes) Lord, if theoretical physicists acted like the fellows on the Ref Desk do ("we can't possibly consider what would happen, even in hypothetical situations!"), we'd never get anywhere. I don't know the answer to this, but I'm sure there are theories on interactions between tachyons and other particles, including tachyons. Google "tachyon-tachkyon interaction" and you get some hits. Obviously they are speculative but lord, that's how theoretical science starts out—try to figure out what might happen, then see if there's any way to distinguish between the many possibilities. That being said, treating tachyons like baseballs is probably not correct (and baseballs don't usually just stop with a velocity of zero; they scatter). My weak, weak, weak understanding of special relativity makes me wonder whether or not you could have strictly simultaneous collisions with objects traveling at superluminal speeds (surely that runs into some sort of causality violation, no?). --98.217.14.211 (talk) 15:29, 21 July 2009 (UTC)[reply]
But that's precisely WHY ref desk replies are not like that. We're not here to generate new theories. Our job is to answer questions with known facts. The FACT is that as far as current knowledge and respectable theory goes - tachyons do not exist - nor is there any reason for them to need to exist in order to explain any real-world phenomenon - and if they did exist, then according to everything we DO know, their speed would have to be infinite and their mass and size would be a complex number - that's nonsensical even by the standards of Quantum theory! The theory that they MIGHT exist is unfalsifiable - and therefore no more worthy of consideration than Russel's teapot. Occam's razor says "Ignore this theory unless/until there is some evidence." Therefore, neither you nor anyone else can say with any authority whatever what would happen if two of them collided. So why answer the question when neither you nor anyone else have any clue whatever what the answer might be? The best service we can give to our OP is to say that these "Tachyon" things don't exist. SteveBaker (talk) 20:50, 21 July 2009 (UTC)[reply]
Positrons couldn't exist (until they were discovered, of course) because it was obvious that they could not exist because there is no way for positive protons to lose mass nor for electrons to change sign. What I'm saying is: even hypothetical things have a right of their own until the theory (any theory) predicting them has been proved wrong by evidence, not only by deficiency of current theories. If the article is not totally wrong, we do not know if tachyons exist or not. 93.132.129.137 (talk) 22:57, 21 July 2009 (UTC)[reply]
So, you're saying that the tooth fairy does give the quarter to Santa's reindeer? Tempshill (talk) 17:44, 21 July 2009 (UTC)[reply]
As I have explained before, my wife is in fact, the actual Tooth Fairy - so I asked her. She said that if she had to pay out for animals, the sharks would bankrupt her in no time. Plus the rules clearly state that teeth must be placed under a pillow. SteveBaker (talk) 20:41, 21 July 2009 (UTC)[reply]
You should know that there is not only one tooth fairy. Others probably have different contracts. 93.132.129.137 (talk) 23:14, 21 July 2009 (UTC)[reply]
Within the framework of special relativity, the answer to the question is no. A tachyon must be a fundamental particle. It cannot be a composite particle, because all known binding forces propagate only at the speed of light and hence the tachyon would literally outrun the forces trying to hold it together. Colliding fundamental particles do not stick together; they either scatter or annihilate. My guess is that the latter would be more likely for colliding tachyons, but I don't really know. Dragons flight (talk) 01:29, 22 July 2009 (UTC)[reply]
Except that there might exist a kind of force we don't know about yet that propagates faster than the speed of light. That shows that there really isn't a point in taking about such things as steve correctly pointed out. Read about positronium for an example of how two colliding fundamental particles might stick together. Dauto (talk) 05:19, 22 July 2009 (UTC)[reply]
You can't form positronium in a high energy collision. Dragons flight (talk) 09:24, 22 July 2009 (UTC)[reply]
The collision doesn't have to be high energy and even if it is, the excess energy can be shed as photons. Dauto (talk) 14:36, 22 July 2009 (UTC)[reply]
Conservation of energy-momentum applies to tachyons, as does the relation E² − |p|² = m² (where m is the rest mass). For a tachyon, by definition, m² < 0, so |p| > |E|, but for two tachyons the momenta can cancel out while the energies add, as in your example, so a bound system of two (or more) tachyons might behave like a tardyon with ordinary mass. As for large bound systems of tachyons that move faster than light ("tachyon stuff"), it's not clear to me whether they could exist. It would depend on how causality works in this universe with tachyons.
It's worth mentioning that "tachyon" doesn't really mean "faster than light" in modern physics, it means "m² < 0", and it's not clear that that actually has anything to do with faster than light motion. See this Physics FAQ page. And there's a phenomenon called tachyon condensation whereby tachyonic fields end up behaving like normal massive/massless fields anyway. This is supposed to have happened to the Higgs field in the Standard Model. -- BenRG (talk) 11:57, 22 July 2009 (UTC)[reply]

Common House Plants

Which common house plant is very sensitive to over-watering? I would think cactuses are the least? --Reticuli88 (talk) 15:04, 21 July 2009 (UTC)[reply]

Least? My understanding is that cactuses are the most sensitive to over-watering. Looie496 (talk) 16:41, 21 July 2009 (UTC)[reply]
Cacti will rot if they get too much water. Googlemeister (talk) 18:06, 21 July 2009 (UTC)[reply]
Most house-plants are sensitive to overwatering. Cacti are quite sensitive, as Looie and Meister have already pointed out. Some orchids will deteriorate and eventually die (if not treated promptly) after as little as one or two overwatering incidents; I have lost a couple of orchids this way, too. On the other end of the spectrum, Pinguicula can tolerate overwatering well, and Aglaonema will tolerate being overwatered once in a while. --Dr Dima (talk) 19:39, 21 July 2009 (UTC)[reply]

Medical/Scientific Term for Non-Aging

Is there a medical/scientific term for someone who definitely does not look their age? Reason is, I went to see my grandfather up in Northern Canada for the first time a while ago and no one pointed him out to me at first. Then this man approached me looking like in his late 50's. He had white hair but smooth skin, barely any wrinkles, he had all his hair and all his teeth. He smiled (with his very yellow teeth; he smoked Malboro Reds) and said he was my grandfather. My jaw dropped and realized that he was pushing ninety. Then I realized I have his genes because I am always mistakened for a much younger age, so is my sister. Is there a term for this? --Reticuli88 (talk) 15:11, 21 July 2009 (UTC)[reply]

Youthful, or neotenic if you prefer sesquipedalian pretentiousness. alteripse (talk) 15:14, 21 July 2009 (UTC)[reply]

"Sesquipedalian"? Ironically, "neotenic", having only 4 syllables, is not sesquipedalian, but the last 2 words of your response are. <:-p --Scray (talk) 01:04, 22 July 2009 (UTC)[reply]
hyposenescence? It's not in the dictionary, but it seems like a perfectly cromulent neologism to me. Some jerk on the Internet (talk) 17:49, 21 July 2009 (UTC)[reply]
ageria, perhaps? Another neologism, but it seems reasonable to me. --Scray (talk) 00:59, 22 July 2009 (UTC)[reply]
The formulaic expression when noting this in medical records is "appears younger than his/her stated age". So, no, no medicalese is involved. Sorry! - Nunh-huh 09:43, 22 July 2009 (UTC)[reply]
And just for the record, Reticuli88: No need to rub it in. You, too, can be hit by a bus tomorrow ;-) --Stephan Schulz (talk) 15:21, 22 July 2009 (UTC)[reply]
All in all, age appearance is nothing more than a cultural attribute. Some identifiers of age in one culture are not identifiers of age in another. In the United States, there are many different ethnicities, races, and cultures. So, it is possible to have rather odd situations such as Brad Pitt (b. 1963) being considered to young for Julia Roberts (b. 1967) in "The Mexican". -- kainaw 15:36, 22 July 2009 (UTC)[reply]

Gravity dependence

Hi!! I would like to know if gravity was dependent on mass or volume? I believe it is directly proportional to mass and inversely proportional to volume. Please help me out. Thanks!!--Lightfreak (talk) 18:31, 21 July 2009 (UTC)[reply]

Using Newton's law of universal gravitation (a useful everyday abstraction), the strength of a gravitational field is directly proportional to product of the masses of the two objects and inversely proportional to the square of distance between them. Volume is irrelevant, except as it often provides a minimum bound for distance. — Lomn 18:40, 21 July 2009 (UTC)[reply]
What Lomn said, except that in cases where you might know the density and volume but not the mass you can calculate mass from the other two. Not a bad way to estimate how much a planet weighs actually if you can get a good estimate of the density of the whole. Googlemeister (talk) 20:23, 21 July 2009 (UTC)[reply]
The physical property you’re thinking of, which is directly proportional to mass and inversely proportional to volume, is density. As Lomn points out, density isn’t what’s important for gravity, just mass (at least for Newtonian gravity). Red Act (talk) 20:21, 21 July 2009 (UTC)[reply]
Certainly, the force due to gravity depends on mass, not volume - but you also have to realise that the gravitational force of (say) our sun - and that of a black hole that's the same mass as our sun does (in a sense) depend on their volume since you can only get to within 700,000km of the sun without crashing into it's surface - where you can get as close as you like to a black hole because it has zero size (or if you prefer - you can get to within 3km of it because that's where it's event horizon is). The point being that both the sun and the black hole have the same mass - so if you are (say) a million kilometers away from the center of each - the force due to their gravities would be the same too. The black hole only has this crazy amount of gravity because it's small size lets you get very close to it's center. SteveBaker (talk) 20:37, 21 July 2009 (UTC)[reply]
You mean ITS CENTER. Steve remember IT'S an encyclopedia with ITS standard of correct grammar [50]. Cuddlyable3 (talk) 22:16, 21 July 2009 (UTC)[reply]
Good point. To refer to Newton's law again, it depends both on mass and distance. EverGreg (talk) 08:05, 22 July 2009 (UTC)[reply]
Actually, thinking about it, depending on how you define what it means for gravity to be proportional to something, saying that gravity is proportional to density can be viewed as not being far off. In general relativity, the Einstein tensor is proportional to the stress-energy tensor, which is essentially a relativistic generalization of density. So if you look at things from the right perspective (the weak field, low speed limit as expressed by the Einstein field equations), the OP can be viewed as being arbitrarily close to being correct. Red Act (talk) 10:01, 22 July 2009 (UTC)[reply]
True - but I think it's important that we straighten out our OP's confusion rather than deepening it. The clearest possible answer is the first one we gave. The mass matters - the volume does not. Period. SteveBaker (talk) 15:05, 22 July 2009 (UTC)[reply]

What kind of intelligence do mathematics require?

My best friend has always been intelectually gifted. I do not know his exact IQ, but I do know that professional, psychologist-supervised tests have shown that, whereas his memory and spatial intelligence are more or less average, his logical abilities are in the 99.999th percentile. He really excels in all fields of knowledge except one: he is quite bad at mathematics and never quite manages to grasp mathematical concepts and formulae, even if he tries hard.

I am wondering why he is so bad with mathematics if his logic is so good - I thought mathematical reasoning was closely tied to logic. Moreover, he is very good at other fields requiring logic. What explaination can you give me for this "paradox"? --81.38.156.193 (talk) 19:42, 21 July 2009 (UTC)[reply]

Do you honestly expect random users on the Internet to diagnose your friend? Learning is not simple. It is complicated. Perhaps your friend simply is not interested in math. My brother claims to have every learning disability and brain defect ever discovered to cover up the fact that he simply doesn't want to read (when forced, he reads just fine, but complains about it). -- kainaw 19:49, 21 July 2009 (UTC)[reply]
Sorry, I'm not buying the premise. There is no formal test, as far as I know, that distinguishes logical abilities from spatial intelligence and mathematics. Do you mean verbal abilities, perhaps? Looie496 (talk) 19:52, 21 July 2009 (UTC)[reply]
This does seem a little screwy. Logic IS mathematics (it's both how mathematics is done - and a sub-field of math in it's own right). Hence your friend is not bad at "mathematics" - he's bad at some branch or branches of mathematics...perhaps he is poor at arithmetic or calculus but great at logic and geometry maybe. Until people are exposed to higher mathematics - it's easy to go away with the impression that arithmetic IS mathematics because almost every educational system on the planet teaches arithmetic before (for example) Algebray. Having a relatively poor memory would be a handicap for arithmetic - where a lot of what gets you fast and accurate is having little "lookup-tables" memorized - the multiplication tables for example. Similarly, calculus requires you to memorize a pile of standard derivatives and integrals and such. Applied mathematics (ballistics, rigid body motion, etc) also demands that you memorize a whole bunch of equations. But logic requires little memorization - and neither does geometry (well, to a point). It might also matter whether one has a 'visual' memory or a 'verbal' one. But most of mathematics is easier if you can memorize the results achieved by others rather than starting with a 'logic' proof from scratch every time. But I'm confused. If his memory is only 'average' - how does he excel at (for example) geography and history - which are almost entirely memorization exercises? How does he do well at physics - when so much of it is mathematics? I doubt very much that these "tests" are really what your friend believes them to be. SteveBaker (talk) 20:27, 21 July 2009 (UTC)[reply]
Well, I am sorry, Looie 496, but I saw the psychologist's report and he put it quite clearly:
  • memory: 101
  • logical abilities: 172
  • spatial coordination: 96 (all on a scale on which 100 is average, <85 is deficient and >137 is intelectually gifted).
My friend has been trying hard to learn maths for his degree, so it's not that he's trying to cover up for failure. As far as I can tell, he is genuinely trying to improve in maths (I'm quite good in maths myself and I'm giving him extra lessons, in which he works hard). Finally, I am not asking anyone ("random users on the Internet") to diagnose my friend - I'm just asking if logic abilities and maths are really intrinsically related.
@ Steve Baker: thanks very much for your answer, I find it very interesting. As for history and geography: he does indeed seem to memorise things very easily, and he always remembers the important facts and figures. He's also capable of memorising a huge amount of information in very little time. In fact, I always thought he had a gifted memory as well, until he showed me the test results to be average, so I really don't know what to say about that. As for physics: he is very, very good at understanding the concepts of physics (knowing why things happen in physics) but he is lousy at doing the practical part - good God, he's not even capable of calculating a derivative! But I guess this will remain an enigma. --81.38.156.193 (talk) 20:37, 21 July 2009 (UTC)[reply]
The rather standard Stanford-Binet Intelligence Scales separately test “fluid reasoning”, “quantitative reasoning”, “visual-spatial processing”, and “working memory”. I could see a psychologist paraphrasing those categories a little if the psychologist typed up a letter regarding the result. It seems plausible to me that someone who scores extremely high at fluid reasoning might score only average at quantitative reasoning. People who are geniuses at one thing are quite commonly only average or even “retarded” at other things. Like a lot of people with Asperger syndrome, for example.
I think the 99.999 probably just means that no questions were missed in that section, and shouldn’t be interpreted to literally mean 99.999. Standard intelligence tests like Stanford-Binet can’t distinguish between very high intelligence levels very well. Red Act (talk) 21:02, 21 July 2009 (UTC)[reply]
Yeah - I flat out don't believe the 99.999% thing - those tests aren't remotely sensitive at the very top end because there just aren't enough people in the general population who are smart enough to pass the tests at that level. Also, there is a phenomena that Mensa have noticed which is that super-intelligent people who are asked the usual "Which of these four diagrams is the odd one out?" kind of question can find obscure reasons why any one of them might be considered wrong and have to resort to trying to guess which of those reasons was in the mind of the person who created the test. There is a very real sense in which the super-intelligent are smarter than the people creating the tests.
But this case seems strange to me. We're told that the test says that memory is strictly average - yet our OP is struck by how the person is exceptionally good at 'rote-learning' subjects like history. We're told that the person has a high score on logic - yet the one topic that truly exemplifies logical thinking is mathematics - and this person is allegedly useless at math. Seems to me that the results of the test are just crap - they bear no resemblance to reality! SteveBaker (talk) 01:32, 22 July 2009 (UTC)[reply]
Yeah, I think it probably wasn’t really “logical ability” that that score was measuring, but “fluid reasoning” as Stanford-Binet calls it, a.k.a. fluid intelligence. The two tests that a psychologist would be most likely to give are Stanford-Binet and Wechsler, and the intelligence categories given above match up well with the categories in Stanford-Binet, and don’t match up well at all with Wechsler. I think the psychologist just paraphrased “fluid reasoning” as “logical ability” when giving the results, because there is a relationship between fluid reasoning and logical ability, and a lot more people would understand the phrase “logical ability” than the phrase “fluid reasoning”. And it’d be really easy to score well on a fluid reasoning test but not get good grades in math. A test of fluid reasoning very intentionally avoids relying on previous knowledge of a topic, so it’s very much a “shallow” test of “innate” intelligence, if you will. Math is very different in that regard – math is very deep, with each math course relying on an understanding of the material presented in the last. If you miss out on a good understanding in one course for any reason, that can really mess you up at all the higher levels.
However, even if it really was logical ability that was being measured, one possible way that a person might have extremely good logical ability but be bad at math is if the person has dyslexia. You can be great at thinking logically with dyslexia, but a big part of math is symbolic manipulation, and all that symbolic manipulation could be quite a problem if you have dyslexia. I’m not trying to make a diagnosis here, I’m just using the possibility of dyslexia as an example. Because, as I said, I think it probably wasn’t actually “logical ability” that was being measured. Red Act (talk) 03:01, 22 July 2009 (UTC)[reply]
Someone who was great at rote memorization and at logic would have done great in calculus, differential equations, and matrix algebra, some difficult math courses I had to take. If they had trouble with details, or tended to reverse digits, somewhat like a dyslexic, they would have come up with incorrect answers and gotten low scores. Inability to do detail work under time pressure or "exam phobia" could have crippling results. There are many component abilities necessary to do well in college math(s). I tended to approach the study of math in a verbal and logical way and it often ended badly. It is necessary, after all, to come up with the correct symbolic or numeric answer to the exam question. Reasoning and logic count for little. Edison (talk) 04:34, 22 July 2009 (UTC)[reply]
I think your friend may have dyscalculia, which is the maths equivalent of dyslexia. I'm in much the same position: apparently I have an IQ that puts me in the top 1% of the population, and my verbal skills are excellent, but my arithmetic skills are hopeless and always have been. I didn't get diagnosed with this condition until I was 38 and training to teach adults with dyslexia. I am fascinated by maths, especially the more complex stuff, but as I can't add/subtract/multiply/divide, I fail exams unless I can use a calculator. If he can get this diagnosis, he may be able to get special support from his university. --88.108.188.105 (talk) 15:46, 22 July 2009 (UTC)[reply]

Infinite amount of mass in the universe?

If the universe is infinite, does it also contain an infinite amount of mass? Does it contain an infinite number of objects such as stars and planets? If yes, where does all that mass keep coming from? Is it even possible that new objects just keep on coming on no matter how far you travel? And if no, then where do the objects stop? And wouldn't they then be contained inside a finite subsection of space, making it effectively the centre of the universe? JIP | Talk 20:45, 21 July 2009 (UTC)[reply]

If the universe is infinite, then yes, it would probably contain an infinite amount of mass. We do have some evidence from WMAP that the universe is not infinite, though. It's most likely finite, but unbounded, just like the surface of a balloon. Which point on the surface of the balloon is the center of that surface? --Stephan Schulz (talk) 20:50, 21 July 2009 (UTC)[reply]
So the size, and amount of mass, of the universe is finite, but it does not have any centre or boundaries? JIP | Talk 20:55, 21 July 2009 (UTC)[reply]
Yes, that's our current best guess (for limited values of "current" - I only follow this in PopSci sources). --Stephan Schulz (talk) 21:07, 21 July 2009 (UTC)[reply]
It looks like we need Stephen Hawking. Is Stephen Hawking in the house? Bus stop (talk) 21:12, 21 July 2009 (UTC)[reply]
There are always a few with a minority opinion, of course. You might be interested in the shape of the universe article. Red Act (talk) 21:19, 21 July 2009 (UTC)[reply]
Sorry for barging in with a [probably quite lame] question, but if the universe is like a balloon, what is on the other side of the balloon's surface? --Leptictidium (mt) 21:52, 21 July 2009 (UTC)[reply]
There isn’t another side. The balloon analogy is inexact. To make the analogy fit better, you’re supposed to think of the balloon as being a 2-D manifold, with no thickness in a third dimension. Red Act (talk) 23:20, 21 July 2009 (UTC)[reply]
Another way to think about it is that you could head off in your super-fast space ship away from earth IN ANY DIRECTION, keep going in a straight line - and then arrive back to exactly the place you started. The universe would then be finite - yet have no edges. However, since the universe appears to be bigger than the visible universe - and it's expanding fast - we could never actually take that trip because we can't ever travel faster than light and so we can't overtake the expansion of the universe. SteveBaker (talk) 23:56, 21 July 2009 (UTC)[reply]
I don't think there's any evidence from WMAP that the universe is spatially finite. The evidence is that the spatial curvature is roughly zero with some error margin, so it could be slightly positive or slightly negative. There's not a whole lot you can conclude from that. Even if the curvature was narrowed down to the positive side of zero, you can't conclude that the whole universe is a sphere with the radius implied by the local curvature; it might just be local fluctuation (where "local" means 10–100 billion light years or more). Also, even if the spatial curvature is positive and constant, you can't circumnavigate space if ΛCDM is correct (which it might not be). According to ΛCDM you can never reach anything that's farther than a present-day distance of around 16 billion light years, because of the runaway expansion, and the universe is certainly larger than that. -- BenRG (talk) 13:17, 22 July 2009 (UTC)[reply]
I'm not sure that an assumption that an infinite universe means infinite mass, actually. Or infinite planets. One could imagine an almost-infinite emptiness with pockets of mass. A lot of mass, sure, but far from infinite. Anyway, as far as we can tell, it isn't infinite (nothing is, in such a case). Infinity is a tricky concept to apply to the real world. --98.217.14.211 (talk) 23:43, 21 July 2009 (UTC)[reply]
If the universe is infinite, does it also contain an infinite amount of mass? - Almost certainly, yes.
Does it contain an infinite number of objects such as stars and planets? - Again, almost certainly.
If yes, where does all that mass keep coming from? - It would all have been contained in singularity at the start of the big bang.
Is it even possible that new objects just keep on coming on no matter how far you travel? - If the universe is infinite, yes.
...but all of these things are only true if the universe is infinite...it may well not be. SteveBaker (talk) 23:56, 21 July 2009 (UTC)[reply]
… and “it may well not be” is a major understatement. Very few active researchers nowadays are considering the possibility that the universe might be infinite in size, or contain an infinite mass. That idea pretty much died out in the late 60’s, after the discovery of the cosmic microwave background radiation in 1965 cemented the big bang theory as the mainstream viewpoint in physical cosmology. There was some brief interest in exploring alternative cosmologies again in the late 70’s and mid 90’s, when certain observations cast some doubt on the big bang theory briefly. But in each of those cases, minor tweaks to standard cosmology resolved the issues within a few years.
The statement above that an infinite sized universe with infinite mass would have arisen from a big bang is highly problematic. Infinite size and infinite mass only make sense with something like the steady state theory of cosmology. If there was a big bang, then the universe must have a finite age (the amount of time since the big bang), and hence must have a finite size (or it would have had to expand infinitely fast). And if the universe has a finite size, then it must contain a finite mass. Infinite mass in a finite volume would be inconsistent with general relativity, since a black hole with infinite mass would have to have an infinitely large radius. As long as the big bang theory is correct, which very few serious cosmologists doubt any more, the universe has a finite size, and a finite mass. Red Act (talk) 12:34, 22 July 2009 (UTC)[reply]
No, that's wrong. Well, the argument is, the claim that the universe is likely finite is not. The big bang is not a simple spatial explosion, i.e. the stuff "over there" was not here and moved away, it was always "over there", and the space in between "here" and "there" expanded - see metric expansion of space. The universe exploded "everywhere simultaneously". Imagine that useful balloon suddenly being expanded from a singular point - at one moment the surface is zero, but as soon as the balloon has a non-zero diameter, all of the universe is there - it just keeps getting diluted as the balloon is inflated. This can just as well work with an infinite balloon - with a tiny bit of extra imagination ;-). --Stephan Schulz (talk) 12:50, 22 July 2009 (UTC)[reply]
The big bang is just an explosion, but it's consistent with a spatially infinite universe because it's somewhat arbitrary what you decide to call "space" (as opposed to spacetime). The Milne universe is a simple example of this. It's a special case of big bang cosmology. The universe is contained in the future light cone of the origin of Minkowski spacetime. But the cosmological time is different from the Minkowski time: it's defined to be , where t and x are the usual Minkowski coordinates. A surface of constant τ—that is, the cosmologist's "space"—is infinite in size and negatively curved, even though the whole universe is within the future light cone of the big bang event (the Minkowski origin) and Minkowski spacetime is flat. This doesn't accurately describe the real world, but the same basic principle applies. If the universe expands forever (i.e., if it's infinite in the time direction) then it might as well be spatially infinite also, it doesn't really matter. -- BenRG (talk) 13:17, 22 July 2009 (UTC)[reply]
Interesting. I can see your point. My mistake. Red Act (talk) 14:39, 22 July 2009 (UTC)[reply]
I agree with the poster above stevebaker for the first two answers stevebaker gave - ie it doesn't follow that an infinite universe contains infinite mass/planets - eg if the distibution is e-x^2 (or many other common equations) the mass/objects in an infinite universe would still be finite. In fact if you assume a 'bigbang' with mass being produced with a statistical distribution of speeds (eg p(v)=e-fn(x) (fn(x)is x or something else), after infinite time (by when the universe is infinitely big) there still will be only finite mass83.100.250.79 (talk) 10:30, 22 July 2009 (UTC)[reply]
As I pointed out above, if there was a big bang, then the universe is finite. No matter how long you wait after the big bang, you will never reach an age of “infinity”. Red Act (talk) 12:38, 22 July 2009 (UTC)[reply]
Red Act, read BenRG's explanation above. There are Bigbang-like models with infinite spacial extent. Dauto (talk) 14:22, 22 July 2009 (UTC)[reply]

ART (Active Release Techniques)

I would like to know more about this massage/chiropractic technique that seems to be emerging recently. I did note there was a limited mention of it on Wikipedia as a sub-heading under different chiropractic techniques. Where did it originate? What principles make it effective? Any other applicable info?

208.125.237.242 (talk) 20:56, 21 July 2009 (UTC)[reply]

Is this something derived from massage parlors of the more prostitutional type, like "Full Release Massage," or does it "release" frozen joints? Edison (talk) 04:29, 22 July 2009 (UTC)[reply]
It is designed to release frozen joints by breaking up adhesions. From what I know so far, it seems it is a newer concept/method in the world of sports massage. It's seems to be popular with runners (per "Running Times article"). [citation needed] From that article I gathered it involved targeting the problem sinew and locking onto it while manipulating the rest of the limb in a manner that simulates normal movement. Any input/clarifications/additions etc? 208.125.237.242 (talk) 14:28, 22 July 2009 (UTC)[reply]

Does this budgie have a disease?

Sorry, but questions calling for a medical or veterinary diagnosis are not allowed here. --Anon, 21:22 UTC, July 21, 2009.

Here's why — please consult a veterinarian. Tempshill (talk) 21:37, 21 July 2009 (UTC)[reply]
I wasn't asking for advice nor would have acted on the discussions here. I was also interesting in starting a Wikipedia article on this disease if it were one.-- penubag  (talk) 21:40, 21 July 2009 (UTC)[reply]

Neomogroside

Does anyone know the chemical formula for Neomogroside? Also, put the source you got it from (I've started writing a Wikipedia article). Dogposter (talk) 21:36, 21 July 2009 (UTC)[reply]

Not a small molecule it seems. All I found was http://www.jipb.net/earticle_read.asp?id=11284 with a name (but not IUPAC), and http://www.lookchem.com/cas-189/189307-15-1.html with a CAS number and half a name. Doing a chemical search on the CAS gave no hits - too new to have been indexed.  Ronhjones  (Talk) 21:46, 21 July 2009 (UTC)[reply]
I suspected it might have been large. So it's a protein? Dogposter (talk) 21:50, 21 July 2009 (UTC)[reply]
No. It sounds like it's a glycoside. The listing on the CAS site linked above gives the following: "â-D-Glucopyranoside,(3â,9â,10R,11R,24R)-11,- 25-dihydroxy-9-methyl-19-norlanost-5-ene- 3,24-diyl bis[O-â-D-glucopyranosyl-(1f2)- O-[â-D-glucopyranosyl-(1f6)]-". So, there's at least a chance of deducing a chemical structure. However, are you sure this meets notability criteria? It is mentioned in the Siraitia_grosvenorii article as one of the compounds identified in that fruit, but does it have any independent importance that merits an article of its own? --- Medical geneticist (talk) 22:30, 21 July 2009 (UTC)[reply]

It was requested for Wikiproject Chemicals. It was on the subpage of requested articles and removed when I made it. Dogposter (talk) 22:41, 21 July 2009 (UTC)[reply]

But anyone can edit that list - it doesn't mean it's notable. SteveBaker (talk) 23:32, 21 July 2009 (UTC)[reply]

Blind

What do blind people "see"? 121.220.109.214 (talk) 22:02, 21 July 2009 (UTC)[reply]

Close your eyes. 90.208.66.95 (talk) 22:11, 21 July 2009 (UTC)[reply]

When you close your eyes, you still see small bits of light. Blind people don't even see that. Dogposter (talk) 22:14, 21 July 2009 (UTC)[reply]

I think that it's probably impossible to know for sure unless you happen to go blind and comprehend what it's like for yourself. It's a bit like asking what blind people's dreams are like - they can tell you, but you can't 'experience' it. --Kurt Shaped Box (talk) 22:55, 21 July 2009 (UTC)[reply]
There are different kinds of blindness. It's my understanding that it can range from people who see a blurry image all the way to people who lack the brain hardware needed to handle vision in any way. So, I guess the answer is anything from "They can almost see" to "They don't even see black." depending on who you're talking about. APL (talk) 23:04, 21 July 2009 (UTC)[reply]
It's a complex question. People blind from birth -- and I mean completely blind, as in lacking eyes -- usually don't describe any visual experience at all. People who became blind at some point after birth usually continue to have visual experiences of some sort, for example visual dreams. And then there is the interesting phenomenon of blindsight. Note that the majority of people who are legally blind are not fully blind, they just don't have vision to a useful degree. Looie496 (talk) 23:36, 21 July 2009 (UTC)[reply]
I suspect that totally blind people, or people whose eyeballs have been removed, still see a splotchy background of random neural activity in the low level brain centers which normally process vision, much as sighted people do when sleeping in a very dark room. This sensation might not be processed or analyzed, and they might deny "seeing" it. Similarly, I hear a constant slight noise even in a totally quiet room. The human sensory system is not noiseless. Edison (talk) 04:25, 22 July 2009 (UTC)[reply]
The sensory system is also good at detecting and supprssing noise. People who are blind from birth never have any non-noise input from the vision centers of the brain, so during the first year or so of life, the brain learns to suppress the signals from those areas. --67.185.15.77 (talk) 07:33, 22 July 2009 (UTC)[reply]
For people who aren't blind from birth, consider Eigengrau. 90.208.66.95 (talk) 11:11, 22 July 2009 (UTC)[reply]
I'm wondering is how old is each website. The newer ones is more accurate than the old ones. I'm not certain if Africa is just moving east or north. Our article said Africa is moving northeast. Is it east-northeast or is it north-east north. All my books is update at diff years. Source will not be helpful if is outdate. I'm not certain of the source Dragon flight gave me is update enough. They have plentys of black arrows. The thing is when plates move the shape of land deforms, and they change shape. Some plate motionss will depend on what part of plate.--69.228.145.50 (talk) 03:12, 21 July 2009 (UTC)[reply]
I'm not entirely sure, but it is being pulled apart on the East African Rift. Dogposter (talk) 00:02, 22 July 2009 (UTC)[reply]
The other thing that bothers me (and which may explain the confusion) is which part of the earth is considered stationary? If (for example) two adjacent plates have an increasingly large gap between them - how do you know whether plate 'A' is moving North or plate 'B' is moving South or both? Because the axis of rotation and the magnetic poles shift around on these kinds of geological timescales - who's to say what is the reference point for these various motions? SteveBaker (talk) 01:11, 22 July 2009 (UTC)[reply]
How about the sun? Noodle snacks (talk) 01:28, 22 July 2009 (UTC)[reply]
Er - what do you mean? SteveBaker (talk) 01:44, 22 July 2009 (UTC)[reply]
I would think the axis of rotation would be the natural reference. Not sure what you mean when you say it shifts around -- with respect to what? Looie496 (talk) 02:16, 22 July 2009 (UTC)[reply]
The axis of rotation also drifts so that's not good enough. Dauto (talk) 04:38, 22 July 2009 (UTC)[reply]
Steve, you would be right if all we had was information about the relative motion between the tectonic plates. There is also available information about the relative motion between the plates and the matle beneath taken for instance from hotspot trails.Dauto (talk) 04:38, 22 July 2009 (UTC)[reply]
Ah - cunning! So it's assumed that the hotspots in the mantle are the reference from which all of these positions are measured? Don't those also move around? It all seems rather arbitary to me...but perhaps if one source for these motions is referring to (say) the spin axis of the Earth as North/South and another is using the hotspot reference points - then that could easily explain all sorts of discrepancies in data from different sources. SteveBaker (talk) 14:56, 22 July 2009 (UTC)[reply]

July 22

Wikipedia articles on planets have little/no mention of the origins of the names of the planets in other languages

Why is it that when I look at the Wikipedia articles on Mercury, Venus, Jupiter, and Saturn, there is not a single mention of the names of the planets in southwest Asian cultures. In fact, the only way to learn them is to use the languages tab beneath the search box? (Mars is the exception, in that it has a mention of them. Planets beyond Saturn are all simply Uranus and Neptune in every language, as far as I can tell). Where can I find information on the names of the planets in southwest Asian cultures? Thanks--12.48.220.130 (talk) 01:26, 22 July 2009 (UTC)[reply]

That's because the European names are the ones universally accepted by astronomers around the world, while the southwest asian (arabic?) names are pretty much forgotten. And why would anyone want to use the planets' arabic names, no-one else would understand what you're talking about. If you want info on the planets' arabic names, just get yourself an english-arabic dictionary. 76.21.37.87 (talk) 01:38, 22 July 2009 (UTC)[reply]

Also, if the encyclopedia had to take the topic of every article and translate it into every language and then explain the origin of the name - it would be a VASTLY bigger undertaking and probably 90% of every article would be quite utterly useless to 99.99% of our readership! I can't think of any other encyclopedia that would do what's being asked here. The versions of Wikipedia that are written in these Asian languages will probably explain what you need. SteveBaker (talk) 01:43, 22 July 2009 (UTC)[reply]
So Anglophones don't need to learn about other names for planets? I mean, there isn't even any mention whatsoever. And seeing as how a lot of astronomy has contributions from that part of the world (zenith, Betelgeuse, Almagest, etc.), I would have thought that it would at least have some brief mention. And looking at the articles, it seems to be victim of systemic bias. --12.48.220.130 (talk) 01:53, 22 July 2009 (UTC)[reply]
Why planets in particular, though? Why is Venus more in need of translations than, say, Toaster?
Yeah - exactly. I'm not a linguist - why the heck would I want to learn the name of any object in another language? It's a total waste of brain cells. Of course if I needed to learn a new language - I'd have to learn the names of very many objects in that language - possibly including the planets. Although, realistically, the names of the planets would be WAY down on my list! First I need to learn how to ask for things like food, drink, hotel rooms and taxis - only after becoming pretty amazingly fluent in the language would I need to know the names of the planets - and long, LONG after that, I might give a damn about how those words came about. But that's why we have dictionaries. Cross-language dictionaries are the way that an English speaker looks up the names of the planets in other languages - that's not the function of an encyclopedia. SteveBaker (talk) 14:28, 22 July 2009 (UTC)[reply]
For a specific example, Venus mentions its name as used by the Mayans, and the Aborigines of Australia. Why include them but exclude southwest Asian names?--12.48.220.130 (talk) 01:59, 22 July 2009 (UTC)[reply]
Without having combed through the page histories of the articles, I'd guess that it's less that the names have been *deliberately* excluded than that no-one has gotten around to including them yet. If you are knowledgeable in this area, go ahead and add whatever you think is appropriate. On WP, doing so is (usually) positively encouraged... :) --Kurt Shaped Box (talk) 02:08, 22 July 2009 (UTC)[reply]
Systemic bias is certainly present on Wikipedia, probably on every language version, but Venus isn't a good example of systemic bias on en. As you know because you read the article, the Mayan name is mentioned in the context of a paragraph on the Mayans having "developed a religious calendar based in part upon its motions, and held the motions of Venus to determine the propitious time for events such as war", and the Maasai mention is ostensibly because of "The Orphan Boy" story about Venus. The former seems like an interesting and notable fact; the latter, less so; but if you have similar facts on hand about southwest Asian astronomy and Venus, feel free to add it. This isn't merely a list of translated names in other languages, which would be very non-notable, as SteveBaker said. The English version of Wikipedia may be shunning your culture in other ways, but I don't think Venus is an instance of this. Tempshill (talk) 02:49, 22 July 2009 (UTC)[reply]

If you look at the left-had side column of those pages you'll find links to wikipedia articles on the subject in other languages, including (typically) Arabic, Persian, Hebrew etc. These will provide you the name of the planets in those languages, assuming that you can read the script. Of course, the quality of these articles varies widely, depending upon the interest editors with the needed language+subject knowledge take in developing them. Abecedare (talk) 02:25, 22 July 2009 (UTC) PS: I now realize that you already know this; ignore my redundant comment.:-) Abecedare (talk) 02:28, 22 July 2009 (UTC)[reply]

I agree with Abecedare; the alternative names such as Arabic names are mensioned for planets that were already known to Arab. For example Mars is named ar:المريخ, Jupiter --> ar:المشتري in the Arabic article.Email4mobile (talk) —Preceding undated comment added 09:30, 22 July 2009 (UTC).[reply]
The obvious answer is that this is the English Wikipedia. The appropriate place for extensive language-specific etymology is on the specific language wikipedia he/she is interested in. The very first page of Wikipedia allows you to select your preferred language. Because of demographics, English wikipedia has the broadest contributor-base, while smaller wikipedias like the Farsi edition have less content and fewer editors. Wikipedia welcomes your cultural and language diversity, as long as your contributions are verifiable. Please feel free to add etymologies to the appropriate articles in the language of your choice. Nimur (talk) 14:55, 22 July 2009 (UTC)[reply]

Balloon chain for solar power?

If I had a length of nylon cord, attached a toy helium balloon, reeled out the cord until the weight of the cord equaled the lift of the balloon, attached another balloon and so on, just how high would it go? Also could my experiment be a hazard to aviation? Suppose I used larger balloons attached to an electric cable-could I float a solar panel above the UK's (near continuous) cloud layer to gain a reliable supply of solar energy, including enough to electrolyze hydrogen to refill the balloons, bearing in mind that using helium would be uneconomic?Trevor Loughlin (talk) 02:15, 22 July 2009 (UTC)[reply]

For the first part of the question, I would assume that the second balloon wouldn't "push" up the first balloon at all. It would simply float up to about the same level as the first balloon. ScienceApe (talk) 02:30, 22 July 2009 (UTC)[reply]
ScienceApe, I think the balloon would go up, because the first balloon is limited by the length of its string, not the air pressure. I wonder if people have tried to make the longest chains they could of toy balloons before. Since it would go so high, I would guess you would need some sort of permit. I'm curious too how high you could make it. Gary (talk) 03:00, 22 July 2009 (UTC)[reply]
It would certainly go up as high as the length of the electric cable, assuming that you got enough balloons attached to lift the weight of the cable. It would sure be a hazard to aviation, though, cause what you got now is pretty much a super-high-altitude barrage balloon and any plane that hits the cable will go down in flames. Also FWiW, I don't think that the balloons could lift a big enough solar panel to make this method of producing electricity worthwhile or cost-effective. 76.21.37.87 (talk) 04:01, 22 July 2009 (UTC)[reply]
Sounds like fun. But the first gust of wind would blow it away. And thin, lightweight electric cables could not carry much electric power. nor could small solar cells which a balloon could support supply much power. Put the solar panels on the ground in the desert where the sun is bright every day and transmit the power by high voltage DC or as synthetic liquid fuel via tanker. Edison (talk) 04:50, 22 July 2009 (UTC)[reply]
One other thing to consider is that as the balloons go up, the air pressure around them shrinks, and the balloons expand. Eventually, if it gets high enough they will burst because they are too large. Googlemeister (talk) 13:19, 22 July 2009 (UTC)[reply]
This is done regularly in the car lots where I live. (Isn't it common everywhere else?) The balloons form a curve. Most will be at nearly the same altitude, but the end attached to the ground will curve down. Some lots take this a bit further and attach both ends to the ground to form an arch. -- kainaw 13:35, 22 July 2009 (UTC)[reply]
That idea is not as silly as it may seem. see Airborne wind turbine. Dauto (talk) 13:48, 22 July 2009 (UTC)[reply]
High-altitude wind power was recently in the news around here as well. Nimur (talk) 15:05, 22 July 2009 (UTC)[reply]

Or, flat-panel satellite dish. How it focus signal? I mean, quotation From Wikipedia, the free encyclopedia: The parabolic shape of a dish reflects the signal to the dish’s focal point. But in flat panel antenna there is no focal point. How come that flat panel antenna still capable to archive good signal to noise ratio? So good, that it could receive satellite programming from different geostationary positioned telecommunication satellites, like ones that broadcasts Freesat? Dose it HAVE to be oriented so broadcasting from satellite reach a dish(opps, panel) with a right angle? Short technical description/explanation would be nice. And it looks like thous panels are actually smaller then regular dishes. Examples:

Sqish
SelfSat’s Flat
Satcom

etc... Vitall (talk) 02:43, 22 July 2009 (UTC)[reply]

The third of those links has the answer: it's phased array technology. However, that article seems to be mainly or entirely about transmitting antennas. Anyway, it must be that the signal is assembled from many small components, vaguely like the way a Fresnel lens does it for light, only because radio waves have longer wavelengths, the phase of the incoming waves is involved. --Anonymous, 03:24 UTC, July 22, 2009.
This might help. Tonywalton Talk 12:09, 22 July 2009 (UTC)[reply]

Pervasiveness of popcorn

What makes the odor of popcorn carry so far? I mean, it seems to carry so much farther than any other odor. Why? Dismas|(talk) 05:50, 22 July 2009 (UTC)[reply]

Plain popcorn, or especially buttered? Butyric acid and some of the other primary components of butter flavor are volatile, have sharp odors, and are detectable at quite low concentrations. DMacks (talk) 07:04, 22 July 2009 (UTC)[reply]
The odor of "popcorn" that most people smell is not popcorn at all. It is the oil. The heavier the oil, the further it travels. For that reason, coconut oil (aka coconut butter) was the most popular oil in movie theaters until the public was told that coconut oil was the deadliest substance in the universe and just being in the same room as some coconut oil would cause an instant heart attack. Now, many theaters use bland, odorless air-popped popcorn. To create the smell, they put coconut oil on a hot plate. The public, in an attempt to enjoy popcorn that tastes like wood shavings, has adopted the practice of smothering the air-popped popcorn with a gallon or so of artificially flavored and artificially colored soybean oil. But, that is going off on a tangent. The odor travels very far because the oil is heavy and it is a smell that we don't smell every day (unless we work in a movie theater). -- kainaw 13:31, 22 July 2009 (UTC)[reply]

Binary poisons

A common form for fictional poisons is a pair of substances, each individually harmless, but which when taken together or in succession are fatal. Do these have any basis in reality? --67.185.15.77 (talk) 07:22, 22 July 2009 (UTC)[reply]

Yes, there are many substances which would be much less harmful if taken apart from exposure to a second substance. See drug interactions. 99.27.134.160 (talk) 07:30, 22 July 2009 (UTC)[reply]
See binary chemical weapon, QL (chemical) and M687. Gandalf61 (talk) 12:00, 22 July 2009 (UTC)[reply]

Distribution of total solar eclipes

1. If one plotted on the same map the paths of all the total solar eclipses over the past 10,000 years, say, would there be places on land that get them a lot more often than other places on land?

2. Given sufficient time, would the entire planet eventually be covered by paths of totality? If so, how long would it take for the entire planet to be covered? If not, where are they never seen, and why? -- JackofOz (talk) 13:38, 22 July 2009 (UTC)[reply]

If you will accept 1,000 years of data instead of 10,000 years, you can look at this diagram, attributed to NASA. Poor resolution in this copy, but you might be able to find the original here. Gandalf61 (talk) 13:48, 22 July 2009 (UTC)[reply]
  1. There are numerically more total eclipses closer to the equator than towards the poles. Do a thought experiment for me: Suppose for one moment that the sun/moon/earth system lay in a plane that sliced right through the equator. So the orbit of the earth around the sun, the orbit of the moon around the earth and the circle of the equator lay in the same exact plane. In that hypothetical 'perfect' solar system, eclipses would ONLY happen in a narrow region close to the equator. However, the moon's orbit is tilted at 5 degrees to the plane that the earth orbits in - and the earth's axis is tilted relative to it's orbit by 23 degrees - so the moon appears to wobble up and down compared to the sun - which results in eclipses happening further to the north and south. Although it takes a particular coincidence of time of year, time of month and time of day for the numbers to all add up in one direction to get that maximum deflection of the path of the eclipse - which explains why the poles are so rarely treated to an eclipse. Because of the angle of the surface of the earth to the cone of shadow cast by the moon - the rarer eclipses that are far from the equator cover a much greater area. So the further north and south you go - the fewer eclipses there are - but the longer they last and the larger the number of people who see them - so if you're standing at some particular spot hoping for an eclipse, the probability of that happening on any given day is a more complicated thing to work out.
  2. The map that Gandalf61 linked to shows that almost everywhere has had an eclipse at some time in the past 1,000 years - over longer periods of time, sooner or later, every point on the earth will have one. Figuring how long it would take for everywhere to get one requires more math than I'm prepared to think about! The fact that neither earth nor moon have perfectly circular orbits - that nothing in the solar system is perfectly lined up - the precession of orbits...it's too complicated for a quick answer. The 1,000 year map suggests that 10,000 to 100,000 years ought to get even the last of the polar regions covered...but it's possible that some odd coincidence of orbital timings might put it out further than that.
SteveBaker (talk) 14:18, 22 July 2009 (UTC)[reply]

molar mass

i'm stuck with this problem. plz anyone help me!

a sample of gaseous substance weighing 0.5 g occupies a volume of 1.12 litres at NTP. calculate the molar mass of the substance.

as 1 mole of any gas at NTP occupies 22.4 litres, there are 0.5 moles in the sample, and that weighs 0.5 grams. so, 1 mole of the sample will weigh 1g. so this will be the molar mass, won't it be?

(but unfortunately, it's not. my teacher gave me a big zero on the left side of my answer)